X



トップページ数学
1002コメント474KB
面白い数学の問題おしえて〜な 40問目
レス数が1000を超えています。これ以上書き込みはできません。
0001132人目の素数さん
垢版 |
2021/12/25(土) 23:47:28.89ID:tw+0E7O3
面白い数学の問題を紹介して解き合うスレです
質問スレではありません
出題者が答えを知らない問題はお控えください
統計学などはスレ違い、数学以外の話題は論外です
荒らし、煽りはスルー推奨

前スレ
面白い問題おしえて〜な 39問目
http://rio2016.5ch.net/test/read.cgi/math/1633923732

過去ログ(1-16問目)
http://www3.tokai.or.jp/meta/gokudo-/omoshi-log/

まとめwiki
http://w.atwiki.jp/omoshiro2ch/
0002132人目の素数さん
垢版 |
2021/12/25(土) 23:48:04.93ID:tw+0E7O3
過去スレ
1 //cheese.5ch.net/test/read.cgi/math/970737952/
2 //natto.5ch.net/test/read.cgi/math/1004839697/
3 //mimizun.com/log/2ch/math/1026218280/
4 //mimizun.com/log/2ch/math/1044116042/
5 //mimizun.com/log/2ch/math/1049561373/
6 //mimizun.com/log/2ch/math/1057551605/
7 //science2.5ch.net/test/read.cgi/math/1064941085/
8 //science3.5ch.net/test/read.cgi/math/1074751156/
9 //science3.5ch.net/test/read.cgi/math/1093676103/
10 //science4.5ch.net/test/read.cgi/math/1117474512/
11 //science4.5ch.net/test/read.cgi/math/1134352879/
12 //science6.5ch.net/test/read.cgi/math/1157580000/
13 //science6.5ch.net/test/read.cgi/math/1183680000/
14 //science6.5ch.net/test/read.cgi/math/1209732803/
15 //science6.5ch.net/test/read.cgi/math/1231110000/
16 //science6.5ch.net/test/read.cgi/math/1254690000/
17 //kamome.5ch.net/test/read.cgi/math/1284253640/
18 //kamome.5ch.net/test/read.cgi/math/1307923546/
19 //uni.5ch.net/test/read.cgi/math/1320246777/
20 //wc2014.5ch.net/test/read.cgi/math/1356149858/
0003132人目の素数さん
垢版 |
2021/12/25(土) 23:49:24.35ID:tw+0E7O3
過去スレ (続き)
21 //wc2014.5ch.net/test/read.cgi/math/1432255115/
22 //rio2016.5ch.net/test/read.cgi/math/1464521266/
23 //rio2016.5ch.net/test/read.cgi/math/1497416499/
24 //rio2016.5ch.net/test/read.cgi/math/1502016223/
25 //rio2016.5ch.net/test/read.cgi/math/1502032053/
26 //rio2016.5ch.net/test/read.cgi/math/1518967270/
27 //rio2016.5ch.net/test/read.cgi/math/1532793672/
28 //rio2016.5ch.net/test/read.cgi/math/1540739963/
29 //rio2016.5ch.net/test/read.cgi/math/1548267995/
30 //rio2016.5ch.net/test/read.cgi/math/1572866819/
31 //rio2016.5ch.net/test/read.cgi/math/1580123521/
32 //rio2016.5ch.net/test/read.cgi/math/1586230333/
33 //rio2016.5ch.net/test/read.cgi/math/1598637093/
34 //rio2016.5ch.net/test/read.cgi/math/1608679703/
35 //rio2016.5ch.net/test/read.cgi/math/1614399625/
36 //rio2016.5ch.net/test/read.cgi/math/1622242743/
37 //rio2016.5ch.net/test/read.cgi/math/1624644393/
38 //rio2016.5ch.net/test/read.cgi/math/1629715580/
39 //rio2016.5ch.net/test/read.cgi/math/1633923732
0004132人目の素数さん
垢版 |
2021/12/25(土) 23:51:12.18ID:tw+0E7O3
俺が立てると言っておきながら規制で2日間も立てられなかった
申し訳ない
0005132人目の素数さん
垢版 |
2021/12/25(土) 23:55:01.03ID:tw+0E7O3
スレタイは前スレで言ったように、タイトルに数学を入れさせてもらった
0006132人目の素数さん
垢版 |
2021/12/26(日) 00:02:25.70ID:sHS00mFY
>>1 乙 テンプレもいい感じ
ではさっそく

実関数 f:R→R であって任意の実数 x,y について
f(x-y) = f(x)f(y) + sinx*siny
を満たすものを全て求めよ
0009132人目の素数さん
垢版 |
2021/12/26(日) 07:04:10.62ID:08zORs9/
f(π/2) = f(π/2)f(0) (∀x), f(0) = f(π/2)^2+1
∴ f(0) = 1 , f(π/2)=0
∴ f(x) = f((x+π/2) - π/2) = sin(x+π/2) = cos(x)
0010132人目の素数さん
垢版 |
2021/12/26(日) 07:16:50.00ID:6nKtf0PS
>>6
f(x)を定数だとすると sinx * siny が定数になるので矛盾
f(x) = f(x)f(0) なので f(0) = 1
f(x)^2 = f(x - x) - sin^2 x
= 1 - sin^2 x
= cos^2 x
-cosx は不適なので f(x) = cosx


類題もどき。
以下の条件を満たすときのf: R -> R, g: R -> Rの性質について調べよ
1. f(x - y) = f(x)f(y) + g(x)g(y)
2. ∃s > 0 s.t. (f(s) = 1 かつ ∀x ∈ [0, s] g(x) >= 0)
0011132人目の素数さん
垢版 |
2021/12/26(日) 08:06:04.98ID:sHS00mFY
>>9
おお、そんな方法が…正解です

>>10
厳密には f(x)=|cosx| 等の可能性も排除する必要あるから満点は出しにくいけど、
まあ正解出てるし修正も面倒じゃないだろうからOKかな…

ちなみに自分は f(0)=1, f(x)^2=cos^2 x, fが偶関数であることまで出たところで
f(x) = f(x/2-(-x/2)) = cos^2(x/2)-sin^2(x/2) = cosx
とするのを想定してました
0012132人目の素数さん
垢版 |
2021/12/26(日) 11:29:52.02ID:sFRnX62i
某大学の入試問題
結構鬼畜
てか高校で複素変数複素数値関数の概念扱わないだろう




【問】  f( x) は複素数平面全体で定義された関数であり,以下の条件(N)を満たすものとする.

*条件(N): Re( f( z) f (w ) )‾ )= Re (z w‾ ) が任意の複素数 z , w に対して成り立つ.

(但し,複素数 α に対して, Re�ソ は α の実部を, α‾ は α の共役複素数を表す.)

(1) 複素数 z の絶対値が 1 ならば, f( z) の絶対値も 1 であることを証明せよ.

(2) 以下の等式を証明せよ.

(a) 任意の複素数 z , w に対して f (z +w) =f( z)+ f( w) が成り立つ.

(b) 任意の実数 r と任意の複素数 z に対して f (r �z) =r�f ( z) が成り立つ.

(3) 絶対値が 1 の複素数 α を用いて,

f( z)= a�z または f (z )=a �z‾

と表せることを証明せよ.
0014132人目の素数さん
垢版 |
2021/12/26(日) 14:55:58.68ID:ivcj9cBc
z=rcis(θ),w=scis(φ),f(z)=r'cis(θ'),w='scis(φ')
のとき
LHS = r's'cos(θ'-φ')
RHS = rscos(θ-φ)
r=0である場合RHSはwについて恒等的に0だからLHSも任意の(s,φ)について0となる必要があり、よってf(0)=0
またr=s,θ=φのときr'=s', θ'=φ'でありr^2=r'^2
よって|z| = |z'|
さらに∠z0w = ∠z'0w'である
g(z)=f(z)/f(1)とおけばg(z)も同じ条件を満たすがこの時さらにg(1)=1
∠i01 = ∠g(i)0g(1), |g(i)| = 1によりg(i) = ±i
前者のときh(z)=g(z), 後者のときh(z) = g(z)^とすればh(x)も同じ性質を有しh(1)=1,h(i) =iである
この時任意のz≠0に対して
|h(z)| = |z|
∠z01 = ∠h(z)01
∠z0i = ∠f(z)0i
によりh(z) = z
0015132人目の素数さん
垢版 |
2021/12/26(日) 16:10:07.06ID:5VnkCav4
>>12
(1) w=z とすれば|f(z)|^2=|z|^2となるので、|f(z)|=|z|である。
特に|z|=1のとき、|f(z)|=1 である。

(2) a,b∈C に対して|a+b|^2=|a|^2+|b|^2+2Re(a \bar{b}) であり、
右辺には Re(a\bar{b}) という項が出現している。この項を利用すると、z,w∈C と r∈R に対して

|f(z+w)−f(z)−f(w)|^2 = …(地道に全て展開)… = 0,
|f(rz)−rf(z)|^2 = …(地道に全て展開)… = 0

が示せて、(a),(b)が成り立つ。

(3) x,y∈R に対して f(x+iy)=f(x)+f(iy)=xf(1)+yf(i) なので、
f(1)とf(i)の値を詳しく見ればよい。
f(1+i)=f(1)+f(i) なので、|f(1)+f(i)|=|f(1+i)|=|1+i|=√2 である。
また、|f(1)|=1=|f(i)|なので、f(1)=e^{ia}, f(i)=e^{ib}, a,b∈R と表せる。
よって、|e^{ia}+e^{ib}|=√2 である。両辺を2乗すると、2+2Re(e^{i(b−a)})=2 なので、
Re(e^{i(b−a)})=0 となり、e^{i(b−a)}=±i となる。
e^{i(b−a)}=iのときは、x,y∈R に対して f(x+iy)=xf(1)+yf(i)=e^{ia}(x+iy) となる。
これは f(z)=e^{ia}z (z∈C) を意味する。
e^{i(b−a)}=−iのときは、x,y∈R に対して f(x+iy)=xf(1)+yf(i)=e^{ia}(x−iy) となる。
これは f(z)=e^{ia} \bar{z} (z∈C) を意味する。
0018132人目の素数さん
垢版 |
2021/12/26(日) 19:39:04.68ID:MtCSJ91m
毎回湧いてくるバカのためにわざわざスレタイに数学を入れなきゃならんとは
0019132人目の素数さん
垢版 |
2021/12/26(日) 22:04:41.68ID:Ini2anRA
勝手にスレタイを変えるなよ
いまさら変えても意味ないやろ
0020132人目の素数さん
垢版 |
2021/12/26(日) 22:04:59.38ID:sFRnX62i
>>17
どこかで見た(3)を直接示す解法


{ w f(z)−z f(w) } { w‾ f(z)−z‾ f(w) } = 0
を示す

こんなん思いつく訳ないわ
0021132人目の素数さん
垢版 |
2021/12/26(日) 22:08:06.00ID:LCDqRPw5
ゆとりが始まる前の世代なら
A(z),B(w)のとき
OA→•OB→=re(zw^)
は受験の必須公式だったんだけどな
0022132人目の素数さん
垢版 |
2021/12/26(日) 23:06:47.09ID:Fh2z24BT
次の条件を満たす二次元平面上の図形は何か?

(1)図形の全ての長さは有限

(2)二次元平面上のどの円(半径、中心は自由)内とも図形との共通部分があり、かつその円内での図形の長さは0より大きい
0023132人目の素数さん
垢版 |
2021/12/26(日) 23:24:27.34ID:5VnkCav4
うおおお>>6が一般化できた。

f,g:R → R は写像で、g(0)=0 かつ f(x−y)=f(x)f(y)+g(x)g(y) (∀x,y∈R) とする。
このとき、f(x)=g(x)=0 (∀x∈R) または f(x)=1−2g(x/2)^2 (∀x∈R) であることを示せ。
0026132人目の素数さん
垢版 |
2021/12/27(月) 00:12:52.03ID:RLCBegf6
>>24-25
それは必ずしも成り立たない。f(x) = cos(3x), g(x) = sin(3x) のとき、

f,g:R → R, g(0)=0, f(x−y)=f(x)f(y)+g(x)g(y) (∀x,y∈R)

が成り立つことが確かめられるが、
f(x) = cos(x) となる x は [0,2π] の範囲に有限個しかない。
0027132人目の素数さん
垢版 |
2021/12/27(月) 00:35:16.17ID:Y6GwOFls
>>23
同じ仮定で任意のx,yについて g(x+y) = f(x)g(y) + g(x)f(y) が出せるな
あとは F(x)=f(x)+ig(x) と置いてごにょごにょしたら色々導けそう
0028132人目の素数さん
垢版 |
2021/12/27(月) 06:24:55.38ID:dee73DLH
これは数字が現れないけど、広義の数学の問題なのか?
まあ、面白いと感じるかどうかは個人差があるだろうけど。

某女子大には決して嘘をつかない女子大生と必ず嘘をつく女子大生がいることがわかっている。
この女子大の学生(嘘つきかどうかは不明)から
「あなたのいうことが正しければ手コキかフェラをしてあげる、間違っていれば何もしてあげない」と言われた。
女子大生にフェラをしてもらうには何と言えばいいか?
0029132人目の素数さん
垢版 |
2021/12/27(月) 06:28:16.06ID:dee73DLH
(1) サイコロを100個振ったときの目の和が100である確率を求めよ。
(2)サイコロを100個振ったときの目の和が101である確率を求めよ。
(3)サイコロを100個振ったときの目の和が102である確率を求めよ。
(3)サイコロを100個振ったときの目の和が期待値になる確率を求めよ。

サイコロの目のでる確率はどの目も完全に等しいというのは現実離れした想定であるので、いずれも概算値でよい。
0030132人目の素数さん
垢版 |
2021/12/27(月) 09:02:16.78ID:YtemeARj
>>16
△ABCの内心をI、AIとIにおいて直交する直線とAB,ACの交点をD,Eとする
BD=2,CE=3であるときDEを求めよ
0032132人目の素数さん
垢版 |
2021/12/27(月) 09:20:24.53ID:2qfEh3DC
スレタイ無視の荒らしは運営に通報した方がいいな
0033132人目の素数さん
垢版 |
2021/12/27(月) 12:31:50.10ID:sQZhwpNd
((x))をxの小数部分とする
∫_0^1 ((1/x)) dx を求めよ.
0034132人目の素数さん
垢版 |
2021/12/27(月) 12:54:39.97ID:RLCBegf6
>>27の指摘をもとに、更なる一般化。

f,g:R → R は写像で、g(0)=0 かつ f(x−y)=f(x)f(y)+g(x)g(y) (∀x,y∈R) とする。
このとき、次の(i),(ii)のうちいずれかが成り立つことを示せ。

(i) f(x)=g(x)=0 (∀x∈R) である。

(ii) ある写像 θ:R → R が存在して、θ(0)∈Z, θ(x+y)−θ(x)−θ(y)∈Z (∀x,y∈R),
f(x) = cos(2πθ(x)), g(x) = sin(2πθ(x)) (∀x∈R) である。

また、(ii) が成り立つ場合、f(x)=1−2g(x/2)^2 (∀x∈R) が成り立つことを示せ。
0035132人目の素数さん
垢版 |
2021/12/27(月) 13:44:08.39ID:REGf6dBj
>>33
Σ[k=1,N]∫[1/(k+1),1/k](1/x-k)dx
=log(N+1) - Σ[k=1,N]1/(k+1)
→1-γ
0036132人目の素数さん
垢版 |
2021/12/27(月) 14:30:54.42ID:sQZhwpNd
>>35
素晴らしい
正解です
0037132人目の素数さん
垢版 |
2021/12/27(月) 14:31:51.42ID:dbdDCsGM
サイコロをn個(nは偶数)投げて出た目の和が期待値になる確率は
n=2のとき0.167
n=4のとき0.113
n=6のとき0.0928
n=8のとき0.0809
確率が5%未満になるのはnがいくつ以上のときか?
0038132人目の素数さん
垢版 |
2021/12/27(月) 14:33:20.07ID:dbdDCsGM
期待値という名称は誤解を招く命名だと思う。
期待したのに期待外れだから。
0039132人目の素数さん
垢版 |
2021/12/27(月) 14:37:03.26ID:dbdDCsGM
有意水準とか危険率だと5%以下だと稀な現象と判断する。
>サイコロを100個振ったときの目の和が期待値になる確率を求めよ。
期待値というから稀じゃないと思って期待したら期待外れだった。
0040132人目の素数さん
垢版 |
2021/12/27(月) 15:00:27.38ID:dbdDCsGM
ある幼稚園で、砂場で遊んでいたA,B,C,D 部屋で遊んでいたE,F,Gの7人の中に、
逆上がりができる子が2人いることが分かっている。

そこで、A〜Gに尋ねたところ、それぞれ以下の発言をした。
ただし、7人うち、本当のことを言っているのは2人だけで、あとの5人は間違ったことを言っていた。これらのことから確実にいえるのはどれか。

A:Bは逆上がりできるよ。
B:Aは間違ったことを言っているよ。
C:AもBも2人とも間違ったことを言っているよ。
D:砂場で遊んでいた子の中には逆上がりできる子はいないよ。
E:私は逆上がりできない。
F:逆上がりができるのは2人とも砂場で遊んでいた子だよ。
G:EとFの少なくともどっちかは本当のこと言っているよ。
0043イナ ◆/7jUdUKiSM
垢版 |
2021/12/27(月) 17:11:28.35ID:RbkoMXJU
>>29
(1) サイコロを100個振ったときの目の和が100である確率は1/6^100
(2)サイコロを100個振ったときの目の和が101である確率は100/6^100=25/(3^2・6^98)=25/(9・6^98)
(3)サイコロを100個振ったときの目の和が102である確率は(100+9900)/6^100=(2^2・5^2)/(2^100・3^100)=25/(2^98・3^100)
(3)サイコロを100個振ったときの目の和が期待値350である確率は365である確率よりは高い。
0044イナ ◆/7jUdUKiSM
垢版 |
2021/12/27(月) 17:24:53.32ID:RbkoMXJU
>>43
>>40
BとEは逆上がりができるんじゃないかなぁ?
先生の気持ち的に。
0046132人目の素数さん
垢版 |
2021/12/27(月) 18:04:30.30ID:Oxd4yvSC
3柱の神様A、B、Cは、それぞれ誰かが真神、偽神、乱神です。真神は必ず真実の答えを、偽神は必ず嘘の答えを言いますが、乱神が真実を答えるか嘘を答えるかは完全にランダムです。あなたは3つのイエスかノーかで答える質問をして、A、B、Cの正体(真神か偽神か乱神)を決めてください。1つの質問には1柱の神様しか答えてくれません(こちらで質問先A、B、Cの指定は可能)。神様は私たちの言葉を理解しますが、返答は私たちの言葉ではなく、神の言葉「Da」と「Ja」で返します。DaとJaのどちらがイエスでどちらがノーを意味するかは分かりません。
0049132人目の素数さん
垢版 |
2021/12/27(月) 18:36:28.88ID:Oxd4yvSC
>>48
そうだけどなんでそんなに怒るん?
ちゃんとした数学と問題だと思うけど?
0052132人目の素数さん
垢版 |
2021/12/27(月) 19:56:12.27ID:Oxd4yvSC
>>51
問題の意味もとれてないねぇ
0054132人目の素数さん
垢版 |
2021/12/27(月) 20:04:08.76ID:dbdDCsGM
サイコロをn個(nは偶数)投げて出た目の和が期待値になる確率は
n=2のとき0.167
n=4のとき0.113
n=6のとき0.0928
n=8のとき0.0809
確率が5%未満になるのはnがいくつ以上のときか?
0055132人目の素数さん
垢版 |
2021/12/27(月) 20:06:41.24ID:Oxd4yvSC
>>53
答え貼られた問題になんでいつまでも固執するん?
そういうのが迷惑なんだよ
0056132人目の素数さん
垢版 |
2021/12/27(月) 20:11:25.17ID:Oxd4yvSC
全前スレでアホにながされた問題

平面上のPL-Jordan閉領域Dの境界J=∂D上に4点をとりJを4つの閉線分にわけて正の向きに順にR,T,L,Bとする
Jordan経路PがL∩P≠φ、R∩P≠φを満たすときPはDを横断すると呼び、T∩P≠φ、B∩P≠φを満たすときPはDを縦断すると呼ぶとする
Dのその2つのPL部分集合による被覆D=X∪Yを取る
このときXがDを横断する経路を持つか、またはYがDを縦断する経路を持つかのいずれかが成立する事を示せ
0059132人目の素数さん
垢版 |
2021/12/27(月) 20:22:00.64ID:Oxd4yvSC
>>58
自分で考えるのは勝手にすればいい
それをここに書くからみんな迷惑するんだよ
一生懸命数学の勉強して、実力つけて、それなりに面白い問題頑張って作って答えも用意して、それがそういうしょうもない、くっだらないレスにどんどん流されて言ってる時の気持ちわかる?
自分がどれだけ数学的になんの意味もない迷惑なだけのレスを貼り続けてるかわからんの?
時にはすごい面白い名作が並んでるのわかる?
みんなそれをどれだけ頑張って準備してるのか想像できんの?
0061132人目の素数さん
垢版 |
2021/12/28(火) 00:58:15.66ID:LoE9TaIF
〜このスレの皆さんへ〜
現在、無意味なプログラムを書き込む悪質な荒らしが常駐しています
通称「プログラムキチガイ」「害悪プログラムおじさん」は医療・医者板にいる尿瓶、脳内医者と呼ばれている荒らしです
https://egg.5ch.net/test/read.cgi/hosp/1607687111/

数学Iの三角比の問題や中学数学の平面図形の問題でさえ手計算では解けずに
わざわざプログラムで解くような人物です
二項分布の期待値npすら知らないレベルです
かなり低レベルの数学の問題もどきを出題してマウントを取りに来ます
下ネタが大好きです
0062132人目の素数さん
垢版 |
2021/12/28(火) 05:54:59.28ID:wS7aPNDN
>>41
期待という日本語にはpositiveな含蓄がある。期待される人間というと没落が期待されてはいないと思う。
hot waterに違和感はないが熱い水というのは違和感を覚える。
expcted lossを期待損失と訳すのは熱い水と同じ違和感を感じる。

こうのが理解できない気の毒な頭のようである。
0063132人目の素数さん
垢版 |
2021/12/28(火) 06:01:12.00ID:wS7aPNDN
>>59
面白いと考えた人が、サイコロを100個振ったときの目の和とか論理パズルにレスしてんじゃないの?
>30の問題って数学的に何か意味があんの?
俺は解くのが面白いから作図して計測した。
まあ、プログラムで方程式の数値解を出して座標を計算して長さを計測するというのをやっただけだが。
0064132人目の素数さん
垢版 |
2021/12/28(火) 06:02:51.08ID:wS7aPNDN
>>57
ワクワク期待していたのに、期待どおり(期待値どおり)になる頻度が2%とは期待外れと体感できた。
0065132人目の素数さん
垢版 |
2021/12/28(火) 06:46:28.78ID:wS7aPNDN
脱字補充

>>41
期待という日本語にはpositiveな含蓄がある。期待される人間というと没落が期待されてはいないと思う。
hot waterに違和感はないが熱い水というのは違和感を覚える。
expected lossを期待損失と訳すのは熱い水と同じ違和感を感じる。

こういうのが理解できない気の毒な頭のようである。
0066132人目の素数さん
垢版 |
2021/12/28(火) 06:54:05.58ID:6cL0voRz
>>63-65
専用のスレを作ったので、
今後、そういう問題は以下のスレに書き込んでください。

数値解析の問題を書き込むスレ

https://rio2016.5ch.net/test/read.cgi/math/1640641452/
>主に数値解析を中心とする問題を紹介して解き合うスレです

>数学的な厳密解が求まりそうになく、
>プログラム・シミュレーション等で概算値を出せれば十分、
>という問題が中心となります

>ただし、普通の数学の問題もOK
0068132人目の素数さん
垢版 |
2021/12/28(火) 08:39:15.56ID:4kktpeCR
>>63
そういう言い訳もいらん
お前自分で数学の勉強した記憶ないやろ?
数学勉強してるした人間が何を面白いと思うのか、何が参考になるのかなんかお前みたいな能無しのカスにわかるわけないやろ?
黙ってロムっとけ
能無し
0069132人目の素数さん
垢版 |
2021/12/28(火) 08:59:53.61ID:KV1243AX
実関数f:R→Rであって任意の実数x,yについて
f(x+y)=f(x)cosy+f(y)cosx
を満たすものを全て求めよ
0070132人目の素数さん
垢版 |
2021/12/28(火) 11:23:11.25ID:wS7aPNDN
専門用語が一般用語のconnotationから外れる。
医学用語にはそういうのが多いな。合併症とか縫合不全とか術者の責任を回避するような用語がめだつと俺は感じている。
0072132人目の素数さん
垢版 |
2021/12/28(火) 11:31:09.86ID:9DyNMn+s
そういえばめちゃイケのテストで突然ひらがなが混じるのがバカの特徴みたいなこと言われてたなww
案外当たってるかも
0075132人目の素数さん
垢版 |
2021/12/28(火) 11:54:44.33ID:wS7aPNDN
さっそく罵倒厨がこういう投稿をしていて笑える
数学スレにはこういうのが多いね。

数値解析の問題を書き込むスレ
https://rio2016.5ch.net/test/read.cgi/math/1640641452/5

5 名前:132人目の素数さん[sage] 投稿日:2021/12/28(火) 10:53:21.01 ID:V7fjP+Uc
>>1
よそでやれ


つまり、ここでやれとの「助言」だわなwww

さて、ベンチタイムが終わったので成形して二次発酵にとりかかろう。
0077132人目の素数さん
垢版 |
2021/12/28(火) 12:04:05.53ID:KV1243AX
>>70
関係ない雑談の頻度が過ぎてると思うから雑談はこっちでやってね
雑談はここに書け!【61】
https://rio2016.5ch.net/test/read.cgi/math/1637315946/

あと >>75 あんたの問題が統計スレ
https://rio2016.5ch.net/test/read.cgi/math/1640641452/5
の方が適している事実は変わらないから今後はこっちに投稿すること

あんたの問題が面白くない(それにしつこい)と感じている人が大勢いるんだから
せめてコテつけてこっちが自衛できるようにするぐらいの配慮はしてよ
0078132人目の素数さん
垢版 |
2021/12/28(火) 14:26:33.91ID:QkgHFURT
連続関数f:R→Rついて、
任意のx>0に対して、数列{f(x),f(2x),f(3x),...}が0に収束するとき、
lim(t→∞)f(t)=0を示してください.
0080132人目の素数さん
垢版 |
2021/12/28(火) 15:08:48.73ID:QkgHFURT
>>79
ああ見てたのか
その場合は(・∀・)ニヤニヤでお願いします
0082132人目の素数さん
垢版 |
2021/12/29(水) 00:17:57.78ID:fiSnwSek
>>69
実数 z を任意に取り、x=z−π/2, y=π/2 として、

f(z)=f(z−π/2)cos(π/2)+f(π/2)cos(z−π/2)=f(π/2)sin(z)

これが任意の z で言えるので、結局、ある λ∈R に対して
f(x)=λsin(x) (∀x∈R) となる。逆に、このように表せる f は与式を満たす。
0083132人目の素数さん
垢版 |
2021/12/29(水) 00:51:58.85ID:it6W4Lm6
>>78
できた
閉集合Fnを
Fn = { x > 0 ; ∀m≧n |f(nx)|≦ε }
で定める
仮定により(0,∞) = ∪Fn である
ベールのカテゴリー定理により少なくとも一つのFnは内点を持たなければならない
(a,b)⊂Fnとする
n0>nを1+1/n0<b/aと選べば任意のm≧n0に対して(m+1)a<mbである
特に
∪[m≧n0](ma,mb) = (n0a,∞)である
よってx>n0aのときm>n0とy∈(a,b)をx=myと取れるがこの時|f(x)|=|f(my)|≦εである
0085132人目の素数さん
垢版 |
2021/12/29(水) 07:42:29.67ID:ErlZeHnO
>>83
素晴らしいです お見事大正解
まさしく閉集合族F_nをそう置いてベールの範疇定理を使うのがミソでした

元ネタ
数学詳しいやつ来て!!!!!
https://mi.5ch.net/test/read.cgi/news4vip/1640629371/

この>>1は一週間掛けて何も出来なかったと言っていたのに、たった数時間で解けるとはさすがですね
0087132人目の素数さん
垢版 |
2021/12/29(水) 08:23:28.15ID:UW0jVwTy
もしか
こっち仮定したら被服定理証明できたりは?
さすがに無理かな
0088132人目の素数さん
垢版 |
2021/12/29(水) 08:27:16.61ID:ErlZeHnO
>>87
それはどうなんだろう

あと個人的に気になるのは
連続性を課さなかったら>>78の反例が出来るのか?
ということですね
0089132人目の素数さん
垢版 |
2021/12/29(水) 08:37:20.19ID:nGsGIQe9
>>82 正解!シンプルでいいね

>>88 連続じゃない時の反例は例えば
f(x)=1 (ある整数mについて x=e^m を満たす時),
f(x)=0 (それ以外)
があるんじゃないかな

個々の x>0 について f(xn)>0 を満たす正の整数nは高々一つだから
lim(n→∞) f(xn)=0 は成り立つが、一方
lim(x→∞) f(x)=0 は成り立たない
0090132人目の素数さん
垢版 |
2021/12/29(水) 08:38:44.54ID:1zmj0/3Y
連続性がなければいくらでも反例ありますがな
x1,x2,....
をQ上線形独立、limxi=∞ととって
f(x) = 1 ( ∃i x = xi )
0 ( otherwise )
で反例
0091132人目の素数さん
垢版 |
2021/12/29(水) 08:40:01.81ID:1zmj0/3Y
被ったorz
0092132人目の素数さん
垢版 |
2021/12/29(水) 08:45:40.97ID:UW0jVwTy
>>88
さすが無理よな
けんど
もそっと弱いとこからの証明があってしかるべしかとそう思てさ
0093132人目の素数さん
垢版 |
2021/12/29(水) 08:50:37.01ID:ErlZeHnO
>>89
>>90
すごい ありがとうございます
あーなるほど整数刻みであることを利用して
「複数のnでf(nx)≠0とはならない」ことを作ればいいのか 素晴らしい
クソーこういう反例をパッと思い浮かべるようになりたいな
0095132人目の素数さん
垢版 |
2021/12/30(木) 00:00:11.11ID:R3+2KyyT
>>94
その心は?
0096132人目の素数さん
垢版 |
2021/12/30(木) 00:28:41.32ID:BWxUHjcd
数列
1,3,8,21,55,144,377,…
(3倍して前の数を引いたやつが次の数)

数列
1,3,10,33,109,360,1189,…
(3倍して前の数を足したやつが次の数)
の平均が
>>84の数列
1,3,9,27,82,252,783,…
になる
9項目を求めると7777
0097132人目の素数さん
垢版 |
2021/12/31(金) 02:18:53.08ID:JZkFcsei
素数の有限集合で相加平均が2022であるものの元数の最大値を求めよ

#計算機可
#見たぞの人は(・∀・)ニヤニヤで
0099132人目の素数さん
垢版 |
2021/12/31(金) 10:20:00.11ID:QNczF9a7
区別のつかない100個の玉を区別のつかない5個の箱に入れる。
どの箱にも最低1個の玉はいれるものとする。
何通りの入れ方があるか?
0100132人目の素数さん
垢版 |
2021/12/31(金) 10:49:30.88ID:QNczF9a7
お年玉100万円を1万円単位で6人で分ける分け方は何通りあるか?
お年玉をもらいない人がいてもよいものとする。
0101132人目の素数さん
垢版 |
2021/12/31(金) 11:00:36.10ID:hsGyiNoM
119 132人目の素数さん[sage] 2021/12/31(金) 10:50:19.35 ID:QNczF9a7
お年玉100万円を1万円単位で6人で分ける分け方は何通りあるか?
誰も最低1万円はもらえるものとする。

マルチすんなゴミ
0102132人目の素数さん
垢版 |
2021/12/31(金) 11:28:39.39ID:fwwqbJxg
しんじろう 「これ意外と知らない方が多いんですけど
素数ってほとんどが奇数なんですよね」

↑ これは真であるか偽であるか?
0105132人目の素数さん
垢版 |
2021/12/31(金) 19:25:25.54ID:5g7zFcBq
a_i,j(1≦i,j≦n)が
任意の実数x_i,y_i(i=1,2,…,n)に対して
Σ(1≦i≦n)(Σ(1≦j≦n)(a_i,j・x_j))y_i=0
を満たすならば
a_i,j=0であることを示せ
0106132人目の素数さん
垢版 |
2021/12/31(金) 19:52:12.83ID:mhAv9qAm
>>104
ヒントおながいします
0108132人目の素数さん
垢版 |
2021/12/31(金) 20:41:00.34ID:VaU9bGPw
>>106
愚直だけど sin(sin(x)) をテイラー展開してそれぞれの項を評価する感じ
多分πの無理数度に関する結果も使うと思う
0111132人目の素数さん
垢版 |
2021/12/31(金) 20:57:18.73ID:fwwqbJxg
>>110
それって前提として
「自然数のうち、偶数でないものは奇数である」
っていうのが必要だよね?
それって証明されているの?
奇数でも偶数でもない素数が存在する可能性はどうなんだ?

自然数は「偶数、奇数、そのどちらでもない」
の3つの集団に分けられると考えてみよう。
そうすると、
「素数のほとんどが奇数」という表現はおかしくない。
0112132人目の素数さん
垢版 |
2021/12/31(金) 21:11:58.21ID:mhAv9qAm
>>108
ダメだ
大先生も教えてくれない
sin(sin(x))のフーリエ級数展開だけでも教えて下さい
0113132人目の素数さん
垢版 |
2021/12/31(金) 21:19:27.11ID:mhAv9qAm
>>108
ダメだ
円周率の無理数度も上からの評価がちょこちょこあるだけでどう使えばいいかサッパリわからない
もう少しヒントを
0114132人目の素数さん
垢版 |
2021/12/31(金) 22:08:31.39ID:VaU9bGPw
>>112 >>113
フーリエ係数 sin(sin(x)) = Σ_(m=1,∞) c_m・sin(mx) は厳密な値は必要じゃなくて、
任意の定数 r>0 について c_m=O(m^(-r)) (k→∞) であることだけで十分

何ならπの無理数度も有限であることしか使わないと思う

証明の始めは
|a_n| = |Σ_(k=1,n) sin(sin(k))|
= |Σ_(k=1,n) Σ_(m=1,∞) c_m sin(km)|
≦ Σ_(m=1,∞) c_m |Σ_(k=1,n) sin(km)|
…とこんな感じになるかな
0115132人目の素数さん
垢版 |
2021/12/31(金) 22:44:43.38ID:mhAv9qAm
>>114
え?
じゃあフーリエ級数展開を持つ任意の周期2πの関数でいえるって事?
0116132人目の素数さん
垢版 |
2021/12/31(金) 22:49:58.72ID:mhAv9qAm
ともかくフーリエ級数展開して
Σ[k,m]sin(sin(k)) = Σ[k,m]c_m sin(km)
にしてc_mは連続関数のフーリエ係数である事用いてある程度評価できてもm固定してk走らせた方の和
Σ[k]sin(km)
が評価できない
kが大きくなるとアーベルプラナも使えないし
0117132人目の素数さん
垢版 |
2021/12/31(金) 22:52:55.33ID:l6zLAtwi
>>104
解答の筋道はわかった。

sin(sin(x)) = a[1] sin(x) + a[3] sin(3x) + a[5] sin(5x) + ...
この係数はベッセル関数の母関数より
a[m] = 2*BesselJ[m](1)

〇係数の評価
ベッセル関数の積分表示より
|BesselJ[m](1)|
= (1/2)^m/(√πΓ(m+1/2))|∫[0,π]cos(cos(x))sin(x)^(2m)dx|
≦√π/(2^m Γ(m+1/2))

〇項別級数の評価
|Σ[k=1,n]sin(mk)|
=|sin(mn/2)sin(m(n+1)/2)/sin(m/2)|
≦1/|sin(m/2)|

したがって
|Σ[k=1,n]sin(sin(k))|
≦Σ[j=0,∞] √π/(2^(2j) Γ(2j+3/2)|sin(j+1/2)|)
あとは円周率の無理数度の評価からjの和が有界になることを示せばよい。
0118132人目の素数さん
垢版 |
2021/12/31(金) 22:53:33.22ID:VaU9bGPw
>>115
いやいや、sin(sin(x)) の場合はC^∞級だし、
関数の平均 ∫_[0,2π] sin(sin(x))dx の値が0だからね
不連続な関数とかだとこうはいかない
0119132人目の素数さん
垢版 |
2021/12/31(金) 23:00:01.66ID:mhAv9qAm
>>114
πの無理数度が有限というのは

ある定数c>0が存在して任意の自然数p,qに対して

|π-p/q| >1/q^c

が成立する

で合ってる?
フーリエ級数展開を持つ一般の関数について言えてる性質と上の事実だけから示せるで間違いない?
0120132人目の素数さん
垢版 |
2021/12/31(金) 23:14:16.64ID:mhAv9qAm
あ、そうか
sin(mk)cos(k)
= 1/2(sin(mk+k/2) -sin(mk-k/2))
でΣ[k]sin(mk)はexplicitに計算できるやん
なるほど
でsin(sin(x))の方はL^ ∞だから相方の級数はl^1に入ってるのか
0121132人目の素数さん
垢版 |
2021/12/31(金) 23:15:29.82ID:VaU9bGPw
>>117
そういうこと!
あとは無理数度を使うくだりかな

続きを考えたい人のために捕捉
πの無理数度の有限性から、1/πの無理数度の有限性も導けることに注意。
あと、実数 r に一番近い整数 n をとれば |sin(πr)|≧2|r-n| という評価も得られる。
これらを使えばある実数 A,B>0 が存在して、正の整数jについて
1/|sin(j/2)| ≦ Aj^B
が成り立つことを導ける

>>119
うん。知識としてはC^∞関数のフーリエ係数のオーダーと、
πの無理数度の有限性だけで足りるはず
0123132人目の素数さん
垢版 |
2021/12/31(金) 23:33:48.04ID:mhAv9qAm
>>121
うん、今考えててそこまでは行った
相方こフーリエ級数の方がl^1に入ってるからもう片方の1/cos(km)がmについて有界とかならl^1×l^∞でl^1に入ってくれる
しかし1/cos(km)の上から評価ぎできん
inf { | l π/2 - km | ; l } とかを下から評価することになるけど無理数度有限から言えるのはjをとって
| π/2 - km/l | > l^j
という評価ぎ得られるだけで
ここから1/cos(km)をどうやって上から評価していいか分からん
フーリエ級数の方はl^1以上の援助は望めないよね?
0124132人目の素数さん
垢版 |
2021/12/31(金) 23:50:12.76ID:VaU9bGPw
>>123
1/|sin(k/2)|のこと?
>>117 で具体的に提示してくれた通り、
係数はl^1である以上に指数関数的な減少をしてくれるから
1/|sin(k/2)|を多項式で抑えれば収束性は十分示せると思うけど
0125132人目の素数さん
垢版 |
2022/01/01(土) 01:39:10.53ID:vRdsvE8n
>>124
うん、示せた
やはり最低でもsin(sin(x))のような筋のいい関数でないとダメやね
外のsinをマクローリン展開して
Σ[u:odd](-1)^((u-1)/2)1/u!(sin(x))^u
でm次のフーリエ展開の係数はこの係数に(sin(mx),(sin(x)^u) (ただし(,)はL^[-π,π]の内積、sin(mx)の長さが1になるように取っておく)をかけてu≧mで足し合わせるけど(sin(mx),(sin(x)^u) の発散より分母のu!の方が強くで和も1/m!より小さくなるから桶やね
Σ[k]sin(km)が初等的に足せるのうっかりしてた
0126132人目の素数さん
垢版 |
2022/01/01(土) 05:48:52.18ID:sn/pTPRs
A⊂[0,1]^2 はボレル可測で、その2次元ルベーグ測度は正とする。
このとき、ある異なるx_1,x_2∈[0,1]と、ある異なるy_1,y_2∈[0,1]が存在して、
(x_1,y_1),(x_1,y_2),(x_2,y_1),(x_2,y_2)∈A が成り立つことを示せ。
0129132人目の素数さん
垢版 |
2022/01/01(土) 10:28:06.75ID:sn/pTPRs
せっかくだから清書し直すわ。

R^2のルベーグ測度をμと置く。A⊂[0,1]^2 はボレル可測で、μ(A)>0 とする。
このとき、ある異なるx_1,x_2∈[0,1]と、ある異なるy_1,y_2∈[0,1]が存在して、
(x_1,y_1),(x_1,y_2),(x_2,y_1),(x_2,y_2)∈A が成り立つことを示せ。
0130132人目の素数さん
垢版 |
2022/01/01(土) 10:56:44.15ID:AINN/vbo
>>129
X = R×R×R×Rの座標関数をx1,x2,y1,y2として閉集合Fを{ x1=x2 or y1=y2 }とする
F(x1,x2,y1,y2) = | x1-x2||y1-y2|とする
閉区間の列In,Jn,Kn,Lnと正の数の列cnを
∪In×Jn×Kn×Ln = X\F, p∈∪In×Jn×Kn×Ln→F(p)>cn
ととっておく
仮定から少なくとも一つのnで
μ(A×A×A×A ∩ In×Jn×Kn×Ln) > 0
であるからこの時∫[A×A×A×A]f(p)dp > 0である
よって特にA×A×A×Aは0ではない
f(p)≠0であるA×A×A×Aの点p=(x1,x2,y1,y2)をとればこれが条件を満たす
0131132人目の素数さん
垢版 |
2022/01/01(土) 11:13:16.52ID:sn/pTPRs
>>130
>μ(A×A×A×A ∩ In×Jn×Kn×Ln) > 0

何かおかしい。次元が一致してない。
A⊂[0,1]^2 なので、A×A×A×A ⊂ R^8 となるが、
In,Jn,Kn,Ln は R の閉区間なので、In×Jn×Kn×Ln ⊂ R^4.
0132132人目の素数さん
垢版 |
2022/01/01(土) 11:41:58.61ID:AINN/vbo
あ、ホントだ
0133132人目の素数さん
垢版 |
2022/01/01(土) 12:07:05.90ID:yOpHBm/c
>>129
対偶を示す。つまり、A⊂[0,1]^2 をルベーグ可測な集合とする時、
もし異なる x1,x2∈[0,1] と異なる y1,y2∈[0,1] について
(xi,yj) (i,j=1,2) の全てがAの元になることがないならば、
Aのルベーグ測度が0となることを示す。

R^3の部分集合Bを B' = {(x1,x2,y)∈R^3 | (x1,y),(x2,y)∈A} と定める。
これはルベーグ測度 μ(A)^2 を持つ A^2⊂[0,1]^4 の射影であるから、
μ(A)^2≦μ(B') が成り立つ。
また、P={(x1x2,y)∈R^3 : x1≠x2} のルベーグ測度は0であるから、
B=B'-P と定めると μ(B)=μ(B') が成り立つ。

ここで、もし二点 (x1,x2,y),(x1,x2,y')∈R^3 が共にBの元であるとすると、
Bの定義より x1≠x2 となるが、これとAの仮定より y=y' でなければならない。
よって集合Bの、y軸(注意:yは第三成分)と平行な任意の直線との交点は高々一つ。
ゆえに μ(B)=0 であるから μ(A)=0.
0135132人目の素数さん
垢版 |
2022/01/01(土) 22:49:52.72ID:EdZz+tuI
>>129
できた

容易にA⊂I×Iとしてよいとわかる( ただしI = [0,1] )
各x∈Iに対してAx={y | (x,y)∈A}とおく
まずS = {x | μ(Ax)>0}は非可算である
そうでなければA' = A\∪[x∈S]AxとおくとSが可算ならA'も可測でμ(A')=μ(A)でなければならないが一方でFubiniの定理より
μ(A') = ∫[x]μ({y|(x,y)∈A'})dx = 0
となるので矛盾を生じる
この時Sの可算部分集合Cと正の定数e>0をμ(Ax)>eと選ぶことができる
何故ならはφ:S→(0,∞)をx→μ(Ax)で定める時いずれかの自然数nでφ^(-1)((1/2^n,1/2^(n-1)])が非可算集合とならねばならないからそこから可算無限集合Cを選びe=1/2^nと取ればよい
この時Cから異なる2点x,x'をμ(Ax∩Ax')>0となるように選べる
そうでなければCの元をx1,x2,...と並べてBxi = Axi \∪[j<i]Axjと定めればBxiは互いに素で全て測度がeより大きいからμ(∪Bxi) = ∞となるが、コレはBxiが全てIの部分集合であることに反する
以上によりCから2点x,x'をμ(Ax∩Ax')>0と選べるが、この時特にAx∩Ax'は異なる2点を含む□
0136132人目の素数さん
垢版 |
2022/01/01(土) 23:34:05.95ID:sn/pTPRs
>>135
S は実際に非可算だが、その証明が何かおかしい。

> A' = A\∪[x∈S]Ax

Ax ⊂ R なので ∪[x∈S]Ax⊂R だが、一方で A⊂R^2 なので、
A\∪[x∈S]Ax は次元の異なる集合の差を取っていて意味を成さないように見える。

それ以外の部分は論証込みで合ってると思う。
0137 【中吉】
垢版 |
2022/01/02(日) 03:23:16.66ID:BBcGQXha
>>110
自然数は奇数か偶然のどちらかです。
かつ2以外の偶数はすべて2で割れます。
∴ 素数で偶数なのは2だけだよ((-。-)
0138132人目の素数さん
垢版 |
2022/01/02(日) 12:37:20.02ID:bTMXpddk
>>136
書き方悪かったな
A'= A\∪[x∈S] {(x,y) | y∈Ax }

可算個しかないx∈Sに対して∪[x∈S] {(x,y) | y∈Ax }を抜く
まぁ抜かなくてもどのみち∪[x∈S] {(x,y) | y∈Ax }は測度0なんだけど
0139132人目の素数さん
垢版 |
2022/01/02(日) 14:31:28.64ID:5zFz/o7f
>>137
自然数が奇数か偶数かのどちらかであるって
そんなの未だに証明されていないよ。
0140 【上級国民】
垢版 |
2022/01/02(日) 16:13:54.90ID:BBcGQXha
>>137奇遇なのか奇数偶数交互にならんでんだ、しかも1差の等差数列。奇数偶数以外に自然数はないよ。
_____‖/ |;;;;;;;;;;;;;;;;;;;;;;;;;;;;;;;;;
 ̄ ̄ ̄ ̄ ̄‖  |;;;;;;;;;;;;;;;;;;;;;;;;;;;;;;;;;;
□ □ □  ‖,彡ミ、;;;;;;;;;;;;;;;;;;;;;;;;;;;;;;;;
_____‖川` , `; ;;;;;;;;;;;;;;;;;;;;;;;;;;;;;;;;;;;;プスッ!
_____‖/U⌒U、 ;;;;;;;;;;;;;;;;;;;;∩∩ ;;;;/;;;;;;;;
 ̄ ̄ ̄ ̄ ̄ ̄;_~U U~ ;;;;;;;;;;;;;;(_ _ )`⌒つ;;;;
;;;;;;;;;;;;;;;;;;;;;;;;;;;;;;;;;;;;;;;;;;;;;;;;;∪;;;;;;∪’ ;;;;;;;;;;;;
0141132人目の素数さん
垢版 |
2022/01/02(日) 16:20:31.47ID:yL5jPeCX
暇つぶしに
-1<re(α)<1とする
∫[0,∞]x^α/(1+x^2)dx
を求めよ
0142132人目の素数さん
垢版 |
2022/01/02(日) 17:13:18.06ID:k20LBJbn
>>138
了解。それなら大丈夫そう。まあ付け加えておくと、

S が高々可算なら、μ(Ax)=0 a.e.x∈[0,1] なので
μ(A)=∫[0,1] μ(Ax) dx = 0 となって矛盾。よって S は非可算。

…の方が直接的かなとは思う。
0144132人目の素数さん
垢版 |
2022/01/03(月) 10:55:38.76ID:CQE+dlLY
(tan1°)^20は有理数か.
0145132人目の素数さん
垢版 |
2022/01/03(月) 18:08:11.55ID:AJKeQhJJ
定理
自然数nに対して
[Q(cot(2π/n)):Q]
= φ(n) ( n : odd )
φ(n) ( n ≡ 2 ( mod 4 ) )
φ(n)/2 ( n ≡ 4 ( mod 8 ) )
φ(n)/4 ( n ≡ 0 ( mod 8 ) )
( ∵ )
まず最後のケースに全て還元できることを示す
最後のケースで正しいと仮定してn≡4 ( mod 8 )とする
仮定より
[Q(cot(2π/(2n))):Q] = φ(2n)/4 = φ(n)/2
である
cot(2π/n)∈Q(cot(2π/2n))は明らかである
仮定より2π(k-1)/n<π/4<2πk/nとなる自然数kがとれるがこの時
cot(2π/(2n))=cot(2πk/n-π/4)∈Q(cot(2π/n))
であるからQ(cot(2π/(2n)))=Q(cot(2π/n))となり主張は示された
最後の2つのケースで正しいと仮定しててn≡2 ( mod 4 )とする
仮定より
[Q(cot(2π/(2n))):Q] = φ(2n)/2 = φ(n)
である
仮定より2π(k-1)/n<π/2<2πk/nとなる自然数kがとれるがこの時
cot(2π/(2n))=cot(2πk/n-π/2)∈Q(cot(2π/n))
であるからQ(cot(2π/(2n)))=Q(cot(2π/n))となり主張は示された
最後の3つのケースで正しいと仮定しててn:oddとする
仮定より
[Q(cot(2π/(2n))):Q] = φ(2n) = φ(n)
である
仮定より2π(k-1)/n<π<2πk/nとなる自然数kがとれるがこの時
cot(2π/(2n))=cot(2πk/n-π)∈Q(cot(2π/n))
であるからQ(cot(2π/(2n)))=Q(cot(2π/n))となり主張は示された
以上により8|nの場合に示せば十分である
nを8の倍数としてζ=exp(2πi/n)とおく
K=Q(tan(2π/n)),L=Q(ζ)とする
[L:Q] = φ(n)でありζはKにi,cos(2π/n)を添加した体に含まれどちらもK上の最小多項式の次数は1か2であるから[L:K]≦4である
σ,τ∈Gal(L/Q)をσ(ζ)=σ(1/ζ),σ(ζ)=-1/ζで定めればσは複素共役をとる変換だからσはtan(2π/n)を動かさない
τ(i)=τ(ζ^(n/4))=(-1/ζ)^(n/4)=-i
τ(ζ-1/ζ) = ζ-1/ζ
τ(ζ+1/ζ) = -(ζ+1/ζ)
によりτ(tan(2π/n))=tan(2π/n)
であり少なくともGal(L/K)は位数4以上である
よって[L:K]≧4である
以上により
[K:Q] = [L:Q]/4 = φ(n)/4を得る□
0146絶対こいつと契約するな!!
垢版 |
2022/01/03(月) 18:18:33.18ID:mEAABgyf
絶対こいつと契約するな!!

超使えねー男!!
一番いらねー男!!

佐田恭一(さだきょういち)50歳
sada kyoichi
最高のAV男優

横浜出身
0147132人目の素数さん
垢版 |
2022/01/03(月) 19:31:57.29ID:NBovi1OW
>>143

>>139ってそんなに変か?
「拡張した整数論」とかで探せば
実際にどこかの誰かが考えそうだが。
0148132人目の素数さん
垢版 |
2022/01/03(月) 20:54:14.37ID:CQE+dlLY
>>145
おおすごい
一般化した主張まで載せてありがとうございます
お見事

簡易的な想定解として、(cos1°)^2 = 1/(1+tan1°)∈Q(tan1°)より、
Q(tan1°)⊃Q(cos2°)で、
x^2 -2(cos2°)x +1 = 0の根がζ_180であるから、
[Q(ζ_180):Q(cos2°)]=2より、
[Q(tan1°):Q]≧[Q(cos2°):Q]=φ(180)/2=24

したがって(tan1°)^20は無理数

というものでした
0149132人目の素数さん
垢版 |
2022/01/04(火) 15:47:27.96ID:SARDw29x
さらに一般化
>>145だと(tan1°)^30とかは有理数になる可能性を否定できてない

定理 3以上の自然数nにおいて、ある自然数mで(cot(2π/n))^mが有理数となるのはn = 3,4,6,8,12のときである

補題 0でない実数aにおいて、I = { m∈Z | a^m ∈Q }が0以外の整数を含むとする
この時mをIに属する正の数の最小値とすればx^m-a^mがaの最小多項式である

(∵) ζ=exp(2πi/m)とおく
aの最小多項式はx^m-a^mの因子であるからaの共役元はaζ^kと表示される
よってS⊂{0,1,..,m-1}を{ aζ^k | k∈S } がaの共役元全体となるようにとれる
この時N(a) = ±a^#Sが有理数となる(ただしN:Q(ζ,a)→Qはノルム写像)
特に#S∈Iであるからm≦#Sである
しかしSの取り方から明らかに#S≦mであるから#S = mでありaはちょうどm個の共役元を持つ□

(定理の証明) I = { m∈Z | cot(2π/n))^m∈Q }が自然数を含むとする
補題よりIの正の数の最小値mを取ればx^m - cot(2π/n)^mがcot(2π/n)の最小多項式となる
特にcot(2π/n)exp(2πi/m)はcot(2π/n)の共役元となる
しかし一方でQ(cot(2π/n))はQのアーベル拡大であり特にQのガロア拡大で共役を取る操作について閉じている
よってcot(2π/n)exp(2πi/m)はQ(cot(2π/n))の元でなければならず、よって特に実数でなければならない
よってm=1,2しか許されない
よって[Q(cot(2πi/n)):Q] = 1,2しか許されず前定理より
n:奇数、φ(n) = 1,2
n≡2 ( mod 4 ), φ(n) = 1,2
n≡4 ( mod 8 ), φ(n) = 2,4
n≡0 ( mod 8 ), φ(n) = 4,8
が必要である
コレを満たす3以上の自然数はn = 3,6,4,12,8,16,24しかなく、さらに
cot(2π/3)=-1/√3,
cot(2π/6)=1/√3,
cot(2π/4)=0,
cot(2π/12)=√3,
cot(2π/8)=1,
cot(2π/16)=1+√2,
cot(2π/24)=2+√3
であるから主張は成立する□
0150132人目の素数さん
垢版 |
2022/01/04(火) 21:04:21.90ID:D0t91gE+
>>149
さらなる追加の一般命題をありがとう
なるほど確かに冪が一般の場合どうするのか考えてなかったですが
ζ_m∈Q(cot(2π/n))→m=1,2のみ(それ以上だと虚数を含む)→拡大次数が1,2のみ
という流れはとても筋がいいですね
0152132人目の素数さん
垢版 |
2022/01/07(金) 16:27:20.63ID:FI3vu8kc
>>151
おぅ、お前も頑張れよ ( '‘ω‘)
0153132人目の素数さん
垢版 |
2022/01/08(土) 13:02:57.56ID:O7HhITXp
f:R→Rを無限回微分可能な関数とする
どんなa∈Rに対しても、fを何度か微分して、aを代入すると0になるとき、fは多項式であることを示してください
0154132人目の素数さん
垢版 |
2022/01/08(土) 14:10:25.03ID:s1MB/Msh
多項式でない関数は、無限べき級数に展開。
多項式の関数は、有限級数。
よって自明。
0155132人目の素数さん
垢版 |
2022/01/08(土) 15:55:49.27ID:O7HhITXp
>>154
C^∞だからといって、無限べき級数に展開できるとは限りません
有名な例でいうと

f(x) = e^(-1/x^2) (x > 0)
, 0 (x ≦ 0)

なんかがそうです
0156132人目の素数さん
垢版 |
2022/01/08(土) 15:59:09.48ID:O7HhITXp
>>154
さらに言えば
例え級数展開出来たとしても、
>>153の条件から、ただちには有限級数になることは示せないのではないでしょうか

「何度か」微分してaを代入すると0になる
この微分の回数はaに依存しています
0157132人目の素数さん
垢版 |
2022/01/08(土) 16:20:40.19ID:w4r5Z9TU
まあそのaに依存する微分回数に上界があれば自明だけどそうでない時にどうするか
まあそうはならないんだろうけど
0159132人目の素数さん
垢版 |
2022/01/08(土) 16:52:13.41ID:s1MB/Msh
いや、適当に書き込んだので、軽くスルーしてくれ。すまん。
0160132人目の素数さん
垢版 |
2022/01/08(土) 20:26:51.96ID:O7HhITXp
>>158
>>153が偽ではないので
有限級数に展開可⇔多項式

なのは確かなことです
0161132人目の素数さん
垢版 |
2022/01/08(土) 21:13:31.79ID:RmGcOhjc
>>153
ヒントおながいします
0162132人目の素数さん
垢版 |
2022/01/08(土) 21:29:09.14ID:O7HhITXp
>>161
>>83のようにベールのカテゴリー定理を使います!
0163132人目の素数さん
垢版 |
2022/01/08(土) 22:18:54.83ID:RmGcOhjc
>>163
とりあえず閉集合は
Fn = { x | f^k(x)=0 ∀k≧n }
とかですか?
しかしコレが開区間を持ってても何の足しにもならない気が
もっと別の設定ですか?
0164132人目の素数さん
垢版 |
2022/01/08(土) 22:20:36.63ID:RmGcOhjc
あ、f^k(x)=0じゃなくてf^(k)(x)=0
n階以降の微分が全部消えてる集合
この閉集合の合併でどれかが開区間を含んでもそこで終わり
0165132人目の素数さん
垢版 |
2022/01/08(土) 23:37:05.06ID:RmGcOhjc
わかった
Fn = { x | f^(k)(x) = 0 (∀k≧n) }
としてさらに
Un = int Fn ∪ (R \ Fn)
とおく
各Unは稠密開集合である
実際任意のx∈Rに対しdist(x,R\Fn)=0ならlim[i]ui=xとなるui∈R\Fnを取る事ができるしdist(x,R\Fn)>0ならx∈int Fnである
よってカテゴリー定理からD = ∩Unは稠密である
各x∈Dに対してx∈∩[n] int Fn ∪ (R \ Fn)であるが有限個を除いてxは (R \ Fn)の元に含まれる事はないから、ある番号でintFnの方に入る
そのような番号を選んでVx = intFnとおけば∪[x∈D]Vx = Rで各Vxでfは多項式だからR全体でも多項式□
0167132人目の素数さん
垢版 |
2022/01/09(日) 01:57:55.14ID:VG2Vm4C/
>>165
お見事!
大変美しい解答をありがとうございます

このように、ベールのカテゴリー定理は
「任意のx∈Rに対してあるn∈Nが存在して、なんかホニャララを満たす」
みたいな命題の「任意」と「存在」をひっくり返したいとき、度々役に立つ
というお話でした
0168132人目の素数さん
垢版 |
2022/01/09(日) 13:06:45.71ID:2tW7OSpV
んー?自分が理解できてないのかな…

>>153 って『各 x∈R に対してある自然数 n が存在し、f^(n)(x)=0 が成り立つ』って意味で合ってる?

もし『各 x∈R に対してある自然数 n が存在し、任意の整数 k≧n について f^(k)(x)=0 が成り立つ』
って意味で書いてたら、自分の読み違いってことで申し訳ないんだけど

前者の解釈で合ってるなら、>>165 の回答の Fn って
全て空集合になる可能性もあり得るのでは?

そうすると、どんなxでもある番号でintFnに入るというのは成り立たないと思うんだけど
0169132人目の素数さん
垢版 |
2022/01/09(日) 14:12:35.72ID:sOUZOFWN
>>168
確かにそう読めるね
でもそれなら
Fn = { x ; f^(n)(x) = 0 }
でやれば同じ
0171132人目の素数さん
垢版 |
2022/01/09(日) 15:49:18.12ID:yVccsDU6
あれ>>165穴あるかな
0172132人目の素数さん
垢版 |
2022/01/09(日) 15:57:08.46ID:VG2Vm4C/
>>168
すみません前者の
「各 x∈R に対してある自然数 n が存在し、f^(n)(x)=0 が成り立つ」
という意味で書いていました

なので>>169の通りF_nはこうするべきでしたね
0173132人目の素数さん
垢版 |
2022/01/09(日) 17:29:02.71ID:2tW7OSpV
>>169
あーそうかなるほど
そして >>165 のよくわかんなかった所もようやく自己解決できたっぽい
『ある番号でintFnに入る』を導くために ∪_n Fn = R という仮定を使ってるわけか
んで x∈intFn ということは関数 f^(n) が x の近傍でべったり0にくっついてることを意味するから
f は x の近傍で多項式。そしてこれが稠密な D 上で言えるから結果 f 全体も多項式、と…
よくできてるなあ
0174132人目の素数さん
垢版 |
2022/01/09(日) 20:02:48.29ID:JvJHC9jI
>>165は穴あるわ
Dが稠密で∪VxがDを被覆してても∪Vx=Rとは限らん
D={無理数}, Vx=( [x], [x]+1 )
のとき∪Vx=R\Zになってしまう
以下訂正版

S = { U : open ; f はUの各連結成分上で多項式 }
とおく
R = ∪Fnだからカテゴリー定理いずれかのFnは内点を持つがintFnはSに属するからSは空ではない
またSの元は合併で閉じているから最大元を持つ
それをUとする
G = R\Uとおく
G≠φとして矛盾を導く
再びカテゴリー定理よりG∩Fnは内点を持つ
よって開集合Vをとってφ≠G∩V⊂G∩Fnとできる
まずG∩Vは孤立点を持たない
何故ならもしc∈G∩Vが孤立点なら(a,b)∩G ∩V= {c}であるa,bがとれるが仮定によりfは(a,c), (c,b)において多項式だから十分大きなmで(a,c),∪(c,b)上f^(n)(x)=0でfはC^mだから(a,b)でf^(m)(x)=0
よってfは(a,b)で多項式となるがUの最大性からUは(a,b)を含まねばならずc∈R\Uに反してしまう
よってG∩Vは孤立点を含まない
次にa∈G∩Vのときm≧nなるmに対しf^(m)(a)=0である
コレはmについての帰納法で示される
m=nでは仮定により成立する
m<kで成立するとしてm=kとする
a∈G\Vをとればコレは孤立点ではないからai∈G∩V\{a}をlim[i]ai=aととれる
このとき
f^(m)(a)
=lim[i](f^(m-1)(ai)-f^(m-1)(a))/(ai-a)=0
となるゆえ主張は成立する
最後にVにおいてfは多項式である
c∈V,m≧nに対してf^(m)(x)がVで0になる事を示せば十分でc∈G∩Vの場合はすでに示された
c∈V\Gに対してc∈(a,b)⊂Vをfは(a,b)で多項式、a,bのいずれかはGの元ととれる
f^(deg(f))(x)は[a,b]で0でない定数だからf^(deg(f))(a),f^(deg(f))(b)は共に0ではない
よってdeg(f)<nとなりf^(m)(c)=0 (∀m≧n)である
よってV⊂UとなりG∩V=φであるがVの仮定に反する□
0175132人目の素数さん
垢版 |
2022/01/09(日) 21:07:13.48ID:2tW7OSpV
ま?もしかして
『あるR上稠密な開集合の各々の連結成分上で多項式であるC^∞級関数は多項式である』
ってギャップだったのか、もう何もわからん…
0176132人目の素数さん
垢版 |
2022/01/09(日) 21:31:51.65ID:JvJHC9jI
うん、Rの被覆R = ∪Uiがあって各Uiでfが多項式ならR全体で多項式になるのは簡単に示せる
しかし稠密ではあるがR全体ではない開集合UでいくらfがUの各連結成分上で多項式であったとしてもR全体で多項式だと主張するのは自明では許してもらえないと思う
とは言っても反例は思いつかないんだけど
今回の場合Uの隙間であるG=R\Uにおいても最低でも各点c∈Gにおいて少なくとも一個のf^(n)(c)が0になる事で穴が埋まってる
この仮定がない場合に反例が作れるのかどうか不明
つまり

UがRの稠密開集合でC^∞級関数fはUの各連結成分上では多項式とする
またR\Uは孤立点を持たない閉集合であるとする
このときfはR全体で多項式か?

は不明
反例も見つからない
証明もわからない
0178132人目の素数さん
垢版 |
2022/01/11(火) 08:30:03.10ID:FidiTyBG
>>177
論理学の本に載ってたなそれ
x=sqrt(2)^sqrt(2)としてxが無理数と仮定するとx^√2=2だから無理数の無理数乗だが有理数
xが有理数と仮定すると無理数の無理数乗だが有理数
xが無理数か有理数か分からなくても証明出来るのは面白いよね
0179132人目の素数さん
垢版 |
2022/01/11(火) 08:30:48.53ID:bAgG2dS1
F_nをフィボナッチ数列(ただし, F_1=F_2=1)とする。
このとき,
n≧2m+5を満たす任意の自然数m,nで
不等式
F_n ≧ Σ[k=0→m](n-k+1-2m)・C[m,k]
が成り立つことを示せ。
0180132人目の素数さん
垢版 |
2022/01/11(火) 08:35:27.76ID:bAgG2dS1
どっかで計算ミスしてたので無視してください
0181132人目の素数さん
垢版 |
2022/01/11(火) 08:56:25.94ID:bAgG2dS1
F_nをフィボナッチ数列(ただし, F_1=F_2=1)とする。
このとき,
n≧2m+5を満たす任意の自然数m,nで
不等式
F_n ≧ Σ[k=0→m](n-m-k)・C[m,k]
が成り立つことを示せ

が正しかったです…
0183132人目の素数さん
垢版 |
2022/01/11(火) 14:18:44.33ID:Idqx+l1J
有理数の有理数乗が有利数 ある 1^1=1
有理数の有理数乗が無理数 ある 2^(1/2)=√2
有理数の有理数乗が不定形 ある 0^0
有理数の無理数乗が有利数 ある 1^(√2)=1
有理数の無理数乗が無理数 ある 2^(√2)
無理数の有理数乗が有利数 ある (√2)^2=2
無理数の有理数乗が無理数 ある (√2)^1=(√2)
無理数の無理数乗が有利数 ある ((√2)^(√2))^(√2)=2
無理数の無理数乗が無理数 ある (√2)^(√2)
0188132人目の素数さん
垢版 |
2022/01/12(水) 01:44:15.73ID:nWcZAnE0
>>181
Gn = Σ[k=0→m](n-m-k)・C[m,k]
l = n - (2m+5)
とおく

Gn
= Σ[k=0→m](n-m-k)・C[m,k]
= 2^m×(n-m)-2^(m-1)m
= 2^(m-1)×(2n-3m)
= 2^(m-1)×(2l + m + 10)

F_(2l+2m+5)≧2^(m-1)×(2l + m + 10)
⇒F_(2l+2m+6)≧2^(m-1)×(2l + m + 10)
∴ F_(2l+2m+7)≧2^(m-1)×(2l + 2m + 20)
≧2^(m-1)×(2l+2 + m + 10)
∴l=0の場合示せばよい
∴F_(2m+5)≧2^(m-1)(m+10)を示せばよい
∴F_(k+5)≧2^(k/2-1)(k/2+10)を示せばよい
k=0→LHS = 5, RHS = 5
k=1→LHS = 8, RHS = 10.5/√2 = 7.424...
F_(k+5)≧2^(k/2-1)(k/2+10)
F_(k+6)≧2^(k/2-1/2)(k/2+10.5)
のとき
F_(k+7)≧2^(k/2)(k/4+5+k/√8+10.5/√2)
≧2^(k/2)( k/2 + 11 )
0189132人目の素数さん
垢版 |
2022/01/12(水) 02:01:24.02ID:nWcZAnE0
>>184
[[0,1],[-1,√n]]の特性方程式x^2-√nx+1=0を解いて
n=1→x=exp(2πi/3)
n=2→x=exp(2πi/4)
n=3→x=exp(2πi/6)
n=1→x=1
よってn=1,2,3なら単位行列
n=4はu = [[1],[1]]が固有ベクトルでv = [[0],[1]]がAv = v + u
A^12[u v ] = [ u v+12u] =[uv] [[1,12],[0,1]]
左から[uv]^(-1)かけたら答え
0191132人目の素数さん
垢版 |
2022/01/12(水) 23:21:12.46ID:nWcZAnE0
>>184
よくよく考えたらn=4のとき固有値2つとも1なんだから
[[0,1],[-1,2]]^k
= [[1,0],[0,1]] + k[[0-1,1-0],[-1-0,2-1]]
= [[-k+1,k],[-k,k+1]]
やん
0192132人目の素数さん
垢版 |
2022/01/13(木) 00:23:53.51ID:fIVWsdcb
>>191
ですです
行列計算にして上手く累乗計算するという問題でした
関数の見た目だと1,2,3,4で大差ないように見えるのが面白いなと思って出題しました
0193132人目の素数さん
垢版 |
2022/01/14(金) 08:13:23.77ID:gxPhFubJ
めっちゃ簡単な問題
5以上の素数pについて、p±1のどちらかは必ず6の倍数であることを証明しろ!
0194132人目の素数さん
垢版 |
2022/01/14(金) 08:18:36.44ID:mDEIYTrg
6n+2,6n+3,6n+4(n≧1)は2や3の倍数になるから
5以上の素数は必ずp=6n±1の形になる
0196132人目の素数さん
垢版 |
2022/01/15(土) 05:02:05.28ID:r9TB80Ws
どんな2n個の整数に対しても、その中のn個をうまく選べば和がnの倍数であることを示せ.
0197132人目の素数さん
垢版 |
2022/01/15(土) 10:16:14.74ID:4/6vqdAe
log[a] (log[b] (n)) = log [b^a] (n)
を証明せよ。
0198132人目の素数さん
垢版 |
2022/01/15(土) 10:16:47.82ID:4/6vqdAe
>>193
簡単すぎてハゲた。
0200132人目の素数さん
垢版 |
2022/01/16(日) 06:57:04.50ID:CzRUmJU9
日本人の血液型の割合はおよそA型40%,O型30%,B型20%,AB型10%とされる。
この割合で100人の献血ボランティアがいるとする。
すなわちA型40人,O型30人,B型20人,AB型10人である。
無作為に1人を選んで採血して4種類の血液がそろったら終了する。
同じ人を複数回採血することはない。
献血のお礼にジュースを献血者に1本与える。
99%の確率で4種類の血液をそろえるには何本のジュースが必要か?
0201132人目の素数さん
垢版 |
2022/01/16(日) 11:29:20.38ID:BSH76eZU
おそらくノーヒントでは無理なやつ
とりあえずノーヒント

0<a,b<1に対して

I(a,b) = ∫[0,π/2]1/√(a^2cos^2(θ) + b^2sin^2(θ))dθ

とおく

I((a+b)/2,√(ab)) = I(a,b)

を示せ
特に

π/2 = I(a,b) agm(a,b)

を示せ
0203132人目の素数さん
垢版 |
2022/01/16(日) 16:51:58.45ID:PrgNU+yd
2桁以上の好きな素数を思い浮かべてください。
その素数を6で割ってください。
6で割ったあまりに5を足してください。
5を足したあと、4で割ってください。
最後に、4で割ったあまりを強く強く思い浮かべてください...

その数字は、、2ですね?

がはは
0205132人目の素数さん
垢版 |
2022/01/17(月) 05:35:58.45ID:jD8LbTeh
>>202
スレチ連呼厨って答を出すことができないようだね。
数値解でもサクッと答えればいいのに。
>200みたいな問題は実用的で面白い。
0206132人目の素数さん
垢版 |
2022/01/17(月) 05:47:28.71ID:NYvJ+Lj4
P(n-1,4)=Σ[i=1,n-3]Σ[j=1,n-3]C[n-1,i]C[n-1-i,j]0.4^i×0.3^j×0.2^(n-1-i-j)
P(n-1)=Σ[i=1,4]P(n-1,i)
0208132人目の素数さん
垢版 |
2022/01/17(月) 06:42:56.01ID:jD8LbTeh
>>200
資金不足で臨床試験が中止に追いやられることもときにはある。
ジュースでなくて採血バッグの個数にした方がリアルだな。


日本人の血液型の割合はおよそA型40%,O型30%,B型20%,AB型10%とされる。
この割合で100人の献血ボランティアがいるとする。
すなわちA型40人,O型30人,B型20人,AB型10人である。
無作為に1人を選んで採血して4種類の血液がそろったら終了する。
同じ人を複数回採血することはない。
(1)99%の確率で4種類の血液をそろえるには何個の採血バッグが必要か?
(2)10個の採血バッグしかないときに4種類の血液が揃う確率はいくらか?
0211132人目の素数さん
垢版 |
2022/01/17(月) 10:27:11.78ID:byBENto8
>>210
ぐう自己満で草w
(画像の会話考えた香具師も、登場人物もw)
0212132人目の素数さん
垢版 |
2022/01/17(月) 12:25:49.32ID:NZm4I09Y
宮迫の焼肉屋、
あれって国と自治体の補助金目当てってマジ?

店が繁盛するかどうか、元をとるのに何年かかるかを
まるで考えていないように見えた。
0215132人目の素数さん
垢版 |
2022/01/17(月) 14:52:31.40ID:igsBLdxO
>>201 は多分ノーヒント苦しいのでヒント
まず x = tan(θ) で置換すると

I(a,b) = ∫[0,∞]1/√((x^2+a^2)(x^2+b^2))dx

よって

I((a+b)/2,√(ab))
= ∫[0,∞]1/√((x^2+(a+b)2/2)(x^2+ab))dx
= 1/2∫[-∞,∞]1/√((x^2+(a+b)2/2)(x^2+ab))dx

コレをもう一回置換積分
どう置換するかがミソですがここまで来ればこの手の積分の置換の定石
コレがピタッとI(a,b)になるのがとても心地よい
0216132人目の素数さん
垢版 |
2022/01/17(月) 16:02:55.36ID:igsBLdxO
訂正

I((a+b)/2,√(ab))
= ∫[0,∞]1/√((x^2+(a+b)^2/4)(x^2+ab))dx
= 1/2∫[-∞,∞]1/√((x^2+ab+(a-b)^2/4)(x^2+ab))dx

ですね
受験数学でもでる置換積分
受験の時は「t = ××とおいて...」とそれ答えやんってヒントつくかな?
x^2+abの方が√の外に出てきます
0217132人目の素数さん
垢版 |
2022/01/17(月) 22:14:11.31ID:jWsBHMC1
>>196
これ難しい
0218132人目の素数さん
垢版 |
2022/01/17(月) 23:21:05.13ID:ajsE0Y1f
エルデシュの定理だっけ
素数の場合から始めるはず
0220132人目の素数さん
垢版 |
2022/01/18(火) 01:17:16.99ID:AH7u6ZfN
>>201
>>215
受験定番の置換t=x+√(x^2+ab)ですね

I((a+b)/2,√(ab))
= 1/2∫[-∞,∞]1/√((x^2+ab+(a-b)^2/4)(x^2+ab))dx
ここで置換 x = (t-ab/t)/2, dx = (1+ab/t^2)/2 dt をする
= 1/2∫[0,∞]1/√(((t-ab/t)^2/4+ab+(a-b)^2/4)((t-ab/t)^2/4+ab)) (1+ab/t^2)/2 dt
= ∫[0,∞]1/√(((t+ab/t)^2+(a-b)^2)t^2) dt
= ∫[0,∞]1/√((t^2+a^2)(t^2+b^2)) dt
= I(a,b)

この関係式から楕円関数論を見通したガウスは凄い
0221132人目の素数さん
垢版 |
2022/01/18(火) 01:38:51.60ID:iQvcPKE+
>>220
素晴らしい
大正解
同じ置換2発で
J((a+b)/2,√ab) = 1/2( abI(a,b) + J(a,b))‥(❇︎)
が示せます
そこまでできたとして次は

問題

a[n],b[n]を
a[0]=a, b[0]=b
a[n+1]=(a[n]+b[n])/2,
b[n+1]=√(a[n]b[n])
で定めるとき

J( a,b )/ I( a,b )
= ( a^2 + b^2 )/2 + Σ[n:0〜∞]2^n((a[n] - b[n])/2)^2

である事を示してください
----
(❇︎)がキーですがコレだけでは無理
とりあえずノーヒントで
しかしおそらくノーヒントは苦しいでしょう
0222132人目の素数さん
垢版 |
2022/01/18(火) 09:05:11.83ID:3nZ26UdM
>>218-219
面白そうな問題だから自力で解くまでこのスレ見ないでおこうと思ったけど全く解けないからスレ来てみたら、そういうことか
まず2n-1でもいけることに気付けないとダメだった…
0223132人目の素数さん
垢版 |
2022/01/18(火) 09:55:50.29ID:qwcI+C/q
19の正の倍数において、各桁の和の最小値は何か?
0225132人目の素数さん
垢版 |
2022/01/18(火) 11:40:32.33ID:qwcI+C/q
>>224
不正解です

すみません
19だと
1000...01で作れるので簡単でした
0226132人目の素数さん
垢版 |
2022/01/18(火) 11:40:58.09ID:qwcI+C/q
なので改題

79の正の倍数において、各桁の和の最小値は何か?
0227132人目の素数さん
垢版 |
2022/01/18(火) 11:41:34.84ID:qwcI+C/q
証明も方針だけでいいのでお願いします
0228132人目の素数さん
垢版 |
2022/01/18(火) 11:51:55.45ID:RydfxIbm
それも1000...01で作れるのでは?
0229132人目の素数さん
垢版 |
2022/01/18(火) 11:58:04.94ID:qwcI+C/q
>>228
作れないはずですよ
0231132人目の素数さん
垢版 |
2022/01/18(火) 14:42:46.35ID:lbCzAwWm
素朴な疑問だけど
「各位の和はいくらか?」って
パズルの要素以外で役に立つ機会ってある?
何か自然科学などの計算に応用されたりしてるんかな?

「何桁ですか?」は使う場合があるのは理解できる。
10の何乗になるかが分かるから、
その数の大きさをおおざっぱに把握できるし。
0232132人目の素数さん
垢版 |
2022/01/18(火) 14:45:20.98ID:lbCzAwWm
例えば、 59^59 は何桁になりますか?
は使う場面も想像できるけど
これの各位の和はいくらになりますか?
は使う場面がまったく思いつかん。
というか各位の和って何か意味ある数じゃないよね。
0235132人目の素数さん
垢版 |
2022/01/18(火) 15:55:47.97ID:lbCzAwWm
>>234
そうじゃない。
ただ、各位の和 に
現実への応用が存在するんかなぁと思っただけ。
0236132人目の素数さん
垢版 |
2022/01/18(火) 16:27:52.77ID:qwcI+C/q
>>230
これで正解なんですが、証明もお願いします
0237132人目の素数さん
垢版 |
2022/01/18(火) 16:36:09.75ID:qwcI+C/q
>>236
すみません
想定解は発想もかなり必要ですが、かなりの力技でもあるので方針だけでおkです
0238132人目の素数さん
垢版 |
2022/01/18(火) 16:40:47.56ID:mzBz/OzM
自作の中学生用実力テスト作ったんだけど、解かせる人いなくて詰んだ

道ゆく中学生に全部正解したら1500円あげると言いながらビラみたいに撒いたら解いてくれるだろうか
不審者扱いされるだろうか
0239132人目の素数さん
垢版 |
2022/01/18(火) 17:05:55.06ID:0REspzHf
中学23年生のお兄さんでよければ解くよ?書いてごらん
0240132人目の素数さん
垢版 |
2022/01/18(火) 17:08:31.31ID:+HFfh/RS
3^n/2^n < k < (3^n-1)/(2^n-1) を満たす正整数n, k が存在しないことの証明か、反例をあげよ
0241132人目の素数さん
垢版 |
2022/01/18(火) 17:56:05.26ID:UcYWsuXo
>>237
ヨコだけど計算機マターやろ
1が不可能は自明
10^nを79で割ったあまりは
[1,8,10,18,21,22,38,46,52,62,64,65,67]
で78出てこないので2も無理
78-10^nを79で割ったあまりは
[11,13,14,16,26,32,40,56,57,60,68,70,77]
一致してるの出てこないので3も無理
0243132人目の素数さん
垢版 |
2022/01/18(火) 18:04:24.37ID:qwcI+C/q
>>241
解答ありがとうございます

すみません
10^nを79で割ったあまりは
[1,8,10,18,21,22,38,46,52,62,64,65,67]
↑このパターンだけというのはどのように導いたのでしょうか

あと3が無理の結論ですが、1と2を使った数字が出ないことも否定されるのでしょうか?
0244132人目の素数さん
垢版 |
2022/01/18(火) 18:15:25.68ID:qwcI+C/q
>>243
すみません前半自己解決しました
単純に余りがループするだけでしたね
0245132人目の素数さん
垢版 |
2022/01/18(火) 18:24:29.47ID:UcYWsuXo
>>243
10^m + 2×10^n ≡ 0 ( mod 79 )が解を持つなら
10^m + 10^n + 10^k ≡ 0 ( mod 79 )が解を持ち
10^m ≡ 78 - 10^n ( mod 79 )が解を持つから最後だけ否定しとけば十分でしょ
0246132人目の素数さん
垢版 |
2022/01/18(火) 18:26:50.78ID:qwcI+C/q
>>245
あーなるほど
失礼しました

ありがとうこういう解法もあるのか
お見事でした
0247132人目の素数さん
垢版 |
2022/01/18(火) 18:28:20.41ID:qwcI+C/q
用意していた想定解は次の通りです

まずmod 79で考えて、
0,1,2,...,78を頂点に持つ有向グラフを考える
頂点kに対して、頂点(k+1)にコスト1の有向辺を、
頂点10*kにコスト0の有向辺をつける

これをk=0,1,...,78の全てに行う

問題は頂点1から頂点0への最短経路問題になる

あとはダイクストラ法などを使ってこのグラフの最短経路問題を解けばいい

という感じでした
0248132人目の素数さん
垢版 |
2022/01/18(火) 22:49:55.26ID:koGbUQoF
test
0249132人目の素数さん
垢版 |
2022/01/18(火) 22:58:45.70ID:koGbUQoF
>>245
>10^m + 10^n + 10^k ≡ 0 ( mod 79 )が解を持ち
>10^m ≡ 78 - 10^n ( mod 79 )が解を持つから

ここがよくわからない。
10^(k+l) ≡ 1 ( mod 79 )
となるような適当な l をみつけて、
10^(m+l) ≡ 78 - 10^(n+l) ( mod 79 )
が解を持つからってこと?
0250132人目の素数さん
垢版 |
2022/01/18(火) 23:19:15.45ID:iQvcPKE+
>>249

> ここがよくわからない。
> 10^(k+l) ≡ 1 ( mod 79 )
> となるような適当な l をみつけて、
> 10^(m+l) ≡ 78 - 10^(n+l) ( mod 79 )
> が解を持つからってこと?

そう、l ≡ -k ( mod 78 ) を満たす正の整数でよい
0253132人目の素数さん
垢版 |
2022/01/19(水) 13:49:07.05ID:a7kTVwx1
7×143=1001
11×91=1001
13×77=1001
17×5882353=100000001
19×52631579=1000000001
23×4347826087=100000000001
29×3448275862069=100000000000001
0254132人目の素数さん
垢版 |
2022/01/19(水) 15:29:18.15ID:YHT5YPbg
>>226の出題者ですが「41」の方が良かったかもしれませんね

41の場合、実は5が最小値なのですが
その場合は100021や1002001、10003、などの可能性を全て否定しないといけないので

>>247のようにグラフ理論を用いた解法の方が幾分か楽かもしれません
0255132人目の素数さん
垢版 |
2022/01/19(水) 16:40:07.22ID:h0H/Iv3u
いわゆるゼロサム問題やな
上の方で出てたエルデシュの問題と同じ類
でもこっちは和がゼロになる最小の元数なので簡単
プログラム理論的にはdpの典型演習問題
10はZ/41Zの乗法群で位数5
A1=[1,10,18,16,37]
位数が奇数なのでこの中で和が0になるやつはない
2つの和で表される集合を探す
A2 = ns [ mod ( x+y ) 41 | x<- A1, y<-A1 ]
( ns は同じダブり消してるだけ)
= [2,6,11,12,14,17,19,20,26,28,32,33,34,36,38]
以下同文で繰り返す
let a3 = ns [ mod ( x + y ) 41 | x<-a2,y<-a1 ]
let a4 = ns [ mod ( x + y ) 41 | x<-a3,y<-a1 ]
let a5 = ns [ mod ( x + y ) 41 | x<-a4,y<-a1 ]
[2,6,11,12,14,17,19,20,26,28,32,33,34,36,38]
[1,2,3,4,5,6,7,8,9,10,11,12,13,14,15,16,17,18,19,20,21,22,23,24,25,26,27,28,29,30,31,32,33,34,35,36,37,38,39,40]
[0,1,2,3,4,5,6,7,8,9,10,11,12,13,14,15,16,17,18,19,20,21,22,23,24,25,26,27,28,29,30,31,32,33,34,35,36,37,38,39,40]
a5にゼロが出て終了
まぁ位数の最小素因子で絶対0が出るのでもちろん4まで出なかった時点で5確定やけどな
0256132人目の素数さん
垢版 |
2022/01/20(木) 02:05:04.04ID:zmXpeh64
f:(-1,1)→Rを
f(x) = x-x^2+x^4-x^8+...=Σ_{k=1}^∞ x^(2^k)
と定める.

極限lim(x→1-0)f(x)は存在するか?
0257132人目の素数さん
垢版 |
2022/01/20(木) 02:05:55.11ID:zmXpeh64
>>256
訂正します

f:(-1,1)→Rを
f(x) = x-x^2+x^4-x^8+...=Σ_{k=0}^∞ (-1)^k x^(2^k)
と定める.

極限lim(x→1-0)f(x)は存在するか?

です
0259132人目の素数さん
垢版 |
2022/01/20(木) 19:12:27.18ID:++pihKpq
>>257
g(t) = Σ[k=-∞,∞] e^(-πi(k+t)) x^(2^|k+t|)
と置くときgは周期1の関数で
g(t) = Σ[n=-∞,∞] a_n e^(2πint)
とフーリエ展開できる

係数は
a_n = ∫[0,1]Σ[k=-∞,∞] e^(-πi(k+t)) x^(2^|k+t|) e^(-2πint) dt
= ∫[-∞,∞] x^(2^|t|) e^(-πi(2n+1)t) dt
= ∫[1,∞] x^u u^(-πi(2n+1)/log2) du/(u log2) + complex conj.
= Γ(-πi(2n+1)/log2,-log x) (-log x)^(πi/log2) / log2 + complex conj.

ガンマ関数は虚軸方向に関して指数で急減少するので、g(0)の主要項はn=0で
a_0→2|Γ(-πi/log2)/log2| cos(πlog(-log x)/log2 + θ)
= 0.00274922168 * cos(πlog(-log x)/log2 + θ) as x→1-0

したがって振動する
0260132人目の素数さん
垢版 |
2022/01/20(木) 23:58:36.40ID:++pihKpq
>>259
修正:係数の式のミス
a_n = ...
=...
...
× Γ(-πi(2n+1)/log2,-log x) (-log x)^(πi/log2) / log2 + complex conj.
〇 Γ(-πi(2n+1)/log2,-log x) (-log x)^(πi(2n+1)/log2) / log2 + complex conj.


補足:θはarg(Γ(-πi/log2))=1.513321789...,
f(x)=x-x^2+x^4-...
≒ 1/2 + 0.00274922168 cos(πlog(-log x)/log2 - 1.513321789)
(x→1-0)
0261132人目の素数さん
垢版 |
2022/01/21(金) 22:25:05.92ID:5kxkT9aM
>>257
f(x)=x-f(x^2) --- (*)
x^4=1-ε,0<ε<1/2と置くとx-x^2>ε/4より
f((1-ε)^(1/4))=f(x)=x-x^2+f(x^4)>ε/4+f(1-ε)
これを繰り返して
f((1-ε)^(1/4^n))
>f(1-ε)+ε/4+ε/4^2+...+ε/4^(n+1)
=f(1-ε)+ε(1-1/4^n)/3

ε=0.1,n>2とすると
f(0.9^(1/4^n))
>f(0.9)+0.1*(1-1/4^3)/3
>0.9-0.9^2+0.9^4-0.9^8+0.9^16-0.9^32+0.9^64-0.9^128+0.1*21/64
>0.50058
したがって(*)と合わせて考えると
0.5から少なくとも0.00058の距離で振動し収束しない
0262132人目の素数さん
垢版 |
2022/01/22(土) 01:51:19.99ID:YwPImppC
フーリエ展開するって方法もあるんだな
0263132人目の素数さん
垢版 |
2022/01/22(土) 16:09:37.75ID:+B+HT00f
f((1-ε)^(1/4))=f(x)=x-x^2+f(x^4)>ε/4+f(1-ε)
これを繰り返して
f((1-ε)^(1/4^n))
>f(1-ε)+ε/4+ε/4^2+...+ε/4^(n+1)

コレ合ってる?
2回目は
f((1-ε)^(1/16))>(1-(1-ε)^(1/16))+f((1-ε)^(1/4))
ではないの?
ε変化していかない?
0264132人目の素数さん
垢版 |
2022/01/22(土) 16:23:33.58ID:+B+HT00f
最小の40項足してもf(0.9)が1/2を超えると思えないんだけど

https://ideone.com/dW9xfg
0265132人目の素数さん
垢版 |
2022/01/22(土) 16:27:30.47ID:+B+HT00f
おっと評価してるのはf(0.9^(1/16)) = f(0.993436601584)か
それでもダメっぽい

https://ideone.com/2qGFH4
0266132人目の素数さん
垢版 |
2022/01/22(土) 16:38:50.88ID:5Dsrd4wC
>>259,260
おー面白い解法ありがとうございます。
tを加えて周期関数にしてフーリエの流れは全く思いつきませんでした

>>261
おみごとです こちらが想定していた解法でした

以下用意していた解答

極限が存在していると仮定する
f(x) = x-f(x^2)
より存在するならば極限値は1/2
さらにx<1ならば、x^2<xより、
f(x) = x-x^2+f(x^4) > f(x^4)

よって任意のa<1に対して
f(a),f(a^(1/4)),f(a^(1/16)),...は単調増大
f(0.999) = 0.500125...
より極限値が1/2であることに矛盾
0267132人目の素数さん
垢版 |
2022/01/22(土) 17:37:50.31ID:QDOFk/aj
>>257 の別の面白い解答見つけたので貼ります(Hardy, Divergent Series, p77より)

g(t) = f(e^t) - Σ[n=0,∞] t^n/(n!(1+2^n))
は恒等式
g(t) + g(2t) = 0
を満たすのでg(-2^u)はuの周期関数で
x→1-0でf(x)が収束 ⇔ t→-0でg(t)が収束 ⇔ g(t)は定数関数

しかしg(t)はRe t<0で解析的で
x→-0でIm g(x+2πi/3)→∞
(∵Im ω^(2^k) = √3/2 (-1)^k, ω=e^(2πi/3))
であるのでg(t)は定数関数でない

したがってlim[x→1-0]f(x)は収束しない
0268132人目の素数さん
垢版 |
2022/01/22(土) 18:16:51.97ID:+B+HT00f
>>267
それいけてる?
f(e^t)
=e^t-e^(2t)+e^(4t)-e^(8t)+...
はre(t)<0のとき成立するけどre(t)=0のとき成立するかはアーベルの定理くらいしか思いつかないけどアーベルの定理なら展開
f(x)
=x-x^2+x^4-x^8+...ー@
本体がx=exp(2πi/3)まで連続に延長できないとダメなのでは?

f(e^t)-Σ[n=0,∞] t^n/(n!(1+2^n))
が定数で収束半径∞
→f(e^t)が収束半径∞

はいえてもそこから@にexp(2πi/3)を代入できる事は正当化できるの?
0269132人目の素数さん
垢版 |
2022/01/22(土) 19:30:40.48ID:QDOFk/aj
>本体がx=exp(2πi/3)まで連続に延長できないとダメなのでは?
x=r exp(2πi/3),0<r<1まで解析接続できる(境界は含まない)ので問題ない

fとgの級数の定義式から導かれることは
f(x)は|x|<1で解析的でg(t)はRe(t)<0で解析的

論法はg(t)が定数関数と仮定すると
一致の定理よりg(t)はRe(t)<0の全領域で定数になるはずだが
x→-0でg(x+2πi/3)は∞に発散し
(なぜならばr→1-0でf(r exp(2πi/3))→∞)
∞に発散する点の近傍では異なる値を取るので定数関数であることに矛盾
0270132人目の素数さん
垢版 |
2022/01/22(土) 19:36:52.67ID:+B+HT00f
あぁ今わかった
虚部の方が+i∞に行くのね
なるほろ
0272132人目の素数さん
垢版 |
2022/01/22(土) 21:10:59.34ID:4PzxvlaW
>>271
n = 2^1011= 2x2x2x....
0274132人目の素数さん
垢版 |
2022/01/22(土) 21:43:16.99ID:4PzxvlaW
それ、 総乗 が 2022 やんけ
0276132人目の素数さん
垢版 |
2022/01/23(日) 01:00:59.45ID:5wLF+suT
よくよく考えたら虚部の方が∞に行くってのも変な言い方だったな
実部も-∞に行くわな
h(t)=Σ[n=0,∞] t^n/(n!(1+2^n))は明らかに整関数
なのでg(t) = f(exp(t)) - h(t)が定数関数ならf(exp(t))も整関数で特にφ(r) = f(exp( r + 2πi/3 )( r∈(-1,0))は有界な関数にならなければいけないが
φ(r) = exp(2πi/3 )(e^r + e^(2r) + e^(4r) + e^(8r)+...)
はr→-0で発散してしまう
arg(x)=0での議論がarg(x)=2π/3での議論に結びつくのが面白い
そこでは振動の話ではなく収束性の話だから初等的に処理できる
計算機もいらんし
素晴らしい
0277132人目の素数さん
垢版 |
2022/01/23(日) 01:11:24.10ID:5wLF+suT
あ、いや嘘書いた
やっぱり実部の挙動はよくわからんな
re φ(r) = √3/2(e^r - e^(2r) + e^(4r) - ... )
im φ(r) = 1/2(e^r + e^(2r) + e^(4r) + ... )
で相変わらず実部は交代級数になってるんやな
しかし虚部は絶対級数でr→-0で発散してしまう
0278132人目の素数さん
垢版 |
2022/01/23(日) 01:22:33.03ID:5wLF+suT
そうか、そしてさらにわかった
そもそも
ψ(r) = f(e^(r+πi)) ( r∈(-1,1))
でいいんだ
ψ(r) = -e^r + e^(2r) + e^(4r) + e^(8r) +...
最小の一項以外全部符号揃う
つまり

「lim[x→1-0]f(x)は収束しない、なぜならlim[x→-1+0]f(x)が発散するから」

となるわけだ
面白い
0279132人目の素数さん
垢版 |
2022/01/23(日) 01:45:03.34ID:b4eNAgaa
>>278
それはおかしいんでないか?
f(e^z)=e^z - e^(2z) + e^(4z) - e^(8z) +...
に注意深くz=r+πiを代入すると
ψ(r) = -e^r - e^(2r) + e^(4r) - e^(8r) +...
だと思う

2πi/3でうまくいってたのは
2^k≡(-1)^k mod 3
が成り立つからim((-1)^k e^((2πi/3)2^k))>0が言える
0280132人目の素数さん
垢版 |
2022/01/23(日) 01:51:48.12ID:5wLF+suT
>>279
あ、ほんとだ
さすがにそこまで甘くはないのかw
0281132人目の素数さん
垢版 |
2022/01/23(日) 22:50:45.39ID:5wLF+suT
a>0とする
lim[x→1-0](x+x^a+x^(a^2)+x^(a^3)+...)=∞
を示せ
0283132人目の素数さん
垢版 |
2022/01/24(月) 06:43:49.95ID:kBoDFwVG
>>271
約数の総和が2022になる自然数は1346の1個しかないが、
約数の総和が24になる自然数は14,15,23の3個がある。
約数の総和が72になる自然数は30,46,51,55,71の5個がある

1から2022までの自然数で
約数の総和がその数になる自然数がもっとも多いのはいくつか?
0287132人目の素数さん
垢版 |
2022/01/24(月) 08:54:43.22ID:pR03RQnq
内訳は

1440 : 552 570 594 616 790 826 874 885 957 958 969 1015 1045 1077 1195 1253 1343 1349 1357 1363 1439

2016 : 660 672 852 858 910 940 992 1002 1012 1162 1222 1245 1353 1435 1495 1509 1547 1757 1837 1909 1927
0289132人目の素数さん
垢版 |
2022/01/24(月) 18:41:07.89ID:Q5uqN03p
>>282-285
解説ありがとうございます。
勉強になりました。
アホな書き込みをして失礼しました。

ち、ちなみに謙虚な神戸大卒TOEIC700です ブンケー \(^o^)/
0290132人目の素数さん
垢版 |
2022/01/25(火) 01:52:00.83ID:eaqdfLu0
辺の長さが1,2,3,4,…,95の適当な並び替えであり、内角が全て等しい95角形は存在するか?
0291イナ ◆/7jUdUKiSM
垢版 |
2022/01/25(火) 02:53:27.28ID:BqmNp3Mp
>>140
>>290
トリケラトプスもレッドキングも実際には見たことない。
四角形で四辺を1,2,3,4にしようとしても4が入らないし、1,2,4,3にしても3のところが√14
∴むりっぽい。
0292132人目の素数さん
垢版 |
2022/01/25(火) 07:35:22.26ID:9pk2RfXn
>>288
列挙した数字の約数の和が1440や2016になる

552の場合:1+2+3+4+6+8+12+23+24+46+69+92+138+184+276+552=1440
2016の場合:1+41+47+1927=2016
0293132人目の素数さん
垢版 |
2022/01/25(火) 07:36:39.74ID:AfBDil19
>>289
やはり、国立大学卒業の人は卒業大学を言えるんだなぁ
尿瓶おまる洗浄係はシリツ卒であることが判明している。
0294132人目の素数さん
垢版 |
2022/01/26(水) 02:57:11.90ID:m/6PyNwW
>>291
不正解
0295132人目の素数さん
垢版 |
2022/01/26(水) 12:45:49.08ID:dBV9OW32
>>290
素因子が2つあるから存在する
0296132人目の素数さん
垢版 |
2022/01/27(木) 00:43:29.59ID:A3dAS8AN
非可換有限群は非可換可解部分群を持つ事を示せ
0297132人目の素数さん
垢版 |
2022/01/27(木) 01:38:38.15ID:MEBtoWn1
>>295
正解です
複素平面を使い、5角形を2π/19ずつ回転させて等差数列の重みを乗せるという解法でした
0301132人目の素数さん
垢版 |
2022/01/28(金) 08:20:18.27ID:WUveO5fQ
等差数列でなくてもよい

>>290
1辺が1の正95角形を用意し、辺に順番に
0, 1, 2, ..., 94の番号をつける

番号を5で割った余りが同じ辺ごとに
0, 1, 2, ..., 4 (または 0, 19, 38, ..., 76)
から決まった値を辺の長さに足す

番号を19で割った余りが同じ辺ごとに
0, 5, 10, ..., 90 (または 0, 1, 2, ..., 18)
から決まった値を辺の長さに足す

できた多角形は、角の値を保ったまま
1辺が 1, 2, 3, ..., 95 の相異なる値の図形となる
0302132人目の素数さん
垢版 |
2022/01/28(金) 08:27:17.57ID:WUveO5fQ
5角形×19でなく、19角形×5で
0+1, 5+2, 10+3, 15+4, 20+5, 25+1, ..., 85+3, 90+4, 0+5, 5+1, 10+2, ...
とおけば、全体が
(螺旋に近い19本の辺)×5
となって、肉眼でも円でない図形になるはず
0304132人目の素数さん
垢版 |
2022/01/28(金) 12:17:51.45ID:wLOcFCqX
長さがそれぞれ1,2,3,…,9の辺をどのように組み合わせても、内角が全て等しい9角形は作れないことを示せ.
0305132人目の素数さん
垢版 |
2022/01/28(金) 13:00:53.43ID:svQgi52W
>>301
なるほどありがとうございます
複素平面を使わないもっと初等的な解法もあったんですね
0306イナ ◆/7jUdUKiSM
垢版 |
2022/01/28(金) 14:04:47.95ID:Bf3nMRl3
>>291
>>303
1〜3、5〜16、17、19ら辺の数字を15個のマス? かぎ? に入れろってこと?
せやて異なる数ならあと9個しかないぜ。
0309イナ ◆/7jUdUKiSM
垢版 |
2022/01/28(金) 19:08:05.68ID:Bf3nMRl3
>>306
>>303
——————
3|4|6|7
——————
9|2|0|9
——————
1|3|7|3
——————
7|1|9|7
——————
231がだめか。
0311132人目の素数さん
垢版 |
2022/01/29(土) 00:59:10.23ID:EMlBSkbP
nを自然数、uをnの素因子の数とする
ζ=exp(2πi/n)とおく
このとき全単射σ{1..n}→{1..n}で任意の0≦v<uを満たすvに対してΣ[k=1,n]k^uζ^(σ(k))=0となるものが存在する事を示せ
0312132人目の素数さん
垢版 |
2022/01/29(土) 01:26:54.68ID:EMlBSkbP
>>304
ζ=exp(2πi/9)とする
全ての係数が0〜8の並べ替えである8次多項式f(x)でζがその根とならない事を示せばよい
8次の項に0がくるように調整できるので7次多項式で係数が1〜8の並べ替えとしてよい
f(x) = ax^7+bx^6+cx^5+dx^4+ex^3+fx^2+gx+h
とおく
x^6+x^3+1で割ったあまりは
g(x)=cx^5+(d-a)x^4+(e-b)x^3+fx^2+(g-a)x+(h-b)
であるが条件よりコレが0になる事はない
0313132人目の素数さん
垢版 |
2022/01/29(土) 01:56:47.32ID:qwDuhe9o
>>312
お見事 正解です!
最高次に0を付けるのはうまいな

ζの最小多項式x^6+x^3+1にかけて8次の多項式にすると、
(x^6+x^3+1)(ax^2+bx+c)
=ax^8+bx^7+cx^6
+ax^5+bx^4+cx^3
+ax^2+bx+c
の形にしかならないので、内角の等しい9角形は
時計回りに3種類の長さの辺を3回繰り返す形しかありません
0314132人目の素数さん
垢版 |
2022/01/29(土) 01:59:46.03ID:qwDuhe9o
>>311
最後
Σ[k=1,n]k^v ζ^(σ(k))=0となるものが存在する事を示せ

の間違いですか?
0315132人目の素数さん
垢版 |
2022/01/29(土) 12:02:31.11ID:HTemwmFv
任意の正の実数α、正の実数βに対し、
必ず以下の不等式を満たす整数の組(m,n)が存在することを示せ。

| ((2^m)/(10^n)) - β | < α
何か、1024/1000が1に近いなと思ったから問題にしてみた。
0317132人目の素数さん
垢版 |
2022/01/29(土) 14:01:09.50ID:HTemwmFv
>>316
定数
0318132人目の素数さん
垢版 |
2022/01/29(土) 14:04:56.94ID:YpZKAQJ0
(X,d)は完備距離空間で、Xに孤立点は無いとする。
このとき、X の濃度は実無限以上であることを ZFC の中で示せ。
0319132人目の素数さん
垢版 |
2022/01/29(土) 14:06:15.65ID:YpZKAQJ0
>>318の背景:
(X,d)は完備距離空間で、Xに孤立点は無いとする。
もしXが高々可算無限なら、X = ∪_i {x_i} と1点集合の高々可算和に表せて、
各 {x_i} は (X,d) の閉集合なので、ベールのカテゴリ定理から、
ある {x_i} は内点を持つことになる。このとき、{x_i} は孤立点となって矛盾する。
よって、X は非可算無限であることが分かる。
従って、連続体仮説のもとでは、X は自動的に実無限以上となる。

実は、このことは連続体仮説を使わずに証明可能で、それが>>318の問題。
この背景だけ聞くと難しい問題に見えるかもしれないが、
実際にはわりと普通に示せる。
0320132人目の素数さん
垢版 |
2022/01/29(土) 18:19:04.84ID:IV150k2+
>>314
そうです
0321132人目の素数さん
垢版 |
2022/01/29(土) 18:53:05.71ID:IV150k2+
>>319
+-の有限列(空列も含む)の全体をΛとして順序をλ≦μ⇔μはλの後ろにいくつか+-を追加した形で入れる
Λでパラメトライズされた非可算閉集合の族Fλで
λ≦μ→Fλ⊃Fμ
Fλ∩Fμ≠φ→λ≦μ or μ≦λ
diam Fλ≦1/2^(lenλ)
となるものを以下のように構成する
まずFφは任意の点を選びその点中心の半径1/2の閉球とする
コレが可算無限なら>>319より矛盾するので非可算である
Fλまで構成されているとする
Fλから2点x,yを選びx,yを中心とする閉球を互いに素で半径が1/2^(lenλ+2)以下になるようにとりそれぞれFλ+, Fλ-にする
やはり非可算閉集合で直径に関する条件は満たしている
残りの条件を満たす事の確認も容易である
C仮定しているのでΛ全体まで定義できるとしてよい
Λの無限増加列は2^ωあるが増加列σ:λ1≦λ2≦...に対して閉集合の減少列Fλ1⊃Fλ2⊃...が定まり完備性とdiamFλの条件からx∈∩Fλiを満たすxがとれる
コレをx(σ)とするとき相異なる減少列σ、τからはFλの非交和性の仮定からx(σ),x(τ)は一致する事はない
よってXは2^ωより濃度が大きくなる
0322132人目の素数さん
垢版 |
2022/01/29(土) 19:19:49.05ID:IV150k2+
>>315
γ>0を
| log[10](x) - log[10]β | < γ→|x-β|<α
を満たすようにとれる(∵ y=10^xは連続)
Kroneckerの稠密定理から自然数m,nを
|m log[10]2 - n - log[10]β|<γ
を満たすようにとれる
このとき
|log[10]2^m/10^n - log[10]β| <γ
だから仮定より
|2^m/10^n-β|<α
である
0324132人目の素数さん
垢版 |
2022/01/29(土) 20:55:36.08ID:HTemwmFv
>>322
正解です。
0325132人目の素数さん
垢版 |
2022/01/30(日) 01:42:38.86ID:L3qd5Cw8
集合Sはn個の元を持つ有限集合とする。
S上の二項演算fであって、Sの任意の元a,bとS上の任意の全単射σに対して
f(σa,σb)=σf(a,b)
が成り立つようなものの個数を求めよ。
0327132人目の素数さん
垢版 |
2022/01/30(日) 10:18:08.12ID:AebVW8ek
>>326
プログラミング問題として面白いけど
4ない奴だと転置同一視して何通りあるの?
3×3の問題を含んでるけど5×5の問題の一部になる?
0329132人目の素数さん
垢版 |
2022/01/30(日) 13:01:49.32ID:AebVW8ek
>>325
n=1 #f=1
n≧2では任意の互換σ=(ab)で成立することが条件
a≠bで
f(aa)=a,bならf(bb)=b,a, f(ab)=a,bならf(ba)=b,a
n=2 #f=4
n=3でS={a,b,c}
f(aa)=cならf(bb)=c
σ=(ac),(bc)でf(cc)=a≠b=f(cc)で矛盾なのでf(xx)=x
f(ab)=cならf(ba)=c
σ=(ac)でf(cb)=a,f(bc)=a
σ=(bc)でf(ac)=b,f(ca)=b
f(ab)=a,bならf(ba)=b,a
σ=(ac)でf(cb)=c,b,f(bc)=b,c
σ=(bc)でf(ac)=a,c,f(ca)=c,a
#f=3
n≧4でS={a,b,c,d,…}
f(xx)=x
f(ab)=cならd=f(ab)で矛盾
x≠yでf(xy)=x,y,f(yx)=y,x
#f=2^{nC2}
0331132人目の素数さん
垢版 |
2022/01/30(日) 14:50:22.24ID:AebVW8ek
独立には選べないか
f(ab)=aならf(cd)=d,f(dc)=c,f(ab)=bで矛盾
#f=0
0332132人目の素数さん
垢版 |
2022/01/30(日) 14:54:02.89ID:AebVW8ek
f(ab)=a,bならf(xy)=x,yか
#f=2
0334132人目の素数さん
垢版 |
2022/01/30(日) 18:29:00.53ID:L3qd5Cw8
>>332
まあそういうこと 正解でいいかな
集合上の二項演算で全ての元に関して対称的になるものを考えた時、
nが大きいと前者か後者を選ぶしかなくなるけど、
小さければ少しだけ特別なものが存在できるというお話でした
0336132人目の素数さん
垢版 |
2022/01/30(日) 20:48:58.07ID:AebVW8ek
>>335
数学的には意味は無いけど
n×nに必ず解があると面白い
0337132人目の素数さん
垢版 |
2022/01/31(月) 01:00:45.26ID:NQ4HnBGX
>>299
をちょっと改造

https://sagecell.sagemath.org/?z=eJxtks1ugzAQhO-R8g57tMM2QNr0T-LaXHvoDXFwAyFI1mJhVzJv3zXQxklrIaz9PPYOg7WFAsoMId8hvPDEzz3CHoHr_AHhEYHrnBVPCFznrHiu1qv1qm5OoO1H_24FzxJe1yvgQXygbkgwmoE7N04x3G1Ml9LMhMdRMhMZZotMOI_uFprJXjUXp34ABR2x3-2W7vJq6RiGDl1tqaoLqn04Sye53Bx7KyYbGyUjwfgrsB39I6Bwgk9qH6GwZ0zq8YJcUDmfUCRzQebGhCLd9C38SqAU5JFGGZmdAikWPuMDyw-DMufuaMVPHtfxtAWnlhLnlpK8hNTFIUGUEpnJQtlVf5KL0dSZfZpej21Pu1pc1sIoWzRIpsJrPLSfx15z_0LkKLRMBfeXbPZGp7Q5qyLb7m-5b2zxprRtooXodwTzZOZ6aNzXQOLAy8sd5Pv2DQ5VnP8=&;lang=sage&interacts=eJyLjgUAARUAuQ==

重心を計算して辺の長さで色変えて表示できるようにしたのと、辺の長さのリストを手計算で計算して直接書き込めるようにした
なので自分が見つけた辺の長さのリストさえあればsagemathの文法知らなくてもすぐ使える
上の例は15角形の例
ls=[0, 5, 1, 3, 2, 4]
に書き換えれば六角形のが作れる

https://sagecell.sagemath.org/?z=eJxtkj9vgzAQxfdI-Q432sHlT9oslVjL2qEbYnADASTrsLAr2d--Z6DFSXsDyL979j0eVgZKqHMBFwGFgGcBZwEvzfFwPLTdDZT5mN4NozeH1-MBqJA2qA4ZoRXYobOS4PmkxwxXxpzwnBjLRb7JmHXCPkK9jG_WxW2aQcKI5CdN8alotomhVJhqatnsqHXhLJUU_HSdDFtsnCSPBP5XYEb8R4DhBJe0LkJhj09avyMbVNYlGMlskFmfYKRbvoUeCdQMnUDPI7NLIOXGV1yRvJqlHsarYT953MfTl5RahpRbhnwPaYxDgigl1IuFemz-JBejZTL51JPy_YTnlu29UHUvtEDdiHs895_XSdH8khWCKZ4xms_J7INOKj3IMk8vj9x1pnyTynRRI_odwTzqdT139mtGVlF7u4N0374B-YSXhg==&;lang=sage&interacts=eJyLjgUAARUAuQ==
0339132人目の素数さん
垢版 |
2022/02/02(水) 04:29:29.92ID:/urM6dEO
正整数全体の集合を N と置く。

(1) A⊂N は liminf[n→∞] |A∩[1,n]|/ n > 0 を満たすとする。
このとき、Σ[a∈A] 1/a = +∞ が成り立つことを示せ。

(2) A⊂N は limsup[n→∞] |A∩[1,n]|/ n > 0 を満たすとする。
このとき、Σ[a∈A] 1/a = +∞ が成り立つことを示せ。
0340132人目の素数さん
垢版 |
2022/02/02(水) 09:42:46.51ID:XeItgt7C
>>339
(1)は(2)から導かれるので(2)のみ示せば良い。

(2) 正の数 ρ を ρ = limsup[n→∞] |A∩[1,n]|/n と定める。

仮定より、正の整数列 {n_k}_(k=1,2,…) を、
任意の正の整数 k について次が成り立つように定めることができる。
・n_(k+1) ≧ 2n_k
・|A∩[1,n_k]|/n_k > ρ/2

この時、
Σ_(a∈A) 1/a
= Σ_(a∈A) ∫_[1,∞) |{a}∩[1,x]|/(x^2) dx
= ∫_[1,∞) |A∩[1,x]|/(x^2) dx
≧ Σ_(k=1,2,…) ∫_[n_k, 2n_k) |A∩[1,n_k]|/(x^2) dx
= Σ_(k=1,2,…) |A∩[1,n_k]|/(2n_k)
> Σ_(k=1,2,…) ρ/4 = ∞.
0342132人目の素数さん
垢版 |
2022/02/03(木) 15:54:04.79ID:+D0Tlu2q
平面上の格子点が以下の条件で有限色に塗り分けられている。
・格子点からなる任意の正方形は、少なくとも一つの頂点に色が塗られている
このとき、すべての頂点が同じ色で塗られている正方形が存在することを示せ。
0343132人目の素数さん
垢版 |
2022/02/03(木) 15:58:11.99ID:+D0Tlu2q
一辺1の正n角形型の道路の各頂点に紙が落ちていて、各紙には0以上n以下の整数が一つずつ書かれている。
ある頂点をスタートとして落ちている紙を拾い以下の操作を繰り返したとき、必ずちょうどスタートに戻ってこれることを示せ
・持っている紙に書かれた数字の距離だけ時計回りに道路を進み、持っている紙と落ちている紙を交換する
0344132人目の素数さん
垢版 |
2022/02/03(木) 16:02:06.49ID:h+W7BR/6
何その条件?
塗られてない可能性もあるの?
塗られてない4点の正方形は認められないの?
0345132人目の素数さん
垢版 |
2022/02/03(木) 16:13:09.49ID:+D0Tlu2q
>>344冷静に見返したらこの条件いらないですね。無視してもらって大丈夫です
0346132人目の素数さん
垢版 |
2022/02/03(木) 16:19:34.58ID:h+W7BR/6
>>343
置かれてる紙と持ってる紙の組みを状態と呼ぶとする
ルールに従って一回の操操作で状態Aから状態Bに移る事をA→Bと表すとする
ルールには“逆回し操作”が定義できる、すなわち
「落ちている紙と持ってる紙を交換し、反時計回りに交換した紙に書いてる数字だけ進む」、この操作でBからAに移動する事をB⇒Aと書くとする
初期状態A0から始めてA0→A1→‥としm0= min{ m | ∃n n>m, Am = An} とおけばm0=0である、でなければAm0=An, (n>m0)であるnをとるときAm0⇒A(m0-1), An⇒A(n-1), Am0=A(n-1)からA(m0-1)=A(n-1)とならねばならず、m0の最小性に反する
0347132人目の素数さん
垢版 |
2022/02/03(木) 16:20:21.37ID:h+W7BR/6
>>345
つまり全ての頂点は塗られてる、だけね
ラジャ
0348132人目の素数さん
垢版 |
2022/02/03(木) 16:37:36.05ID:+D0Tlu2q
>>346
すみません、後半部分の論証がまだ理解できていないのですが、「スタートからゴールまでを元の操作通り行った場合と、逆操作の通りに行った場合とで紙の配置が同じとは限らないこと(=操作が順序を逆にして一対一対応とは限らないこと)」は後半の論証に影響はないでしょうか?
例えばA0=[0,2,1,3]で2をスタートとして元の操作を行うと[0,2,3,1]の状態でゴールしますが、逆操作で行うと[0,3,1,2]の状態でゴールします。
0349132人目の素数さん
垢版 |
2022/02/03(木) 16:59:16.32ID:gLhKws+q
>>348
なんの事?
状態とは元のところに戻るではなく落ちてる紙、持ってる紙、経ってる場所、全ての状態なんだから逆回しできるでしょ?
0350132人目の素数さん
垢版 |
2022/02/03(木) 17:18:42.45ID:+D0Tlu2q
>>349 勘違いしてました。正解です
0352132人目の素数さん
垢版 |
2022/02/04(金) 01:38:52.54ID:nWaHt9CQ
年をとると我慢が効かなくなる
>>343ググって答え見てしまった
コレはおもろいけど絶対思いつかん
まぁもっと初等的な解放あるのかもしれんけど
0353132人目の素数さん
垢版 |
2022/02/04(金) 01:39:29.18ID:nWaHt9CQ
>>342だった
天才的や
0354イナ ◆/7jUdUKiSM
垢版 |
2022/02/04(金) 08:55:01.11ID:5m5l1icE
>>309
>>343
0からnまでの紙の枚数は(n+1)枚だから数字の和は、
n(n+1)/2
n個ある頂点のいずれにも0の紙が置かれなかった場合、
数字の和はn(n+1)/2で、
nかn+1のどちらかが2で割れるから自然数。
題意の操作は可能で、
nで割りきれるから(n+1)/2周した頂点まで進む。
nが奇数なら元の頂点に戻るが偶数なら最遠方の頂点にいる。
n個ある頂点のいずれかに0が置かれた場合、
n+1枚ある紙のうちの1枚が置かれない。
それが0でないなら、すなわちn/(n+1)の確率で、
置かれない数のぶんだけ元の頂点の位置より手前の頂点になる。
0355イナ ◆/7jUdUKiSM
垢版 |
2022/02/04(金) 10:50:17.79ID:5m5l1icE
>>354訂正。
置かれない紙の数字も右回りに進む数に入るからやっぱり元に戻る。
0356132人目の素数さん
垢版 |
2022/02/05(土) 01:29:40.20ID:566zsf+l
>>342
mが整数のとき
y=x+2mを色Aで
y=x+2m-1を色Bで塗ると2色で…?
0357132人目の素数さん
垢版 |
2022/02/05(土) 13:57:31.00ID:O3jLbt6F
>>356
一辺1じゃなくてもいいんやろ
0358132人目の素数さん
垢版 |
2022/02/05(土) 21:08:06.74ID:HjNUnJ5s
時計をイメージしてn=12のとき
1時の位置から出発するとする。
落ちている紙の番号の初期値が1時から12時まで
4 6 2 8 8 10 5 10 2 8 6 7
のときは
1 5 1
即ち1時、5時、1時で元の位置に帰る。拾った紙の枚数は2枚である。

落ちている紙の番号の初期値が
5 10 7 3 1 4 8 1 5 2 11 5 のときの移動は
1 6 10 12 5 6 11 10 2 12 2 6 7 3 10 9 2 4 7 8 9 8 9 10 5 10 11 4 6 10 3 11 12 10 2 7 10 8 7 12 1
拾った紙の枚数は40枚である。

紙の番号は無作為に選ばれているとして
スタートに戻るまでに拾う紙の枚数の期待値はどれくらいか?
0359132人目の素数さん
垢版 |
2022/02/06(日) 03:15:22.85ID:ap8ybDhe
平面上に円が与えられている

定規のみで、円の接線を引くにはどうすればよいか?
0360132人目の素数さん
垢版 |
2022/02/06(日) 04:02:22.86ID:CD9oU0ke
>>359
定規2本で L字 90度を作って
円にかぶせる。
0361132人目の素数さん
垢版 |
2022/02/06(日) 04:05:12.72ID:ap8ybDhe
>>360
すまん条件をちゃんと書きます

・定規は「2点を通る直線を引く」操作しかできない抽象的な道具とする

・線上にしか点は打てない

・線と線の交点は正確に打つことが出来る

・線上の点は目分量で好きに打つことが出来るが、正確な位置は不明とする
0362132人目の素数さん
垢版 |
2022/02/06(日) 04:05:48.42ID:ap8ybDhe
あと紙を折るとかのインチキも出来ません
0364132人目の素数さん
垢版 |
2022/02/06(日) 07:27:45.19ID:wrDpQZur
A=([0,1]×(0,1)\Q^2)⋃(([0,1]∩Q)×{0})

B=([0,1]×(0,1)\Q^2)⋃(([0,1]∩Q)×{0,1})

同相でないことを示せ。
0365132人目の素数さん
垢版 |
2022/02/06(日) 07:41:09.08ID:kNDQ5iKn
>>359
円と接点が与えられているとする
2点を結ぶ線分の垂直2等分線を描く手順があるので
円上にあと2点選んで2点ずつの垂直2等分線の交点として円の中心を求め
直線上の点における直交直線を描く手順があるので
中心と接点とを通る直線に接点で直交する直線を引けばいい
0367132人目の素数さん
垢版 |
2022/02/06(日) 08:23:36.85ID:ap8ybDhe
>>365
「定規のみ」でどうやって垂直二等分線を作図するの?
0368132人目の素数さん
垢版 |
2022/02/06(日) 08:25:53.79ID:dmawJuNU
>>355
直線しか引けないから垂直二等分線を作図する一般的な方法はないのでは?
0370132人目の素数さん
垢版 |
2022/02/06(日) 08:32:59.72ID:ap8ybDhe
>>369
それだとイージーすぎるので無しです
0371132人目の素数さん
垢版 |
2022/02/06(日) 08:57:19.42ID:kNDQ5iKn
>>370
じゃ多分無理
0373132人目の素数さん
垢版 |
2022/02/06(日) 09:19:04.70ID:kNDQ5iKn
>>364
離散位相で同相
0374132人目の素数さん
垢版 |
2022/02/06(日) 09:19:45.21ID:ap8ybDhe
>>371
可能です
0375132人目の素数さん
垢版 |
2022/02/06(日) 09:25:24.46ID:kNDQ5iKn
>>374
へぇ
接点は与えられてる?
0376132人目の素数さん
垢版 |
2022/02/06(日) 09:54:31.05ID:dmawJuNU
>>364

まずf:[0,1]→Aで(1/2,0)∈im(f)となるときim(f)⊂[0,1]×{0}である
そうでないとしてqを第2成分への射影としてm=sup(im(qf))>0となるが0<r<mとしてxi∈[0,1]をlim(qf(xi))=rとできる
必要なら部分列をとってlim xi = xが収束してるとしてよい
このときf(x)∈Aかつqf(x)は正の有理数であるがAの元で第2成分が正であるときそれは無理数でなければならないので矛盾である
特に[0,1]からAへの単射連続像写像は(1/2,0)への定値写像とホモトピックでホモとピー類はひとつしかない
一方でBへのそれは同様にして(1/2,0)への定値写像と(1/2,1)への定値写像へホモトピックなものがあるが、それらはホモトピックではありえない
何故ならもしそうならpt→[0,1]を前に合成してpt→(1/2,0)とpt→(1/2,1)がホモトピックになるが途中第二成分が(0,1)∩Qとなる点が通れないので不可能である□
0377132人目の素数さん
垢版 |
2022/02/06(日) 13:28:33.95ID:dmawJuNU
>>365
ヒントおながいします
0378132人目の素数さん
垢版 |
2022/02/06(日) 13:38:09.38ID:kNDQ5iKn
>>361
>・線上にしか点は打てない
与えられた円が唯一の曲線ね?
0379132人目の素数さん
垢版 |
2022/02/06(日) 13:41:47.35ID:ap8ybDhe
>>375
接点が与えられている場合でも解けますが
ひとまずどこでも好きなところに接線をひくで構いません
0380132人目の素数さん
垢版 |
2022/02/06(日) 13:42:27.67ID:ap8ybDhe
>>378
そうですね
初期段階では円周しか線がありません
0381132人目の素数さん
垢版 |
2022/02/06(日) 13:49:54.75ID:ap8ybDhe
>>377
>>359のヒントということですか?
0382132人目の素数さん
垢版 |
2022/02/06(日) 13:53:26.22ID:dmawJuNU
>>381
そうです
なんかないですか?
0384132人目の素数さん
垢版 |
2022/02/06(日) 14:02:35.91ID:kNDQ5iKn
>>361
線上にしか点は打てず
2点無いと直線も引けないから
最初にすることは円上に適当な2点取ってそれを通る直線を引くことで確定か
うーんむず
0385132人目の素数さん
垢版 |
2022/02/06(日) 14:03:03.14ID:ap8ybDhe
>>382
了解です
ヒントは>>363に書かれていますが、パスカルの定理を応用します
0386132人目の素数さん
垢版 |
2022/02/06(日) 14:05:44.55ID:dmawJuNU
>>385
イヤ、さすがにそれだけではわからない
もう少しおながいします
0387132人目の素数さん
垢版 |
2022/02/06(日) 14:15:26.12ID:ap8ybDhe
>>386
>>383
に書かれていますが、このように適当な(正五芒星ではない)星形を描くところまでは確定します
https://i.imgur.com/GxVugun.jpg

あとはこれらの交点からうまく工夫をします

うまいヒントが出せずすみません
0388132人目の素数さん
垢版 |
2022/02/06(日) 14:15:35.13ID:kNDQ5iKn
>>385
>パスカルの定理
6角形の隣り合う2頂点を1つにするとそれらを通る直線が接線になるってことか
0389132人目の素数さん
垢版 |
2022/02/06(日) 14:16:02.62ID:ap8ybDhe
>>388
おお! その通りです
0390132人目の素数さん
垢版 |
2022/02/06(日) 14:19:11.66ID:dmawJuNU
>>388
へ?
どゆこと?
頭悪くて申し訳ない
もう少し解説おながいします
0391132人目の素数さん
垢版 |
2022/02/06(日) 14:27:13.87ID:dmawJuNU
パスカルの定理はコレで間違いないですか?

定理
二次曲線上に頂点を持つ6点を順に結んだ直線をl1,l2,l3,l4,l5,l6としl1とl4の交点をP,l2とl5の交点をQ,l3とl6の交点をRとすればP,Q,Rは同一直線上に並ぶ

コレで6点のうち隣接する2点を同一点にとるとその2点を結ぶ直線が接線になるのはいいとして(eg. l1は接線として)そのl1は選んだ5点(仮にA,B,C,DE,F=Aで直線AF=l1が接線)からどうやって作図するんですか?
0392132人目の素数さん
垢版 |
2022/02/06(日) 14:52:59.16ID:ap8ybDhe
>>391
パスカルの定理はおっしゃる通りです

ABCDEFを二次曲線上の6点としたとき、
・ABとDEの交点
・BCとEFの交点
・CDとFAの交点

この3点は一直線上にあります

https://i.imgur.com/fJwdsm6.jpg
図が汚くてすみません

このようにABCDEFを配置したとき、
3点の位置関係を観察します
0393132人目の素数さん
垢版 |
2022/02/06(日) 14:53:38.15ID:kNDQ5iKn
>>390
その点の他に4点取って
その点を2点と勘定した6角形の対辺の交点3点が1直線に並ぶから
その点の対辺以外の4辺で2辺ずつ6角形としての対辺の交点を得てその2交点を通る直線と
その点の対辺の交点を得ると
そこからその点に引いた直線が接線
0394132人目の素数さん
垢版 |
2022/02/06(日) 14:55:47.20ID:kNDQ5iKn
言葉だと分かりにくいな
A点に接線を引くためにBCDE取って
直線ABと直線DEの交点をP
直線ADと直線BCの交点をQ
直線PQと直線CDの交点をRとすると
直線ARが求める接線
0395132人目の素数さん
垢版 |
2022/02/06(日) 14:56:25.09ID:ap8ybDhe
>>392
この状態でEをDに近づけることを考えます
0396132人目の素数さん
垢版 |
2022/02/06(日) 14:56:32.12ID:kNDQ5iKn
平行だと交点得られないから
適度に一般に点を取るのが肝要か
0397132人目の素数さん
垢版 |
2022/02/06(日) 14:59:00.46ID:kNDQ5iKn
パスカル16歳でこの定理証明したのはさすが天才
さらに一般に2次曲線なら成り立つんだってさ(ウィキペ調べ)
0398132人目の素数さん
垢版 |
2022/02/06(日) 15:00:55.58ID:dmawJuNU
>>393
それも考えだけどダメなんじゃないの?
l1は今作図されてないから不明
なのでPを作図することはできない
l2,l5の交点Q,l3,l6の交点Rは作図できてるからこのQR上のどこかにPがある
AFを置いといてB,C,D,Eを取り直してB'C'D'E'にすると新たにQ',R'は作図できるけどこの直線Q'R'とl1の交点は別の新たなP'であって元のPではない
なのでQRとQ'R'の交点はl1上にあるとは限らない
0399132人目の素数さん
垢版 |
2022/02/06(日) 15:03:41.25ID:ap8ybDhe
>>392
において、E→Dとすると、直線DEが作図出来ない
ということですかね?

パスカルの定理より
赤線の交点、緑線の交点、青線の交点が一直線上にある

というのがポイントです
0400132人目の素数さん
垢版 |
2022/02/06(日) 15:05:27.77ID:dmawJuNU
>>394
ダメだ、わからない
5点しかないですよね?
パスカルの定理は6点取るんですよね?
A,B,C,D,Eのどこが重複してとってる点なんですか?
0401132人目の素数さん
垢版 |
2022/02/06(日) 15:07:11.57ID:dmawJuNU
>>399
すいません、もう降参です、疲れ果てました
答え教えてください
5点A,B,C,D,Eとってパスカルの定理applyするときにどうapplyするんですか?
どの2点を2回使うんですか?
0402132人目の素数さん
垢版 |
2022/02/06(日) 15:12:53.87ID:ap8ybDhe
>>401
分かりました
解説をします
https://i.imgur.com/NNmxHmi.jpg

今回はこのように5点を配置して、
・DCとAFの交点
・DFとEFの交点

を結ぶ直線Lを描きます

さらに、直線ABとLとの交点をP
とすれば直線PDが接点Dでの接線になります
0403132人目の素数さん
垢版 |
2022/02/06(日) 15:14:06.91ID:ap8ybDhe
PDが接線になる理由は>>392の通りで、
EをDに近づければわかります
0404132人目の素数さん
垢版 |
2022/02/06(日) 15:15:26.09ID:ap8ybDhe
ごめんミスです

今回はこのように5点を配置して、
・DCとAFの交点
・DFとBCの交点

を結ぶ直線Lを描きます

さらに、直線ABとLとの交点をP
とすれば直線PDが接点Dでの接線になります

でした
0405132人目の素数さん
垢版 |
2022/02/06(日) 15:15:50.42ID:kNDQ5iKn
>>398
I1とか何それ?
0406132人目の素数さん
垢版 |
2022/02/06(日) 15:18:17.65ID:dmawJuNU
>>402
イヤ、すいません、証明の方が知りたいんです
本来パスカルの定理は6点取りますよね?
それをあえて2つを重複させるんですよね
6点の順番をA,B,C,D,E,Fとして重複させる一方をAとします
もう片方がF(又はB)の場合は既に書いた通りQ,Rは作図できますがPが作図できません
もう片方が他の3点の場合はパスカルの定理はトートロジーになってしまい接線も出てきません
その>>402で正しく接線になってる理由はパスカルの定理をどうapplyしたら出てくるんですか?
0407132人目の素数さん
垢版 |
2022/02/06(日) 15:22:05.00ID:dmawJuNU
>>405
記号の設定は>>391
もう少しちゃんと整理します

パスカルの定理はコレで間違いないですか?

定理
二次曲線上に頂点を持つ6点を順に選んでABCDEFとする
直線FAをl1,直線ABをl2,...直線EFをl6とする
l1とl4の交点をP,l2とl5の交点をQ,l3とl6の交点をRとすればP,Q,Rは同一直線上に並ぶ

これがパスカルの定理
基本「隣接する2点は相異なる」だけど同じに取った場合でもその場合は「その点における接線」に読み直しても主張自体は成立してる
0408132人目の素数さん
垢版 |
2022/02/06(日) 15:23:47.49ID:ap8ybDhe
>>406
>>392の図にある通り、DEが離れている場合はPの作図が可能ですよね?

E→Dに近づける、という極限操作を考えればPDが、D上の接線になるのは問題ないと思うのですが、
0409132人目の素数さん
垢版 |
2022/02/06(日) 15:26:35.88ID:ap8ybDhe
https://i.imgur.com/LGaiyja.jpg
↑において、青交点をP、赤交点をQ、緑交点をR
としたとき、パスカルの定理によりPQRは一直線上です

E→Dの極限操作においてもPQR一直線上が崩れないことを説明して
ということ?

たしかにそうすると連続性の議論をきちんとしないといけないかもしれない
0410132人目の素数さん
垢版 |
2022/02/06(日) 15:30:59.92ID:dmawJuNU
>>409
やっとわかった
DとEを重複させたんですね
イヤそれだけです
お騒がせして申し訳ない
0411132人目の素数さん
垢版 |
2022/02/06(日) 15:31:54.33ID:ap8ybDhe
>>410
あーなるほど
記号の誤解があったのかな?
それならいろいろややこしくてすみません
0412132人目の素数さん
垢版 |
2022/02/06(日) 15:33:46.33ID:dmawJuNU
>>411
よくありますね
自分はAとFを重複させたのに説明されてる方は最初の自分の設定でDとEを重複させて説明してるから話がいつまで経っても噛み合わない
疲れ果てました
0413132人目の素数さん
垢版 |
2022/02/06(日) 17:19:36.13ID:kNDQ5iKn
>>400
分かったみたいだけど>>394がすべて
0414132人目の素数さん
垢版 |
2022/02/06(日) 17:27:58.32ID:dmawJuNU
>>413
あなたの垂直二等分線論法はまだわかってないけど
どうやんの?
0415132人目の素数さん
垢版 |
2022/02/06(日) 17:55:40.80ID:kNDQ5iKn
>>414
だからできないって
0416132人目の素数さん
垢版 |
2022/02/06(日) 17:57:52.47ID:kNDQ5iKn
>>371に書いたようにコンパス無いとできないと思ってたよ
0417132人目の素数さん
垢版 |
2022/02/06(日) 18:00:21.05ID:kNDQ5iKn
定規で引けるのは直線だけだから
直角は作図できない
0418132人目の素数さん
垢版 |
2022/02/06(日) 19:37:51.33ID:GjDmKWda
結局、
「平面上に適当な5点をとると、直交する2直線を作図できる。」
ってこと?
0419132人目の素数さん
垢版 |
2022/02/06(日) 19:51:36.90ID:kNDQ5iKn
>>418
どして?
0420132人目の素数さん
垢版 |
2022/02/06(日) 21:45:00.63ID:eYmxroor
>>418
円が書いて有ればね
0421132人目の素数さん
垢版 |
2022/02/07(月) 00:32:17.75ID:qEBnaip6
円に重ねるように
上へはみ出す感じの五芒星を描けば
接線がとれるってことか。
0422132人目の素数さん
垢版 |
2022/02/07(月) 00:56:24.10ID:kLz4pdwa
三角形ABCと点Pがあるとき,
APとBCの交点L
BPとCAの交点M
CPとABの交点N
とするとデザルグの定理により
ABとLMの交点
BCとMNの交点
CAとNLの交点
の3点は共線になるけど、この共線になる直線と点Pは「L,M,Nが接点となっている二次曲線」の極線、極点になってるのが
パスカルの定理からわかるんやね。
0423132人目の素数さん
垢版 |
2022/02/07(月) 02:40:24.48ID:2lffdyTm
ウルトライージー版コラッツ問題

どんな自然数でも

・偶数なら2で割る
・奇数なら1を足す

この操作を繰り返せば必ず1になることを証明せよ
0425132人目の素数さん
垢版 |
2022/02/07(月) 08:06:24.45ID:pZ0whciw
1に戻らない最小反例がをnとする
nが奇数の場合n→n+1→(n+1)/2<nで矛盾
nが偶数の場合n→n/2<nで矛盾
0426132人目の素数さん
垢版 |
2022/02/07(月) 10:38:23.37ID:2lffdyTm
>>425
正解!
0427132人目の素数さん
垢版 |
2022/02/07(月) 13:42:23.67ID:w8fbXDdf
>>361
あとで思ったのだけど
平行なら交点無いよね
平行かどうかは交点があるかどうかで判断するとすれば
ほんとに平行な場合有限の立場からすれば
判定不能にならないかな
判定できてもいいけど
その場合取り直すとかなんかちょっとカツコワルイ
何が言いたいかというと
点を取る順序工夫すべきでないかなと
A点に接線引くとき
まずB点取って直線ABを引き
直線AB上の円の外部にP点を取り
円上にC点を直線PCが円ともう一点Dで交わるように取り(ここにも一般の位置に取れるという公理が必要かなと)
直線AC上の円の外部にQ点を直線QDが弧BD
と交わるように取り(ここにも必要)
交点をE点とし直線ACと直線BEの交点(取り方からこれは交わりそう)をR点とし
直線PRと直線DEの交点(こちらも交わりそう)をS点としたら
直線ASが求める接線
0428132人目の素数さん
垢版 |
2022/02/07(月) 13:49:42.42ID:w8fbXDdf
任意に点を取るときに
特別な位置関係にならないように取れる
ということはユークリッド幾何の公準公理にないと思うけど
暗黙のうちに仮定されてるのかね
0429132人目の素数さん
垢版 |
2022/02/07(月) 14:38:37.57ID:tQn76Jsl
厳密には、作図できないよね?
円周上の2点を無限に近づけることはできても、
重ねた瞬間に、連結させる直線が引けない。
0430132人目の素数さん
垢版 |
2022/02/07(月) 14:56:24.65ID:Yn7sO/jv
>>429
なんも解答理解してなくて草
少しでも知的なコメントしようと頑張ったのかな
恥ずかしい
0431132人目の素数さん
垢版 |
2022/02/07(月) 14:59:47.47ID:pZ6+xVdH
>>429
え?
できるのだけど、、、
曲線上の2点を通る直線は考えられるが
2点を1点に重ねたら
そこ通る直線は無限にあるから
接線は考えられないって
高校生あるあるだな
0435132人目の素数さん
垢版 |
2022/02/08(火) 00:58:32.51ID:cWgwZd5w
>>342
[0..t-1]を[t]と略記する
超立方体Q=[t]^nのt元集合Lが組み合わせ論的ライン、あるいは単にラインであるというのをLの元は全て同一直線上でありかつ、その直線の方向ベクトルで、成分が全て+1または0のいずれかであるものがとれるときとする
ラインの座標の総和が最小である点を始点、最大である点を終点と呼びそれぞれL-,L+で表す
+1になる成分に対応するi∈{1..n}をLのactive座標、0になる成分に対応するものをfix座標と呼ぶ
Qの各点がr色に塗り分けられているとする
このとき幅sの扇をs本のラインの集合{L1,...,Ls}で
・終点Li+は全て共通
・Liは全て単一色でないが、Li\Li+は単一色
・総配色数はちょうどs+1
を満たすものとする

主張 任意のr,t,sに対してN(r,t,s)を選んで任意のn≧N(r,t,s)とQ=[t]^nのr色による配色が常に単一色のラインを含むか幅sの扇を持つようにできる
あるいは同値な言い換えとして[t]^nが単一色ラインも幅sの扇も持たないような高々r色による配色を持つようなnは高々有限個しかない

(∵) (t,s)=(T,S)に限定した時の主張をΦ(T,S)として(t,s)の全体に辞書式順序を入れ、この順序に関する帰納法で示す
証明に入る前にΦ(T,T)が成立するとき任意のSについてΦ(T,S)が成立しn≧N(r,t,r)と取れば[t]^nのr色の配色は常に単一色ラインを持つ事に注意する
まずΦ(1,1)は明らか
(t,s)<(T,S)で主張が成立するとして(t,s)=(T,S)のときを考える
s=1なら帰納法の仮定よりΦ(t-1,t-1)が成立しているからn≧N(r,t-1,r)とすれば[t-1]^nを自然に[t]^nの部分集合として考える事により既に注意したように[t]^nのラインLで[t-1]^(n)に制限したとき単一色になるものがとれる
この時Lは[t]^nにおいても単一色であるかもしくは幅1の扇である
よってs=1の場合にΦ(t,s)が示された
0436132人目の素数さん
垢版 |
2022/02/08(火) 00:58:40.53ID:cWgwZd5w
s>1とする
m=N(r,t,s-1), n=N(r^(t^m),t-1,r^(t^m))としてN(r,t,s)=m+nとおけば良いことを示す
[t]^(m+n)を自然に[t]^m×[t]^nとみなしそのr色による配色xを固定する
[t]^nの各元bに対して[t]^m×{b}のxによる配色によってr^([t]^m)の元を決める
これを「[t]^nのr^(t^m)色での配色」とみなせば、nについての仮定によりt^nのラインLでL\L+が単一色となるようなものがとれる
このとき各b∈L\L+に対して[t]^m×{b}に与えられるxによる配色は[t]^mの配色として全て同一なのでこれをyとする
この[t]^mの配色yに関してmについての仮定により単一色ラインまたは幅s-1の扇をとれる
前者の場合は良いので後者について考える
(Li)を幅s-1の[t]^mの扇とする
Liのactive座標をIi⊂{1..m}、Lのactive座標をI⊂{1..n}とする
Liの終点はiによらずPとおける
Lの終点をQとおく
[t]^(m+n)のラインKi、Kを全て終点が(P,Q)でactive座標がIi∪I、Iであるものとする
Ki,Kの中に単一色のものがあるときは主張は示されているのでそうでないとする
仮定によりKi\Ki+は単一色である
K\K+の色はyにおけるPの色になるから特にK\K+は単一色でKi\Ki+のどの色とも異なる
よってKi,Kの全体は幅sの扇となる
以上によりΦ(t,s)が示された□
0437132人目の素数さん
垢版 |
2022/02/08(火) 00:59:03.85ID:cWgwZd5w
定理(Hales-Jewett) t,rを任意にとるとき自然数N(r,t)を選んで任意のn≧N(r,t)において[t]^nのr色による配色が単一色ラインを持つようにできる
(∵) N(r,t) = N(r,t,r)と取れば良い

定理 N×Nを任意にr色に塗るとき正方形の4頂点で同一色になるものが現れる

(∵) n=N(r, 4)として[4]^nを次のようにr色に塗る
・(a1,a2,...,an)∈[4]^nに対してaiの2進表示の1の位をbi、2の位をciとする
x=Σ2^ibi、y=Σ2^iciとおくとき(a1,..,an)の色を(x,y)に塗られている色とする
このときその配色によるラインLがとれる
Lのactive座標をI、fixed座標をJとしx0=Σ[i∈J]2^ibi、0=Σ[i∈J]2^ici、d=Σ[i∈I]2^iとすればライン上の4点に対応する格子点は)x0,y0),(x0,y0+d),(x0+d,y0),(x0+d,y0+d)の4点でありこの4点が同一色に塗られているので主張は示された□
0438132人目の素数さん
垢版 |
2022/02/11(金) 14:07:09.53ID:+cra6L1k
Nは2以上の偶数であるとする。
この時、以下の条件をすべて満たすa,b,cの組み合わせが何通りかを求めよ。
0≦a≦b≦c≦N
a + b < c
b + c < a
c + a < b
a + b + c ≦ N


例)N=8の時、以下の6パターンが考えられるため、6.
1, 1, 1
1, 2, 2
1, 3, 3
2, 2, 2
2, 2, 3
2, 3, 3

=====

これの答えは、[x]をxを超えない最大の整数とし、
[ (N^3 + 6*(N^2) + 72)/144 ]
となる。
0439132人目の素数さん
垢版 |
2022/02/11(金) 15:13:42.62ID:/MM/V0cX
>>438
「<」全部逆では?
0440132人目の素数さん
垢版 |
2022/02/11(金) 16:03:02.26ID:+cra6L1k
>>439
ありがとう。その通りだった。
修正します

Nは2以上の偶数であるとする。
この時、以下の条件をすべて満たすa,b,cの組み合わせが何通りかを求めよ。
0≦a≦b≦c≦N
a + b > c
b + c > a
c + a > b
a + b + c ≦ N

例)N=8の時、以下の6パターンが考えられるため、6.
1, 1, 1
1, 2, 2
1, 3, 3
2, 2, 2
2, 2, 3
2, 3, 3

=====

これの答えは、[x]をxを超えない最大の整数とし、
[ (N^3 + 6*(N^2) + 72)/144 ]
となる。
0441イナ ◆/7jUdUKiSM
垢版 |
2022/02/12(土) 04:39:00.47ID:9BdWGIJs
>>355
>>359
まず与えられた円を定規でできる限り隠し、
透明な定規で円が見えていてもかまわない、
定規をじわじわずらしていき、
ぎりぎり円が定規からはみ出すかはみ出さないか、
という状態で止める。
定規に沿って直線を引けばそれが円の接線である。
0443132人目の素数さん
垢版 |
2022/02/12(土) 12:49:37.97ID:Eq3f2Hdr
>>441
不正解以外の何者でもありません

ルール>>361を見てください

ここでいう定規は2点を通る直線を引く操作「しかできない」抽象的な道具としています
0444132人目の素数さん
垢版 |
2022/02/12(土) 12:51:39.07ID:Eq3f2Hdr
>>429
多分解答誤解されてますが、
極限操作をしているのは「証明」の際であって作図自体は極限操作は一切していません

解答は>>402の通りです
0445132人目の素数さん
垢版 |
2022/02/12(土) 14:38:11.31ID:k1NCoHz5
そのレスを見て ふと思ったのだが、証明の際に極限が必要ないような
上手い証明はないのかな。

パスカルの定理の証明を5点の場合でトレースすればいいだけかな。
0446132人目の素数さん
垢版 |
2022/02/12(土) 14:58:52.23ID:9x+5Rinu
それはこの手の初等幾何の問題考えるときいつでも出てくる問題やな
接弦定理は円に内接する四角形の内角に関する定理の極限とみなす事できるけど、日本の初等教育では当然そういう見方はしない、接弦定理は接弦で別に扱う
しかし正直どうなんと思う
そこにこだわることに意味あるのはわかるけどなんだかなぁ感しかない
0447イナ ◆/7jUdUKiSM
垢版 |
2022/02/12(土) 15:06:54.90ID:9BdWGIJs
>>441
>>443
ほな2点とりなぁや。
定規当てたまま2点とってそれ結んだらええやん。
なにがパスカルや、しょーもない。
0448132人目の素数さん
垢版 |
2022/02/12(土) 15:07:14.10ID:BTyM9/mb
>>402
よくわからん
5点は適当にとるん?
Dでの接線とABが平行であることがあり得るんだからPDがDでの接線にならない場合があるのは明らかじゃないの?
0449132人目の素数さん
垢版 |
2022/02/12(土) 15:17:15.29ID:Eq3f2Hdr
>>447
「定規を当てる」とかそういうことはできません
ましてや透明な定規とか言ってますが、完全に論外です
>>361にある通り、
2点を通る直線を引く「能力しかない」抽象的な道具です

実在の定規で考えないでください
0450132人目の素数さん
垢版 |
2022/02/12(土) 15:22:30.71ID:Eq3f2Hdr
>>448
たしかに赤線青線が平行になる場合がまずいですが、
平行になったら、DとBの間に新たにB’を適当にとればよいだけです
0451132人目の素数さん
垢版 |
2022/02/12(土) 16:41:03.60ID:sRTKfkwy
a[n] = 2+√(2+√(2+√(2+…√2))…) (√がn個)のとき、
lim(n→∞)4^n*(a[n]-4)
を求めよ.
0452イナ ◆/7jUdUKiSM
垢版 |
2022/02/12(土) 16:55:42.95ID:9BdWGIJs
>>447
>>449
透明で5mm幅の格子つき。重宝するよ。
感熱紙のカッティングには事欠かない。
最適なサイズに拡大できる。
0453132人目の素数さん
垢版 |
2022/02/12(土) 17:10:34.30ID:FQFEDxwe
>>451
計算機でやると収束する様に見えないけどほんとに収束する?

https://ideone.com/JNT7sW
0454132人目の素数さん
垢版 |
2022/02/12(土) 18:22:37.77ID:OCz1TQEJ
>>453
2+√(2+√(2+√(2+…√2))…)<4
なのでそもそもマイナスにならないとおかしいです
0455132人目の素数さん
垢版 |
2022/02/12(土) 18:23:43.80ID:FQFEDxwe
あ、失礼しました
-4だ
0456132人目の素数さん
垢版 |
2022/02/12(土) 18:33:11.98ID:FQFEDxwe
100項ほど計算すると

-9.8696044010893586188344909998761511353126779486683228541929970277693954766094909365755222028684623872

辺に落ち着く
なんだろコレ?
0457132人目の素数さん
垢版 |
2022/02/12(土) 18:34:04.23ID:WZzVPKws
地球生物は地球気温45度で死滅するらしいので
海水温が45度になるようなら地球全体が45度になる
100度になるのが1800万年くらいなら
45度までは900万年くらいだな
25度上昇には450万年くらいか
地球気温15度+25度=40度
海水温12度上昇には225万年
地球気温が27度
6度上昇には112.5万年地球気温21度
3度上昇には56.25万年地球気温18度
1.5度上昇には28.125万年地球気温16.5度
0.75度上昇には14.0625万年地球気温15.75度
0.375度上昇には7.03125万年地球気温15.375度
0.1875度上昇には3.515625万年地球気温15.1875度
0.09375度上昇には1.7578125万年
0.046875度上昇には0.87万年
0.023度上昇には0.435万年
0.0115度上昇には0.21万年
0.00575 度上昇には0.1万年
0.00287度上昇には0.05万年
0.00144度上昇には0.025万年
0.00072度上昇には0.0125万年
0.00036度上昇には0.00625万年後
63年後に0.00036度上昇して地球気温が15.00036度になるわけだが
温室効果ガスによって地球気温が2度から4度上昇するという
※問題 地球に存在する温室効果ガスの実際の温室効果は約何倍かを四捨五入して答えてほしい
(めんどくさいのでここまでやってきた海水温の上昇スピードが全て正しいとする)
問題2この海水温上昇と問題1で出した温室効果ガスが正しいことを前提にすると
あと何年で地球気温がデッドラインの45度になるか?答えはおおよそでかまわない
0459132人目の素数さん
垢版 |
2022/02/12(土) 19:16:28.64ID:FQFEDxwe
それや
なんや倍角の公式使うだけやん
0460132人目の素数さん
垢版 |
2022/02/12(土) 19:41:18.42ID:FQFEDxwe
a1 = √2
a(n+1) = √(2+an)
として求めるのはlim((an)^2-4)×4^n
an = 2cos(π/2^(n+1))
だから
((an)^2-4)×4^n
=-(sin(π/2^(n+2)×cos(π/2^(n+2)/(1/2^(n+2)))^2
→-π^2
0461132人目の素数さん
垢版 |
2022/02/12(土) 19:46:32.99ID:FQFEDxwe
あ、初項ずれてる?
ま、いいや、直すのもめんどい
0462132人目の素数さん
垢版 |
2022/02/12(土) 19:59:33.76ID:xmEjBbgk
ほう、Cayley–Bacharach theorem なんてものがあるのか
0463132人目の素数さん
垢版 |
2022/02/12(土) 20:17:53.16ID:OCz1TQEJ
>>460
お見事です
値も方針も大正解です
0464132人目の素数さん
垢版 |
2022/02/12(土) 20:21:03.27ID:OCz1TQEJ
>>459
すみません 自作問題なんですが
クオリティはまだまだでしたかね
作問能力を上げるように精進します
0465132人目の素数さん
垢版 |
2022/02/12(土) 20:47:52.16ID:WZzVPKws
>>457
1700万年で100度になると仮定してもう一度計算し直した結果
550年で31度平均気温は上昇するので46度になるのは550年後の約2550年
地球は年に0.06度平均気温は上昇している
1960年頃から原発が増産されてるからここから550年後なら2510年になる
0466132人目の素数さん
垢版 |
2022/02/12(土) 20:51:10.06ID:WZzVPKws
>>465
東京の最高気温の基準値を36とすると
1960年から60年後の2020年頃は
東京の最高気温が0.06×60=3.6度で
39.6度を記録することができるようになってる
0468132人目の素数さん
垢版 |
2022/02/12(土) 21:06:07.97ID:WZzVPKws
独り相撲で悪かった
数学者には16689000年で100度になると仮定して計算してほしいです
1960年から16689000年後で100度になる計算です
地球の温室効果ガスの能力は海水温上昇温度を「」倍させるとか
地球平均気温が45度になるのは1960年から「」年後とか
0470132人目の素数さん
垢版 |
2022/02/12(土) 21:54:48.57ID:WZzVPKws
他にもおもしろい問題あったな
出産可能年齢を40才までと定義する
2021年の人口で40才までの人口を男女合わせて4000万人と定義する
このうち男3割と女3割は精神疾患知的障害など結婚しない出産もしないとする
男は2000万人で女は2000万人である
※日本の実際の人口を計算しやすいようにしている
この4000万人の出生率を2021年に1とすると出生率はどう変わっていくか?
出産される人数はどうなるか?
日本の40才までの人口は2100年頃にはどうなるか?
このままだといつ日本人が絶滅するか?
0475132人目の素数さん
垢版 |
2022/02/12(土) 22:06:56.32ID:WZzVPKws
やはり問題を最高におもしろいと思って出したのが悪かったなごめんなさい
こんな問題つまらないよな計算する人には悪いよなごめんなさいとか
申し訳なさとかそういう気持ちが必要でしたつまらない問題を出してすみませんでした
0476132人目の素数さん
垢版 |
2022/02/12(土) 22:26:07.73ID:WZzVPKws
これがわかれば日本政府に産めよ増やせよやらないといけないとか
このままだと〇〇年には絶滅するから出産数を〇〇人は増やしてほしいとか
政府に提言したり5chに出た答えを書きこむ必要が出てくる
うまく少子化対策ができれば未来が変わります
0477132人目の素数さん
垢版 |
2022/02/13(日) 18:48:01.56ID:KZLIVByZ
三つの実関数 f,g,h:R→R の組であって、任意の実数x,yに対して
( f(x) - g(y) )( x - h(x-y) ) = 0
を満たすものを全て求めよ。
0478132人目の素数さん
垢版 |
2022/02/15(火) 16:58:33.45ID:zxR7Un2e
できた
E = { (x,y) | h(x-y) = x }とおく
h(0)=eとする
y=x上でEに属するのは(e,e)のみである
〜をR\{e}の上の2項関係で条件
(a,b)∈R×R\E→a〜b
を満たす最小の同値類とする
この同値関係によるR\{e}の類が高々ひとつである事を示す
a<bでaの属する類とbの属する類が相異なるとする
c<a<b<dを等間隔にとる
c-a= a-b、(a,b)∈Eによりa=h(a-b)だからc≠h(c-a)によりa〜cである
よってb〜cではない
よって(c,b)∈Eでなければならない
同様にして(a,d)∈Eでなければならない
よってc=h(c-b)、a=h(a-d)でなければならず、c-b=a-dによりa=cとなってcの取り方に反する
よってこの同値関係による類は高々ひとつしかない
特にa,b≠eのときa=x1,b=xn,である列x1,...,xnで(xi,x(i+1)) or (x(i+1),xi)∈R×R\Eとなるものが取れる
前者のとき(xi,xi),(xi,x(i+1)),(x(i+1),x(i+1))がEに入らないから
f(xi) = g(xi) = g(x(i+1))=f(xi(+1))
であり後者のとき(xi,xi),(x(i+1),xi),(x(i+1),x(i+1))がEに入らないから
f(xi) = g(xi) = f(x(i+1))=g(xi(+1))
となる
以上により任意のx∈R\{e}に対してf(x),g(x)は同一の定数値kを取らねばならない
f(e)≠kとする
この時a∈R\{e}に対して(e,a)がEに属さなければf(e)=g(a)=kとなって仮定に反するから全てのaにおいて(e,a)∈Eとなってe-h(e-a)=0となりh(x) = eが必要である
g(e)≠kとする
この時a∈R\{e}に対して(a,e)がEに属さなければg(e)=f(a)=kとなって仮定に反するから全てのaにおいて(a,r)∈Eとなってa-h(e-a)=0となりh(x) = x + aが必要である
逆に
h(t)が定数値eをとりf(x),g(y)がx≠eにおいて共通の値kを取る
h(t)が一次式t+eでありf(x),g(y)がy≠eにおいて共通の値kを取る
また任意のh(t)に対してf(x),g(y)が共通の値kをとる定数関数
のいずれかの時は条件が満たされるから条件を満たす(f(x),g(y),h(t))の組みは上記3条件のいずれかを満たす組みの全体である
0479132人目の素数さん
垢版 |
2022/02/15(火) 18:11:19.08ID:rFk/7eUm
>>478
正解!お見事
(同値類とか出てきて一瞬身構えたけど、内容よく見たらそれほど複雑じゃなくてほっとした←)

f,g,hについての条件式のこころは
『y=f(x) のグラフを x 軸方向にどのように平行移動しても、
高々一点の例外を除いて y=g(x) のグラフと完全に重なる』
と表現した方がわかりやすかったかな…と少しだけ後悔

元々は別の問題を考えてた途中で出てきた問題だったんだけど、
・基本的な関係式に有限個の例外を許したらどうなる?
・よく考えたら複数の関数が出てくる関数方程式ってあまりやったことないな
ってことでこっちの方をメインで取り上げてみてできた問題でした
0480132人目の素数さん
垢版 |
2022/02/17(木) 16:05:43.30ID:8eGwykQG
正整数全体の集合を N と置く。A⊂N は Σ[a∈A] 1/a = +∞ を満たすとする。
特に A は無限集合なので、Aの元を小さい方から順番に a_1<a_2<a_3<… と表しておく。
2以上の正整数 d を任意に取る。各 a_i を d進法で表示して

a_i=a_{i0}d^0+a_{i1}d^1+…+a_{ik_i}d^{k_i}, a_{ik_i}≠0

と表す。これらの表示の各桁を順番に並べたd進法での無限小数、すなわち

0.a_{10}a_{11}…a_{1k_1}a_{20}a_{21}…a_{2k_2}…a_{i0}a_{i1}…a_{ik_i}…

を考えると、これは無理数であることを示せ。
0481132人目の素数さん
垢版 |
2022/02/17(木) 16:26:01.57ID:LszWIW1J
>>480
0〜d-1からなる周期kの数列biを適当な数ずつだけ区切って単調増大列を作ることになるが同じ桁数のものは最大k個しか作る事ができない
よってできる実数はどんなに多くとも一桁がk個、二桁がk個、...
となる
よってまず全桁が1である一桁の数をk個、次は全桁が1の2桁の数をk個、‥を並べた数列を(ci)とするとci≦aiとなる
このときΣ1/ai≦Σ1/ciで後者は絶対収束するので仮定に反する
0482132人目の素数さん
垢版 |
2022/02/17(木) 16:33:49.05ID:L480KLS8
構成した実数rが有理数と仮定してrのd進法における循環小数の周期をNとおくと、
正の整数nに対し、Aの元のうちd進法でちょうどn桁の整数は高々N個となる。
(なぜなら、周期Nのうちどこが開始位置かが決まればn桁の整数は一意に定まるため)
よって
Σ_(a∈A) 1/a
= Σ_(n=1,2,…) Σ_(a∈A∩[d^(n-1),(d^n)-1)) 1/a
≦ Σ_(n=1,2,…) N/(d^(n-1)) < ∞
より仮定と矛盾。
0483132人目の素数さん
垢版 |
2022/02/17(木) 17:34:12.34ID:8eGwykQG
>>481-482
正解!厳密にいうと、有理数なら

0.(非循環節)(循環節)・・・

という構造をしているので、小さな桁の範囲では、同じ桁のものがたくさん生じている可能性はある。
しかし、十分大きな桁の範囲では、同じ桁のものは定数個しかないので、
どのみち Σ[a∈A] 1/a < +∞ となって矛盾、という感じ。
0484132人目の素数さん
垢版 |
2022/02/17(木) 17:36:17.62ID:8eGwykQG
ちなみに、Σ[p:素数] 1/p = +∞ という有名な結果により、
素数列に対して>>480が適用できて無理数になる。
この場合はコープランド・エルデシュ定数と呼ばれている。

wikiだと、コープランド・エルデシュ定数が無理数であることを、
算術級数定理やベルトランの仮説によって証明しているが、
それより遥かに弱い Σ[p:素数] 1/p = +∞ という性質だけでも示せる、ということ。
0488132人目の素数さん
垢版 |
2022/02/18(金) 18:51:41.32ID:PHYyI9Ox
多面体の各面の面積と同じ長さを持つ法ベクトルを全て足すと0ベクトルになることを示せ.
0489132人目の素数さん
垢版 |
2022/02/18(金) 19:22:08.92ID:aqiUJmqS
各面を三角分割しておき全ての面は三角形としてよい
和は多面体の三角分割に対して加法的だから四面体の場合に示せばよい
4頂点の位置ベクトルを0,p,q,rとして
±和
=p×q+q×r+r×p+(r-p)×(q-p)
=0
0490132人目の素数さん
垢版 |
2022/02/18(金) 19:39:11.47ID:PHYyI9Ox
>>489
おーお見事です 素晴らしい
ガウスの発散定理による証明を用意していましたが、
四面体に分割→外積を利用
は面白いですね
0491132人目の素数さん
垢版 |
2022/02/18(金) 22:25:33.51ID:emC+9wj+
(-10^(-2(π/log10)^2) + 1/2) (2π/log10)^(1/2) 10^(1/8)
を10進16桁で計算し、その数値になる理由を考察せよ
0494132人目の素数さん
垢版 |
2022/02/18(金) 23:20:04.55ID:aqiUJmqS
なら全然分からんwww
最初の方1.1010010001で以下計算誤差かと思いきやこの辺りから期待はずれ
こんなんどう答えたもんやら
0495132人目の素数さん
垢版 |
2022/02/18(金) 23:24:35.78ID:emC+9wj+
>>最初の方1.1010010001で以下計算誤差かと思いきや
それで合ってますよ
0497132人目の素数さん
垢版 |
2022/02/18(金) 23:39:56.09ID:aqiUJmqS
どゆこと?
1.101001000100001000001...
の近似値になってると言う事?
0498132人目の素数さん
垢版 |
2022/02/18(金) 23:46:32.94ID:aqiUJmqS
1+(1/10)^1+(1/10)^3+(1/10)^6+(1/10)^10+...
を近似したと
φ(x)=Σe^(x(x-1)/2)みたいなのは見たことあるけど
θ関数とか言うやつだっけ?
その話?
0499132人目の素数さん
垢版 |
2022/02/18(金) 23:50:23.74ID:emC+9wj+
そういうこと
ヒント
Σ[n=-∞,∞] 10^(-2(nπ/log10)^2) (-1)^n (2π/log10)^(1/2) 10^(1/8)
= Σ[n=-∞,∞] 10^(-n(n+1))
を示せ
0501132人目の素数さん
垢版 |
2022/02/19(土) 00:00:59.97ID:OG9v2E3/
なんやっけ?
モックテータ関数とか言うやつだっけ?
こんなん自力でできるわけないから資料ググるしかないわな
ラマヌジャンの本で見たっけかな?
0503132人目の素数さん
垢版 |
2022/02/20(日) 18:51:45.54ID:3JD83gSz
某パズル本より

a1,a2,...,a2022!は1〜2022の整数からなる長さ2022!の整数列である
このとき部分列で総和が2022!であるものが存在する事を示せ

知ってる人は(・∀・)ニヤニヤよろしく
0505132人目の素数さん
垢版 |
2022/02/20(日) 23:55:23.18ID:47t3M86f
a[0] = 1, a[n+1] = a[n] + √(a[n] + a[n]^2)
とするとき
lim(n→∞) a[n]/2^n
を求めよ
0506132人目の素数さん
垢版 |
2022/02/21(月) 01:38:54.00ID:8xVosLyU
b[n]=a[n]/2^nとおく

2b[n+1]
= b[n] + √(b[n]^2 + b[n]/2^n)
= b[n] + b[n]√(1 + 1/(2^nb[n]))
≦ b[n] + b[n]( 1 + 1/(2^(n+1)b[n]))
= 2b[n] + 1/2^(n+1)
∴ b[n] ≦ b[n+1] ≦ b[n] + 1/2^(n+2)
∴ b[n]は単調増大、上に有界
∴極限値をもつ
極限値をβとして2b[n+1] = b[n] + √(b[n]^2 + b[n]/2^n)の両辺の極限をとってβ=1
0508132人目の素数さん
垢版 |
2022/02/21(月) 02:18:50.77ID:DCqQT7om
あ、ほんとだ
0509132人目の素数さん
垢版 |
2022/02/21(月) 02:40:58.97ID:lKw+PzEA
実験してみたら

0.7213475204444817036799623405009456863233395084059221233827237940382725497794839353943653186145816503

↑これくらいであってる?
0510132人目の素数さん
垢版 |
2022/02/21(月) 03:00:56.97ID:lKw+PzEA
1/(2log(2))?
0512132人目の素数さん
垢版 |
2022/02/21(月) 18:38:03.67ID:XQeaLUq6
>>511
答え1/log(2)?
ヒントおながいします
0514132人目の素数さん
垢版 |
2022/02/21(月) 21:24:04.32ID:4B/+dcZ4
>>513
逆数b[n]の漸化式は

b[n+1] = b[n] / ( 1+ √(1+b[n])) )

もちろんそんなの検討済み
でもわからんorz
0516132人目の素数さん
垢版 |
2022/02/21(月) 21:34:44.44ID:4B/+dcZ4
b[ n + 1 ]= √( b[n] - 1 )

何にも思いつきませんけどorz
0517132人目の素数さん
垢版 |
2022/02/21(月) 21:37:12.27ID:4B/+dcZ4
おっと

b[n+1] = √( b[n] + 1 ) - 1

なんも出てこないorz
0519132人目の素数さん
垢版 |
2022/02/21(月) 21:41:25.59ID:4B/+dcZ4
出てきたwwww
0521132人目の素数さん
垢版 |
2022/02/22(火) 00:13:10.33ID:hFgdf2UY
>>520
問題作る方は簡単だろ
変形していくだけ
解く方は難しい
どういう変形か割り出さねばならん
0522132人目の素数さん
垢版 |
2022/02/22(火) 01:34:09.94ID:JFwMU2wr
でもコレよくネットで見かけるタイプなんだよな
できなかったのちょい悔しい
√(x^2+1)が消える時のx=1/2(a+1/a)の形に固執し過ぎた
反省
0523132人目の素数さん
垢版 |
2022/02/22(火) 01:43:16.02ID:JFwMU2wr
>>503
これ面白いよ
図書館で見つけた某パズル本に載ってた
ある雑誌に掲載された高校生向けの問題(の改題)なんだけど寄せられた解答のひとつが素晴らしくてちょっと感動した
その本なかなか面白い問題載ってたけどこれは名作のひとつだと思う
0524132人目の素数さん
垢版 |
2022/02/22(火) 17:58:03.28ID:M9/JjQnL
平面上に半径の異なる二つの円P,Qと、その中心P,Qが存在する。
二つの円は共有点を持たず、点Pは円Qの外部にある。
この時、以下の図形を定規のみを用いて作図せよ。
(1)点Pを通る円Qの接線(2本)
(2)二つの円の共通接線(4本)
0525132人目の素数さん
垢版 |
2022/02/22(火) 18:02:44.48ID:M9/JjQnL
>>359みたいな問題を見つけたので
中心が与えられてるから難易度は>>359よりも簡単かも
ルールは>>361に一応「ある一点を通る直線を引く」操作も追加で
0526132人目の素数さん
垢版 |
2022/02/22(火) 18:06:42.99ID:WotQfvgK
後半はQも円Pの外側やな
0527132人目の素数さん
垢版 |
2022/02/22(火) 18:10:27.64ID:M9/JjQnL
>>526
一応この条件二つでわかるからわざわざ言わなくてもいいかなと思った
書いといたほうがよかったかな?
0528132人目の素数さん
垢版 |
2022/02/22(火) 18:37:46.23ID:k60RCkuI
>>524
(1) 求める2接線の接点をT,Uとする
T,Uが求まればそこからの作図法は既出
まずPを通り円Pと2点A1,B1で交差する直線l1をとる
既にA1,B1での接線m1,n1を引く
その交点をR1とする
このときR1は直線TU上にある
同様の作業をもう一本別のl2を選んで行いR2を作る
直線R1R2と円Pの2交点がT,Uである

(2)P,Qの半径をp,qとする
p<qとしてよい
中心P,Qをp:qに内分する点と外分する点を作図すればよい
(1)に従いTを作図する
Pを通りPTに直交する直線を作図する(後述)
この直線と円Pの交点のうち直線PQに関してTと同じ側にある交点Rを作図する
PTとQSの交点をXとすれば△XSPと△XTQは相似比がp:sの相似な三角形になる
同じ作業をUに関して行なえば直線PQに関してXと対称である点Yが作図される
XYとPQの交点が共通内接線の交点である
ここから(1)の手順で接線を得る
外分点は直線PQと直線RTの交点がPQをp:qに外分する点である
0529132人目の素数さん
垢版 |
2022/02/22(火) 18:42:10.82ID:C4ilV0C0
あ、後半撤回
0530132人目の素数さん
垢版 |
2022/02/22(火) 18:44:48.33ID:C4ilV0C0
後半訂正
RはPQに関してTと反対側にとる
PQとRTの交点をXとすれば△XPRと△XQTが相似比p:qの三角形
0532132人目の素数さん
垢版 |
2022/02/22(火) 18:52:00.81ID:t4jp8zPW
あと後述忘れた
円OとOを通る直線lが与えられたときOで直交する直線が作図できる
まずAを適当にとってそこでの接線tを作図しtとlの交点をBとする
(1)に従いBを通るもう一方の接線uを作図し接点をCとする
Oに関してA,Cの対称点A',B'が作図できるので同じ手順に従いt,uの対称t',u'が作図される
コレでできた菱形の対角線の一方がm
0533132人目の素数さん
垢版 |
2022/02/22(火) 18:54:16.77ID:t4jp8zPW
>>531
Pを通るQの接線でしょ?
0534132人目の素数さん
垢版 |
2022/02/22(火) 18:58:50.79ID:M9/JjQnL
>>533
Pは円Pの中心なので、
点Pを通り円Pと交点を持つA1B1は直径です
なのでこの方法でm1,n1を引いても並行です
Qの接線という意味なら、Qの外部の点であるm1,n1を通って
Qの接線を引く方法が示せてないのでだめです
0535132人目の素数さん
垢版 |
2022/02/22(火) 19:00:20.12ID:PJGI1nNp
>>538
いやもちろんQでの接線なんだから交点A1,B1は直線と円Qの2交点だよ?
0536132人目の素数さん
垢版 |
2022/02/22(火) 19:04:43.25ID:M9/JjQnL
>>535
>>528で円Pとなってたので勘違いしました
円Qのことであれば、一応R1がTU上に来る証明もお願いします
0537132人目の素数さん
垢版 |
2022/02/22(火) 19:12:02.39ID:5n9Hyrbm
>>536
それはまぁ受験数学の超基本テーマやん

座標を円がx^2+y^2=r^2となるようにとる
R1(a,b)としてR1を通る2接線の2接点を通る直線の方程式はax+by=r^2
Pの座標を(c,d)としてac+bd=r^2
同じ議論逆向きにもっかいして直線TUはR1を通る

受験数学では証明しないで使ったらあかんやろけどこんなもん数学マニアには常識でしょ?
0538132人目の素数さん
垢版 |
2022/02/22(火) 19:24:05.90ID:M9/JjQnL
>>537
丁寧な証明の方がいいかと思ったので
書いてくださりありがとうございます
「一応」と前置きはしましたが、しつこいと感じたらすいません
0539132人目の素数さん
垢版 |
2022/02/22(火) 19:36:04.41ID:At/EqAov
いえいえ大丈夫ですよ
お気になさらず
0540132人目の素数さん
垢版 |
2022/02/22(火) 20:03:51.24ID:M9/JjQnL
ちなみに複数解あります
個人的に一番好きなのは垂心ができるやつです
0541132人目の素数さん
垢版 |
2022/02/22(火) 20:30:16.21ID:M9/JjQnL
ついでなのでこれの証明も募集します
平面上に円Oとその中心O、円の外部の点Aがある
AOと円Oの交点をB,Cと置く。円周上にB,Cでない点Dを取り、
ADと円OのDでない交点をE、BDとCEの交点をF、BEとCDの交点をGと置く
FGと円Oの交点をH,Iとした時、∠AHO=∠AIO=90°を示せ
(ABとかは全部直線ととらえてもらって大丈夫です)
0542132人目の素数さん
垢版 |
2022/02/23(水) 22:52:20.43ID:UXjwCKYI
xyz空間内の1辺1の立方体をxy平面に正射影するとき、その面積を最大にするには立方体をどのように配置すればよいか。
0543132人目の素数さん
垢版 |
2022/02/23(水) 23:02:00.03ID:ajLZzp2P
>>541
なんも思いつかないから大先生のお力を借りた

[ cos(a-b), cos(c-d) ]. [[cos(a+b),cos(c+d)],[sin(a+b),sin(c+d)]]^(-1). [[cos(a+d),sin(a+d)],[cos(b+c),sin(b+c)]]^(-1).[[cos(a-d)],[ cos(b-c) ]]
=1

https://www.wolframalpha.com/input?i=%5B+cos%28a-b%29%2C+cos%28c-d%29+%5D.+%5B%5Bcos%28a%2Bb%29%2Ccos%28c%2Bd%29%5D%2C%5Bsin%28a%2Bb%29%2Csin%28c%2Bd%29%5D%5D%5E%28-1%29.+%5B%5Bcos%28a%2Bd%29%2Csin%28a%2Bd%29%5D%2C%5Bcos%28b%2Bc%29%2Csin%28b%2Bc%29%5D%5D%5E%28-1%29.%5B%5Bcos%28a-d%29%5D%2C%5B+cos%28b-c%29+%5D%5D&;lang=ja
0546132人目の素数さん
垢版 |
2022/02/24(木) 10:35:58.61ID:TKwJA6s4
定理 中心O半径Rの円に内接する四角形ABCDにおいて直線ABと直線CDの交点をX、直線BCと直線DAの交点をYとするとき
OX^2+OY^2-XY^2=2R^2
である

(∵)方べきの定理を用いて主張は
XC×XD+YB×YC=XY^2
と同値であるからこれを示せばよい
一般性を失う事なくBは線分AX上、Dは線分AY上としてよい
辺XY上に点Pをとってv=∠CBX+∠CPXを考える
P=Xの時v>180°でありP=Yのときv<180°であるから辺XY上の点Eにおいてv=180°となる
よって特にBCEXはこの順に同一円周上に並ぶ
よって方べきの定理により
XE×XY=XC×XD‥@
である
一方で
∠YEC=∠CBX=∠ADC
によりDCEYもこの順に同一円周上に並ぶ
よって方べきの定理により
YE×YX=YB×YC‥A
である
@とAを足して主張を得る□

系 定理の設定の元にXを通る円の2接線の接点をS,TとするときY,S,Tは同一直線上に並ぶ
(∵) 定理よりOX→・OY→=R^2であるがこれが結論のための十分条件である事はすでに示されている
0547132人目の素数さん
垢版 |
2022/02/24(木) 10:37:44.91ID:/g1q8eKy
正四面体なら、直観で見当がつくな。
互いにねじれの位置関係にある二辺が共に水平になるような姿勢で射影した時だ。
立方体は解らない。
0548132人目の素数さん
垢版 |
2022/02/24(木) 10:42:57.51ID:TKwJA6s4
立方体を|x|,|y|,|z|≦1/2としてよい
射影する平面の法線ベクトルをn=(a,b,c) (a,b,c≦0)、平面をn・p=0としてよい
このとき射影像の面積はa+b+cである
コーシーシュバルツより
a+b+c≦√3
でa=b=c=1/√3のとき等号成立するから求める最大値は√3で射影する平面と対角線が直交するときである
0549132人目の素数さん
垢版 |
2022/02/24(木) 13:37:33.70ID:FUnRKqSJ
正四面体の時は最小値が難しい
東大の過去問では最大最小両方求めないといけなかった
前期の入試問題で過去最難関クラスと言われている
0551132人目の素数さん
垢版 |
2022/02/24(木) 15:05:29.52ID:vYppSX27
>>548
pって何?ひとつの平面に対してnの取り方なんていくらでもあるんだから、射影の面積nに対して一定の値(a+b+c)になるってのが意味がわからない
0552132人目の素数さん
垢版 |
2022/02/24(木) 15:15:53.43ID:0cCmjhOy
>>551
原点を始点として、球面a^2+b^2+c^2=1の下半分を終点とするようなベクトルを取ってるんでしょ。この取り方だと各平面に対して(a, b, c)はただ一つに定まる。
シュワルツの不等式は(a, b, c)と(1, 1, 1)に対してやってる。
でも像の面積がa+b+cってのは俺もわからん。
0553132人目の素数さん
垢版 |
2022/02/24(木) 15:36:21.54ID:cxTw6HR9
面積がSである平面図形を単位法線ベクトル同士の内積がmの別の平面に射影したときの面積は|m|S
0554132人目の素数さん
垢版 |
2022/02/24(木) 15:37:04.89ID:cxTw6HR9
aあとa,b,c≦0はa,b,c≧0のうち間違い
0555132人目の素数さん
垢版 |
2022/02/24(木) 15:56:16.16ID:0cCmjhOy
>>553
立方体の射影がかならず正方形の射影になってるとは限らなくない?
たとえば法線ベクトルの取り方によっては正方形の射影と正方形の射影が重なることだってあるだろうし。
0556132人目の素数さん
垢版 |
2022/02/24(木) 16:03:19.22ID:NOhl3oQ7
>>555
立方体の射影は光が当たっている正方形3つの射影
つまり必ず3つの菱形の合併
0558132人目の素数さん
垢版 |
2022/02/24(木) 17:01:08.05ID:/g1q8eKy
立方体の六面の単位法線ベクトルは三種類。それらを、
(a,b,c)
(p,q,r)
(s,t,u)
として、
S=a+b+c+p+q+r+s+t+u
ただし、
a・p+b・q+c・r=p・s+q・t+r・u=s・a+t・b+u・c=0

何も考えずに式立てたら、こんな感じだが・・・。
ここからどうやって、a、b、c、の三変数のみの議論に持ってくの?
0559132人目の素数さん
垢版 |
2022/02/24(木) 17:38:50.97ID:0HNskc3D
>>557
なんで直感なんだよ
こんなんすぐ証明できるやろ?光線は直線上を伸びてくるんだからその光線が射影像に作る影は一点のみ
それは最初に当たったただ一点のみ
そして光線の方向ベクトルと面の光線ベクトルの内積が負である面うえの点が最初に光線に当たる事など不可能やろ?
そんな事東大の受験の証明で要求されるわけないやろ
アホか
0560132人目の素数さん
垢版 |
2022/02/24(木) 18:18:04.01ID:XrbJtKoC
ユークリッド空間の三つの単位ベクトル a,b,c∈R^3 s.t. a・b=b・c=c・a=0 が、
ある特定の単位ベクトル e∈R^3 に対して
S = |a・e| + |b・e| + |c・e|
がとり得る値の最大値を求める問題だから、言い換えれば
a=(1,0,0), b=(0,1,0), c=(0,0,1), |e|=1 とした時のSの最大値を求めれば良い。

e=(x,y,z) とおけば条件式は x^2+y^2+z^2=1 となり、S は |x|+|y|+|z| となる。
コーシーシュワルツより
S^2 ≦ (1^2+1^2+1^2)(|x|^2+|y|^2+|z|^2) = 3
であるからごにょごにょ
0561132人目の素数さん
垢版 |
2022/02/24(木) 18:53:28.61ID:uchVKiVD
>>542
立方体の1つの面を、xy平面、yz平面、zx平面に射影した時の面積(それぞれ、S1,S2,S3)の
合計は、立方体全体をxy平面に射影した時の面積に等しい

四平方の定理より、S1^2+S2^2+S3^2=1^2
この条件下での、S1+S2+S3の最大値は、√3 (S1=S2=S3=1/√3 の時)
0562132人目の素数さん
垢版 |
2022/02/24(木) 20:40:48.44ID:nqQZWLxh
正整数全体の集合を N と置く。

A⊂N は Σ[a∈A] 1/a = +∞ を満たすとする。
このとき、A'⊂A で
Σ[a∈A'] 1/a = +∞ かつ lim[n→∞] |A'∩[1,n]|/ n = 0
を満たすものが存在することを示せ。
0563132人目の素数さん
垢版 |
2022/02/24(木) 22:33:35.22ID:aiuelsHy
>>562
Aの有限部分集合の列Aiを以下のように定める
(1)A1はΣ[a∈A]1/a>1を満たすように任意にとる
(2)A(n-1)まで構成されたとする
まずA\(A1∪‥∪A(n-1))の元を小さいものから順に並べてb1,b2,b3...とする
このとき容易に全てのiについてΣ[j≡i(mod n)]1/bj=∞となる
そこで[ j | j≡1 (mod n)]の有限部分集合FをΣ[j∈S]1/bj > 1となるように選びAn={ bj | j∈S }とする
できた(An)に対してA'=∪Anとおく
コレが求める条件を満たす
実際Σ[a∈A']1/aは明らかである
また任意にm>0を選ぶときあるNをA\[1,N]がA1〜Amの元を含まないようにできる
この先の項はm個に一個の割合でしかとっていないので[N+1,N+T]の中にA'の元は高々Tm+1個しか現れない
よってlimsup A'∩[1,n]/n≦1/mである
m>0は任意であった
0564132人目の素数さん
垢版 |
2022/02/24(木) 23:42:24.59ID:XrbJtKoC
>>562
かなりごたごたしてしまったけど一応回答

整数 n≧0 と集合 S⊂N について実数 d(S,n) を下記のように定める
d(S,n) := |S∩[2^n,2^(n+1))|/(2^n)
(意味的には区間 [2^n,2^(n+1)) におけるSの密度と捉えてもらえればと)

(補題1)
集合 S⊂N が lim_(n→∞) d(S,n) = 0 を満たす時、lim_(k→∞) |[1,k]∩S|/k = 0 が成り立つ。
(証明)
正の整数 m を任意に固定する。m≦n かつ 2^n≦k<2^(n+1) の時
|[1,k]∩S|/k
≦ |[1,2^(n+1))∩S|/(2^n)
= d(S,n) + d(S,n-1)/2 + d(S,n-2)/4 + … + d(S,n-m)/(2^m) + |[1,2^(n-m))∩S|/(2^n)
≦ d(S,n) + d(S,n-1)/2 + … + d(S,n-m)/(2^m) + 1/(2^m)
→ 1/(2^m). (as n→∞)
よって limsup_(k→∞) |[1,k]∩S|/k ≦ 1/(2^m) が導かれる。
m は任意であったから limsup_(k→∞) |[1,k]∩S|/k = 0. ゆえに示された. (補題1終)

これより、もし A が補題1の仮定を満たすならば A'=A と取れば良いことがわかる。
以下、A が補題1の仮定を満たさないとする。すなわち
r := limsup_(n→∞) d(A,n) > 0
とする。

数列 {q_n} と集合 A'⊂A を次のように定める。
まず q_1 = 2 とする。
(i) d(A,n) ≦ r/q_n の時
d(A',n) = d(A,n) となるように A'∩[2^n,2^(n+1)) を定める。
つまり、区間 [2^n,2^(n+1)) の範囲で A と A' は同一とする。
また、q_(n+1) = q_n とする。
(ii) d(A,n) > r/q_n の時
d(A',n) ≦ r/q_n となるよう、区間 [2^n,2^(n+1)) の範囲で A から最小限の要素を削減する。
また、q_(n+1) = q_n + 1 とする。
このように各 n に対して A'∩[2^n,2^(n+1)) に属する要素を決めてできた集合を A' とする。
0565132人目の素数さん
垢版 |
2022/02/24(木) 23:43:21.40ID:XrbJtKoC
>>562 の続き

r の仮定から (ii) のパターンは無限回起こることが保証されるので、lim_(n→∞) q_n = ∞.
ゆえに lim_(n→∞) d(A',n) ≦ lim_(n→∞) r/q_n = 0 であるから、A' は補題1の仮定を満たす。

また、d(A,n) > r/q_n を満たす非負整数 n 全体を小さい順に {n_k}_(k=2,3,…) と添字付けをすると、
q_n の定義から q_(n_k) = k が導かれる。
一方 d(A,n) > r/q_n の時に A' から取り除かれた要素は最小限であるから
|[2^n,2^(n+1))∩A'| = [(2^n)r/q_n] > (2^n)r/q_n - 1
となる。これらより
Σ_(a∈A') 1/a
≧ Σ_(n=0,∞) Σ_(a∈A'∩[2^n,2^(n+1))) 1/2^(n+1)
> Σ_(n=0,∞) 1/2^(n+1)・((2^n_k)r/q_n - 1)
≧ -1 + Σ_(k=0,∞) 1/2^(n_k+1)・(2^n_k)r/k
= -1 + Σ_(k=0,∞) r/(2k) > ∞.
以上により示された。
0566132人目の素数さん
垢版 |
2022/02/25(金) 00:00:01.44ID:1JyJudrf
>>565 のうち
> Σ_(n=0,∞) 1/2^(n+1)・((2^n_k)r/q_n - 1)
の行は無視してほしい。削除漏れだと思われ
0567132人目の素数さん
垢版 |
2022/02/26(土) 12:14:24.26ID:phtSVANn
>>503

定理
(ai)がnの正の約数からなる長さnの有限列であれば、その部分列で総和がちょうどnとなるものがとれる

補題 定理に反例があるとする
項の総和が最小となる反例をとるとき、aを1でない項の最小値とすると1の個数はa-2個以下である
(∵) 最小反例においてai>1である項に対してその項を含まない部分列全体の中で項の総和がn以下もの全体の中における項の総和の最大値をmiとおく、すなわち
mi
= max{ Σ[j∈J]aj | Jはiを含まない、Σ[j∈J]aj ≦ n }
である
このときmi=n-1である
何故ならはもしmi<n-1となるiがあれはそのaiを1に交換した数列も条件を満たし、かつ総和が真に減るので最小反例であることに矛盾する
特に1である項はmiを構成する項全てに含まれなければならない
またa=aiが1でない項の最小値とし1である項がa-1個以上あるとするとmiを構成する項の中に1がa-1個以上なければならず、そこからa-1個の1を取り除き代わりにaiを追加すれば総和はちょうどnとなり(ai)が反例であることに矛盾する□
0568132人目の素数さん
垢版 |
2022/02/26(土) 12:15:48.58ID:phtSVANn
(定理の証明)
項の総和が最小となる反例をとる
最大素因子をpとしm = p/n とおく
pの倍数がn/p個以上あれば最小反例であることにより主張が成立してしまうので不可能である
よってpの倍数はn/p個に満たない
また1が半数以上あれば補題より2以上の最小の項がn/2+2より大きい事になるがそのようなnの約数はnしかなく矛盾する
よって1は半数未満である
(i) p=2のとき
2の倍数はn/2個に満たないから奇数が半数以上あるがnの素因子が2のみだから奇数の約数は1しかない
よって1が半数以上ある事になって矛盾する
(ii)p≧3のとき
pの倍数はm-1個以下であったが、残りのn-m+1=(p-1)m+1個以上の項はpと互いに素であるからmの約数である
この(p-1)m+1個の項から長さmの部分列がp-1個以上できるから総和がmである共通項がない部分列をp-1個作成できる
0569132人目の素数さん
垢版 |
2022/02/26(土) 12:16:18.15ID:phtSVANn
ここでmの約数である項がさらにm個以上残っていれば仮定からもう一組総和がmとなる部分列をとれるので矛盾する
よってこの時点で(p-2)m+2個以上の項が使われていなければならない
よって作成されたp-1個の部分列は平均して(p-2)m/(p-1)個より多くの項で構成されておらねばならず、特にどれかひとつは(p-2)m/(p-1)個より多くの項で構成されていなければならない
さらにp≧3であるからそれはm/2個より多くの項で構成されていなければいけない
よってΣ[i∈I]ai=m、♯I>m/2となるIがとれる
ai (i∈I)が全て1である事を示そう
そうでないとしai (i∈I)の中の2以上の項の最小値をbとする
ai (i∈I)の中の2以上の項の数をu,1の個数をvとする
項の個数はu+vで仮定より
u+v>m/2‥@
である
一方項の総和はbu+v以上だから
m≧bu+v‥A
である
さらに補題より
v≦b-2‥B
である
@とAにより2(u+v)<bu+vだから(b-2)u<vであるがさらにBより(b-2)u<b-2となりb=2でなければならない
しかしこのとき1が使えなくなるので項の個数は高々m/2となって矛盾する
よってai(i∈I)の全ての項は1である
よって(ai)全体の中に1が少なくともm個あるとわかる
よって補題により1でない項の最小値はm+2以上である
特にmの約数である項は全て1であり結局pの倍数以外の項は全て1である
よって(ai)には少なくともn/p(p-1)+1=n-n/p+1個の1がある
すなわち半分以上が1であるからやはり矛盾する□
0571132人目の素数さん
垢版 |
2022/02/26(土) 15:53:52.69ID:9RRJypaZ
>>570
いえ、フランクルの方ですw
0572132人目の素数さん
垢版 |
2022/03/02(水) 22:31:20.35ID:PrE67Hmx
暇つぶしに

kを自然数としθ=2π/(6k)とおく
単位円上の円周上に3k個の点を隣接する2点によってできる3k個の円弧のうちその円周角がθ,2θ,3θであるのが全てk個ずつとなるように配置する
このとき、いずれか2点は中心に関して対称点になる事を示せ
0573132人目の素数さん
垢版 |
2022/03/03(木) 00:41:37.80ID:p7AYtKkN
>>571
ピーター・フランクルって
おれが小学生の頃、ビートたけしのTVに出てたな。
あの人、今は何してるんだろう…。
0574132人目の素数さん
垢版 |
2022/03/03(木) 11:44:34.14ID:tp/liIHc
>>573
69歳みたいだから教職からは退いておられるでしょうね
>>572もフランクル本に載ってた問題です
これは>>503よりやさしい感じ
0577132人目の素数さん
垢版 |
2022/03/04(金) 09:11:00.84ID:yuQacSCW
引くって分かりにくいな
足すとか加えるとかのほうがいいんじゃないか
0578132人目の素数さん
垢版 |
2022/03/04(金) 12:08:19.77ID:o4vOzVK9
別スレの問題より

Jを平面PのJordan曲線(つまり単射連続写像S^1→Pの像として得られる閉集合)とする
J上の3点で正三角形を成すものがとれる事を示せ

Jordanの閉曲線定理(P\Cはちょうど2つの弧状連結成分を持つ)くらいは使用可とする
0579美魔女
垢版 |
2022/03/07(月) 14:49:50.36ID:iIAOlVsB
√2は無理数であることを示せ。
0580132人目の素数さん
垢版 |
2022/03/07(月) 18:37:19.54ID:+Dwtlelc
>>579
命題Pを次のように定義する。
「Pならば√2は無理数である。」
Pが偽であると仮定すると、
「Pならば√2は無理数である。」は真である。
これはPが偽であることに矛盾する。
従って、Pは真である。
いま、Pかつ「Pならば√2は無理数である。」が真なので、
modus ponensより√2は無理数である。
0581132人目の素数さん
垢版 |
2022/03/08(火) 01:46:09.46ID:2/NFCtsl
1辺の長さがLの立方体ABCDEFGHのAG上に点Pを取る。
点Pを通りFHと平行な直線を回転の軸として、
△PFHを一回転させる。この回転体の体積をVとする。
このとき、Vの最大値及び最小値を求めよ。
   A ── D
   B ── C
   │  │
   E│── H
   F ── G   ←みたいな形
0583132人目の素数さん
垢版 |
2022/03/08(火) 02:42:52.77ID:EKGlS9cx
L=1とする
Pと直線FHの距離をtとして△PFHの面積はt/√2
重心と回転軸の距離は2t/(3√2)
よって回転体の体積はパップス=ギュルダンの定理より
2π×2t/(3√2)×t/√2=2π/3t^2
tはP=Aのとき最大値√(1+1/2)=√(3/2)
EGの中点をMとしてhの最小値mは
1/2√3m=△AGM=△CGM=1/(2√2)
よりm=1/√6
0584132人目の素数さん
垢版 |
2022/03/08(火) 07:45:17.87ID:90v2Ie0f
a[1] = 1/√2,
a[n+1] = (2√a[n])/(1+a[n])
とするとき
lim(n→∞) log(1-a[n])/2^n
を求めよ
0589132人目の素数さん
垢版 |
2022/03/09(水) 01:21:51.52ID:QmneMiTB
ダメだ
やっぱり分からん
とりあえず

x0=1,y0=1/√2からamとgmでxnとynを作る
もちろんlim xn = lim yn = agm(1,1/√2)
求めたいのは
lim log(1-yn/xn)/2^n
...

分からん
もそっとヒントおながいします
0590132人目の素数さん
垢版 |
2022/03/09(水) 01:34:25.18ID:tUKmvr6S
もう少しヒント出します。

まずa[n]が1に収束することを示す。

そして楕円積分を
K(k) = ∫[0,π/2]dθ/√(1-k^2 sin^2(θ)),
K'(k) = K(√(1-k^2)),
(K'/K)(k) = K'(k)/K(k)
で定義するとき
(K'/K)(k) = 2(K'/K)((2√k)/(1+k))
が成り立つことを示します。
0591132人目の素数さん
垢版 |
2022/03/09(水) 02:05:40.56ID:QmneMiTB
あれ?
記憶と違う
前勉強した時は
I(a,b)
=∫[0,π/2]1/√(a^2cos(t)^2+b.^2sin(t)^2)dt
=1/aK'(b/a)
になったはずだけど
さらに
=I((a+b)/2,√(ab))
=2/(a+b)K'(2√(ab)/(a+b)
になるはず
x=b/aとおいても
K'(x)=2/(1+x)K'(2√x/(1+x))
にしかならない
0592132人目の素数さん
垢版 |
2022/03/09(水) 02:27:31.41ID:QmneMiTB
いや、なる
なるほど
K'(x)=2/(1+x)K'(2√x/(1+x))
で(1+x)(1+y)=2となる文字で置換して
K'(y)=2/(1+y)K'(2√y/(1+y))
を利用するんだな
あとは明日考えよ
おやすみなさい
0593132人目の素数さん
垢版 |
2022/03/09(水) 02:28:57.33ID:tUKmvr6S
>K'(x)=2/(1+x)K'(2√x/(1+x))
>にしかならない
それで合ってる

K(x)の方は
K(x)=1/(1+x)K(2√x/(1+x))
になるから
(K'/K)(x) = 2(K'/K)(2√x/(1+x))

最後にK,K'の展開式
K'(x) = (2/π)log(4/x)K(x)
- 2*(1/2)^2(1/(1*2))x^2 - 2*(1*3/(2*4))^2(1/(1*2)+1/(3*4))x^4 - …
を示す(最初のオーダーのみでよい)
0594イナ ◆/7jUdUKiSM
垢版 |
2022/03/09(水) 15:40:17.28ID:Fig4b7W8
>>452
>>581
最大値をV、最小値をvとすると、
ピタゴラスの定理よりAとFHの距離は√(1+1/2)=√(3/2)
最小値を与える点PとFHの距離は(√2/2)(1/√3)=1/√6
∴V=π(3/2)√2(1/3)=π√2/2
v=π(1/6)√2(1/3)=π√2/18
0595132人目の素数さん
垢版 |
2022/03/09(水) 20:30:39.56ID:NtKKk1/f
>>590
まぁもうほとんど答えみたいなヒントだから改めて答え書くのもなんなんだけど、ヒントおながいしたから一応見つけた答え

K(x) = ∫[0,π/2]1/√(1-x^2(sin(t))^2)dt
とおく
置換して
K(x) = (1/2)∫[0,1]1/√(u(1-u)(1-x^2u))du
一方で超幾何関数のオイラーの積分表示から
2F1(1/2,1/2,1; x)
=Γ(1)/(Γ(1/2)Γ(1-1/2))∫[0,1]u^(1/2-1)(1-u)^(1-1/2-1)(1-xu)^(-1/2)du
=1/π∫[0,1]u^(-1/2)(1-u)^(-1/2)(1-xu)^(-1/2)du
により
K(x) = π/2 2F1(1/2,1/2,1; x^2)
である
一方でlog(1+x)のマクローリン展開とオイラーの積分表示により
1/x log((1+x)/(1-x))
= 2・2F1(1,1/2,3/2,x^2)
= 2Γ(3/2)/(Γ(1)Γ(3/2-1))∫∫0,1]u^(1-1)(1-u)^(3/2-1-1)(1-x^2u)^(-1/2)du
= ∫[0,1](1-u)^(-1/2)(1-x^2u)^(-1/2)du
であるから
2K(x)-log((1+x)/(1-x))
= ∫[0,1](1-xu^1/2)u^(-1/2)(1-u)^(-1/2)(1-x^2u)^(-1/2)du
= ∫[0,1]√(1-x^2u)/(√u√(1-u)(1+x√u)du
である
0596132人目の素数さん
垢版 |
2022/03/09(水) 20:30:56.07ID:NtKKk1/f
積分核はu∈(0,1),x∈[0,1]で
√(1-x^2u)/(√u√(1-u)(1+x√u)
≦1/(2√u(1-u))
を満たし、よってx∈[0,1]で一様に可積分だからDCTよりRHSはx∈[0,1]の連続関数でありx=1のとき
∫[0,1](1-√u)/(√u(1-u)du = 2log2
である
よって
f(x)
=2K(x)/K'(x)+log(1-√x)/K'(x)+log(1+√x)/K'(x)
=2K(x)/K'(x)+log(1-x)/K'(x)
も(0,1]で連続でありf(1)=log(64)/πである(∵K'(x)=K(√(1-x^2))は(0,1]で連続、正値、K'(1)=K(0)=π/2)
ここにx=a[n]を代入
f(a[n])
= 2K(a[n])/K'(a[n])+log(1+a[n])
= 2K(a[0])/K'(a[1])×2^(n-1) + log(1+a[n])
はa[n]→1よりlog(64)/πに収束する
よって
lim log(1+a[n])/2^n = - K(a[1])/K'(a[1]) = -1
である
0597132人目の素数さん
垢版 |
2022/03/09(水) 20:36:36.21ID:NtKKk1/f
訂正

>一方で超幾何関数のオイラーの積分表示から
>2F1(1/2,1/2,1; x)

2xlog((1+x)/(1-x))
=2F1(1,1/2,3/2, x^2)
でした
0598132人目の素数さん
垢版 |
2022/03/09(水) 21:05:20.91ID:ssEW6Afm
訂正というか方針転換する前のやつ生き残ってただけか
2F1(1,1,2;x)のくだり無視してもらえばいいだけやった
0599132人目の素数さん
垢版 |
2022/03/09(水) 21:20:30.09ID:tUKmvr6S
えっと、>>595 >>596 の解答で
> K(x) = π/2 2F1(1/2,1/2,1; x^2)
のπ/2が途中で抜け落ちてない?

答えは
lim log(1-a[n])/2^n = -π/2
になるはずだけど
0601132人目の素数さん
垢版 |
2022/03/09(水) 21:46:02.79ID:qGVvcHKs
あ、ほんとだ抜けてるわ
でももういいや、直すのめんどい
自分のノートだけ直して私は落ちます
0602132人目の素数さん
垢版 |
2022/03/09(水) 22:05:01.26ID:4k8d6X/3
あれ?
でも大先生によると積分表示までは間違いない

∫1/√(u(1-u)(1-x^2u))du from 0 to 1
= 2K(x)
(ただし一般的なK(x) = 大先生ではK(x^2))

https://www.wolframalpha.com/input?i=%E2%88%AB1%2F%E2%88%9A%28u%281-u%29%281-x%5E2u%29%29du+from+0+to+1&;lang=ja

x∫(1-u)^(-1/2)(1-x^2u)^(-1/2)du from 0 to 1
= atanh(x)

https://www.wolframalpha.com/input?i=x%E2%88%AB%281-u%29%5E%28-1%2F2%29%281-x%5E2u%29%5E%28-1%2F2%29du+from+0+to+1&;lang=ja
0605132人目の素数さん
垢版 |
2022/03/09(水) 22:15:26.50ID:PwMtik7V
x→1でも収束してる

https://www.wolframalpha.com/input?i=lim+%5Bx%E2%86%921-0%5D+%282K%28x%5E2%29%2FK%27%28x%5E2%29%2Blog%281-x%29%2FK%27%28x%5E2%29%29&;lang=ja

そもそもこの値は÷2^n (n→∞)で消えてしまうから収束するだけで十分で値そのもの関係ないし

そして2K(x)/K'(x) と 2atanh(x)/K'(x)のx→1-0での∞がキャンセルするのはこの値の比以外ありえないし
0606132人目の素数さん
垢版 |
2022/03/09(水) 22:21:45.01ID:tUKmvr6S
>>596 の下から10行目の項: log(1-x)/K'(x)
がその下のf(x)の式で K'(x) が抜けているので
全体として K'(1)=π/2倍違ってくる
0608132人目の素数さん
垢版 |
2022/03/09(水) 22:45:51.03ID:FJM4N7Ut
でも面白いねこの問題
ヒント見てやっと思い出した
これRamanujanのnotebooksのPart3のchapter17のp93でやった事あったのに全然思い出せなかった
ちょっと悔しい
0609132人目の素数さん
垢版 |
2022/03/09(水) 22:48:28.11ID:tUKmvr6S
>>607
ご苦労様です。
そこを修正するだけでよいので正解とします。

ちなみに >>584 の答えの推測
lim log(1-a[n])/2^n = -π/2
を数値的にするには少なくとも数千桁の精度の計算が必要になります。

しかし >>605 の log(64)/π の補正項を加えて
lim log((1-a[n])/8)/2^n
とするとこの極限はagmと同じスピード(2次収束)で -π/2 に収束します。
0611132人目の素数さん
垢版 |
2022/03/09(水) 23:50:25.78ID:tuEuqDGN
>>572

長さが2kの1,3の2文字からなる円順列でi文字目とj文字目がj≡i+n(mod 2n)の時必ず違う文字であるものを反対称円順列と呼ぶ

補題 長さ2kの反対称円順列の中の連続する1の個数をxとするとx≡k+1 (mod 2)である

(∵) “前半”にある隣接する1-1の数をp、3-3の数をqとするとx=p+qである
よって前半にある隣接する1-3または3-1の数はk-xである
前半の開始と後半の開始は違う文字だから前半にある隣接する1-3または3-1の数は奇数でなければならない
よってk-xは奇数である□

問題の解答
反例があるとする
どこかの円弧から正の向きに中心角÷θの値を並べて1,2,3の値からなる長さ3kの数列でできた円順列を考える
反例では長さθの円弧の中心の原点を挟んだ対称の位置には長さ3θの円弧の中心にくる必要があるから、この円順列から2を取り除いて1,3のみの円順列を考えると反対称円順列になる必要がある
よってこの円順列には隣接する1-1の個数をxとするとx≡k+1(mod 2)である
この円順列に2を2k個加えて元の円順列を復元するには1-3が隣接する部分では“反対側”にあたる3-1が隣接する部分には同数の2を加えなければならない
またいずれかが1-1でもう片方が3-3の部分には1-1の方には3-3の方よりちょうど一個多く2を加える必要がある
この条件を満たすように2を加えた時加えられた2の数をlとするとl≡x (mod 2)となる
よって加える2の個数lはl≡k+1(mod2)でなければならないが、題意より加える2の数はkでなければならないので不可能である□
0614132人目の素数さん
垢版 |
2022/03/11(金) 17:33:54.05ID:yIEnHUTH
>>609
ちょっと漸化式をいじって100桁で20項まで計算

https://ideone.com/Wd2SoK

-1.5707943436849058509551755165447994552805531726447795866981023677557487181073360771069987343969219945e0

真値が-1.570796326795だから6桁しか合ってないw
この問題用に数値計算ルーチンをチューニングすればもっといけるだろうけどそこまでやる気はないなぁ
0615132人目の素数さん
垢版 |
2022/03/13(日) 14:55:11.62ID:xyswE62i
a[1] = 2,
b[1] = 1,
a[n+1] = (a[n]+b[n])/2,
b[n+1] = 2a[n]b[n]/(a[n]+b[n])
とするとき
a[n], b[n] をnの式で表し, lim(n→∞) log(a[n]-b[n])/2^n を求めよ
0616132人目の素数さん
垢版 |
2022/03/13(日) 16:43:02.16ID:G9+0ncLr
>>615
c=√(a[1]b[1])、a'[n]=a[n]/c、b'[n]=b[n]/cとおくと
a'[n]b'[n]=c^2、a'[n+1]=(a'[n]+1/a'[n])/2、b'[n+1]=2/(b'[n]+1/b'[n])
∴ a'[n] = coth(2^(n-1)θ1)、b'[n] = coth(2^(n-1)θ1)、ただしθ1=atanh(a[1]/b[1])
∴ a[n] = c coth(2^(n-1)θ1)、b[n] = c tanh(2^(n-1)θ1)
∴ log(a[n]-b[n])/2^n
= log ( c coth(2^(n-1)θ1) - c tanh(2^(n-1)θ1) ) / 2^n
〜log ( coth^2(2^(n-1)θ1) - 1 ) / 2^n
〜log exp (2^n θ1)/2^n
〜θ1 = atanh(a[1]/b[1])
0618132人目の素数さん
垢版 |
2022/03/13(日) 18:15:37.62ID:l2TKaM2i
エレ解を大人気なく解くコーナー

(1) Dixonの公式

3F2(a,b,c,1+a-b,1+a-c,1)
= Γ(1+a/2)Γ(1+a/2-b-c)Γ(1+a-b)Γ(1+a-c)
/Γ(1+a/2-b)Γ(1+a/2-c)Γ(1+a)Γ(1+a-b-c)

を示せ

(2) ΣC[2k+1,j](-1)^j/(2k-2j+1)
=(-1)^k3F2(1,1/2,-k,3/2,k+2,1)

を示しその値を求めよ
0619132人目の素数さん
垢版 |
2022/03/15(火) 13:17:11.62ID:Fp1dexkA
フランクル本より
アイデア1発のやつ
無限個の碁石が
     ●
   ◯ ◯
  ◯ ◯ ◯
 ◯ ◯ ◯ ◯
‥‥
のように配置されている
コレを以下のルールに従って碁石を交換していく

ールールー
いずれかの
 ●
◯ ◯

 ◯
● ●
に取り替える

この時4段目以上の所に必ず黒石が残る事を示せ
0622132人目の素数さん
垢版 |
2022/03/15(火) 20:59:08.03ID:THL0bBmS
ほぼ同じに解けるペグソリティアの問題

座標平面のx≦0またはy≦0の格子点上に無限個のペグが刺さっている
以下のルールでペグを動かす

--ルール(ペグソリティア)--
平面上に

ぺぺ空

があったとき

空空ぺ

にする、ただしこの3つの向きは4方向どちら向きでも可能

全ての格子点にぺグを持ってくることは可能か?
0624132人目の素数さん
垢版 |
2022/03/15(火) 22:05:41.76ID:iIOBLBUC
あるかも
ウインクラー本にもフランクル本にも載ってる有名なやつだからな
最初のはフランクル本に載ってる簡単な方、2番目のはウィンクラーに載ってるややテクニックもいる方
本質同じ
0625132人目の素数さん
垢版 |
2022/03/16(水) 12:03:48.13ID:8ZJTXg/Z
過去スレに似たようなのがあって、
しかし>>620のような不変量・減少量のやり方では微妙に解けなくて、
Grundy数か何かを使うと解ける、みたいなのがあった気がする
0626132人目の素数さん
垢版 |
2022/03/16(水) 15:02:00.63ID:CdCKD83z
減少量で解けます
要するに

・初期配置の総エネルギーが有限
・ぺぺ空→空空ぺで総エネルギーは非増大
・平面全体でポテンシャルは非有界

となるポテンシャルを見つけてくればよい
そのためには

・x≦0, y≦0では“端”にいくほど小さくなる、例えば等比的に減っていって結果総エネルギーが有限になる
・隣接する3点ではほとんど変わらない、よって“一個増える”効果より“2個減る”効果の方が大きくて変形で総和は減少する
・しかし第一象限ではx,y→∞で発散する、よって初期配置の総エネルギー量を一点で超えてしまう点がある

となるポテンシャルを見つけてくれば了
0627132人目の素数さん
垢版 |
2022/03/16(水) 15:08:35.80ID:8ZJTXg/Z
「面白い問題おしえて〜な 十三問目」にあった。

806 名前:132人目の素数さん:2008/03/18(火) 16:39:41
座標平面の原点(0,0)に駒を1つ置き、
次のルールに従って駒の配置を変化させていくとする。

ルール:
平面上に置かれている駒(座標(x,y))を1つ選び、その駒を取り去り、
新たに2つの駒をそれぞれ(x+1,y)と(x,y+1)に置く。

このゲームの目的は、0≦x≦2, 0≦y≦1の範囲から駒を追い出すことである。
果たしてそれは可能だろうか?

807 名前:132人目の素数さん:2008/03/18(火) 16:40:52
>>806
ルール追加。
(x+1,y)または(x,y+1)のどちらかに既に駒が置かれている場合は、
(x,y)の駒を選ぶことはできない。

808 名前:132人目の素数さん:2008/03/18(火) 17:27:14
既出だったような気がするな。 うち帰ったら過去ログ見てみるわ。

809 名前:132人目の素数さん:2008/03/18(火) 18:13:20
既出。

最初の駒を重さ1の粘土だと思い、操作一回につき、粘土を半分に分けると
思うことにする。このとき、何回操作を繰り返しても平面上の粘土の重さの
和は1であり、もし、0≦x≦2, 0≦y≦1の範囲から駒を追い出せたとすると
重さの和が1より小さくなって矛盾。

810 名前:132人目の素数さん:2008/03/18(火) 19:44:12
じゃあ、(0,0),(1,0),(2,0),(1,0),(1,1)に駒が無いようにすることは可能か?

814 名前:132人目の素数さん:2008/03/18(火) 21:08:49
(0,0),(1,0),(2,0),(0,1)(0,2)
から追い払って(1,1)に残すことは可能だから
不可能だとすれば重さだけで示すことはできないね
難問かも
0628132人目の素数さん
垢版 |
2022/03/16(水) 15:28:58.55ID:8ZJTXg/Z
>>627
これの解答がどのタイミングで提示されたかは分からない
かなり時期が過ぎてから解答が書き込まれてた気がする

しかも、>>625で言ったとおり、Grundy数か何かを使った解答だった
少なくとも、減少量による解答は今までに1つも提示されてないと思う
減少量で解けるなら知りたい
0630132人目の素数さん
垢版 |
2022/03/16(水) 15:54:13.73ID:x/iBQDje
減少量で解くってどういうこと?
手によって変化しない盤面の評価数値を定義して
対象になる盤面の評価数値が異なる(減少)ことを示すという回答のこと?
0631132人目の素数さん
垢版 |
2022/03/16(水) 17:12:55.60ID:6te9UE3I
>>630
そうそう
例えば>>619の問題なら

n段目のポテンシャルを(1/2)^(n-1)と設定する
n段目の碁石を一個取り除いて(n+1)段目に2個追加しても総エネルギーは変化しない(減少はしないで十分)
初期配置の総エネルギーは1
5段目以下のポテンシャルエネルギーの総和は
5/16+6/32+7/64+‥=3/4
なので4段目まで全消しするのは不可能

>>627の問題なら点(a,b)のポテンシャルを((√5-1)/2)^(a+b)にすれば例えばx≧4,y≧4の総エネルギーは1未満になるのでそれだけで終わり
でも>>627の問題そのままだと少なくともこのポテンシャルの設定では無理

https://www.wolframalpha.com/input?i=Table%5B++N%28%28%28%28%E2%88%9A5-1%29%2F2%29%5En+-%28%28%E2%88%9A5-1%29%2F2%29%5E%282n%29%29%2F%281-%28%28%E2%88%9A5-1%29%2F2%29%29%5E2%29%2C+%7Bn%2C1%2C10%7D%5D&;lang=ja
0632132人目の素数さん
垢版 |
2022/03/16(水) 17:50:56.83ID:NvJ2oZr4
てかそもそも可能なんじゃないか

◯◯◯◯   ◯◯◯◯  ●◯◯◯  ●◯◯◯  ●◯◯◯
◯◯◯◯   ◯◯◯◯  ●◯◯◯  ◯●●◯  ●●●◯
◯◯◯◯   ●◯◯◯  ◯◯◯◯  ◯◯◯◯  ●◯◯◯
◯◯◯◯   ●◯◯◯  ●◯◯◯  ●◯◯◯  ◯◯◯◯
●◯◯◯   ◯◯◯◯  ◯◯◯◯  ◯◯◯◯  ◯◯◯◯

0≦x≦2, 0≦y≦1から追い出せますがな
0633132人目の素数さん
垢版 |
2022/03/16(水) 17:52:58.62ID:x/iBQDje
>>632
それ問題地学無い?
0636132人目の素数さん
垢版 |
2022/03/17(木) 00:00:04.93ID:pbaudnIc
>>580
これってあらゆる命題が真であることを証明できるよね
どこがおかしいんだろ?
0638132人目の素数さん
垢版 |
2022/03/17(木) 07:35:28.53ID:r0raOLST
いやそこは問題ないよ
そもそも「Pならば√2は無理数である」という自己言及型命題が真か偽のいずれかであると仮定していること自体が間違い
真か偽のいずれかであれば真にならざるを得ない文章
0639132人目の素数さん
垢版 |
2022/03/17(木) 08:51:54.64ID:aCbYP8SG
>>638
>自己言及型命題
命題じゃないよね
0640132人目の素数さん
垢版 |
2022/03/17(木) 09:41:05.32ID:Gp4ZQ1B2
せやね
こんなもん数学の命題ではない
こんなもんを命題と呼んでよいわけがない
こんな話で盛り上がってもしょうがない
教科書読めでええやろ
0641132人目の素数さん
垢版 |
2022/03/17(木) 10:22:55.33ID:tdYH/oTK
カリーのパラドックスって呼ばれてるやつだな
あれが本気で証明になると思って投稿した訳でもないだろうし
単なるジョークの範疇と思ってたけど、知らない人にとっては新鮮な内容だったのかもね
0642132人目の素数さん
垢版 |
2022/03/17(木) 12:51:08.63ID:kyn1eWCR
てかこんなしょうもない話もういいよ
教科書読めで終わりでいい
キリない
うんざり
0643132人目の素数さん
垢版 |
2022/03/17(木) 14:05:16.43ID:VbAF5qjd
Blackpenredpen

n=2→∞
Σ1/(1+ C)^n=2

ζ(2|1/(1+ C)=2)
=
ζ(1)/ζ(1+C)=ζ(2)
=
ζ(
1=ζ(1+C)^2⇔π^2/6
=
6=ζ(1+C)^2
6/2=ζ(1+C)
-ζ(1)+ζ(3)=ζ(C)^2-ζ(1)/2+ζ(3)/2=ζ(C)
)

-1/2-1/2×(3^-(1/2))=C

☆☆☆☆☆
It's important to note
that when algebraic geometry
collaborates with the rotation group,
It makes you dizzy.
0644132人目の素数さん
垢版 |
2022/03/17(木) 18:25:48.30ID:0sckMBX5
排中律が成り立たないからって数学の命題じゃない訳じゃないけどな
0647132人目の素数さん
垢版 |
2022/03/17(木) 20:25:46.48ID:aCbYP8SG
>>646
だから定義してな
0649132人目の素数さん
垢版 |
2022/03/17(木) 21:18:27.28ID:dj8F3d07
てかこんなしょうもない話もういいよ
教科書読めで終わりでいい
キリない
うんざり
0650132人目の素数さん
垢版 |
2022/03/18(金) 00:15:13.92ID:k7cynHUa
>>627
かなり短くなった


以下問題を視覚的に捉えやすくするために●◯■□の4文字で格子点に石の置かれた状態を表現する
それぞれの文字の意味は
●‥現在石があるがいずれそのマスは開けなければならない
◯‥現在石がないが最終的にも空けなければならない
■‥現在石はあるが明け渡さなくてもよい
□‥現在石はなく最終的に明け渡さなくてもよい
である
すると元の問題は

□□□□□□ 
◯◯◯□□□ 
●◯◯□□□ 

のように表示される、もちろん無限には書けないので右側、上側には無限個の□が並んでいると考える

このような●◯■□からなる配列を状態と呼ぶ
ルールに従って石を動かして●がない状態に移行する手順をこの状態の解と呼ぶ
ルールに沿って石Aを取り除き石B,Cを追加した時、石AはB,Cに分裂したと表現するとする
ある状態が解を持つ時、その解において最終状態に至るまで、各格子点で行われた分裂の回数を格子点毎に足し合わせたリストを回数行列と呼ぶ
(例)

□□ □□ ■□ ■■□ ■■□
◯□ ●□ ◯■ ◯□■ ◯■■
●◯ ◯● ◯● ◯●□ ◯◯■ 

において回数行列は
000
110
110

である(必要に応じて行列のサイズは変更される)
0651132人目の素数さん
垢版 |
2022/03/18(金) 00:15:51.81ID:k7cynHUa
今ある解の回数行列が与えられたとき別の解を構成するために次の回数制限ルールで石を動かす事を考える

(回数制限ルール)
ある格子点に元のルール上で分裂可能な格子点であり、その点の回数行列が0でないとき、その石を分裂させてその点の回数行列の値を1減じる

この回数制限ルールで許される行動内において任意に石を分裂させて移動できなくなるまで繰り返した場合、居残り不能点からは石がなくなっていることを示す
回数制限ルールが完全に適用できなくなった状態を考える
その状態で点(a,b)に石が残っているとする
この時(a,b)での回数行列は0になっていなければならない
この時点までで(a,b)においておかなわれた分裂の回数は
(a-1,b)での分裂の回数+(a,b-1)での分裂の回数
+ 1 ( if 初期配置でここに石がある場合)
-1
である
ここで回数制限ルールにより
(a-1,b)での分裂の回数≦(a-1,b)での回数行列値
(a,b-1)での分裂の回数≦(a,b-1)での回数行列値
だったので
(a-1,b)での回数行列値+(a,b-1)での回数行列値
+ 1 ( if 初期配置でここに石がある場合)
- 1
≧ (a,b)での回数行列値
である
この不等式が成立するのは元の手順において(a,b)に石が残っていた場合であり、元の手順が解を与えるものであったことから(a,b)は居残り不能点ではないとわかる
よって回数制限ルールを任意な順で可能な限り続けた場合、居残り不能点から石をなくすことができる
特にこのような方法で他の解を構成できるとわかった
0652132人目の素数さん
垢版 |
2022/03/18(金) 00:16:21.39ID:k7cynHUa
いま状態

□□□□□□ 
◯◯◯□□□ 
●◯◯□□□ 

が解を持つとする
その解での回数行列値は(0,0),(1,0),(2,0)において0ではあり得ないから

□□□□□□ □□□□□□ □□□□□□ 
●◯◯□□□ ●●◯□□□ ●●●□□□ 
◯●◯□□□ ◯◯●□□□ ◯◯◯■□□ 

から始まる解も持たなけれはならない
よってy=0を無視して下に詰めて

□□□□□□ 
●●●□□□ 

も解を持たねばならず同じ論法で

□□□□□□ 
■■■□□□ 
◯●●■□□

も解を持たなければならない
よってx=0を無視して左に詰めて

□□□□□ 
■■□□□ 
●●■□□

も解を持たねばならないが石の総エネルギーは上から
0.5 + 0.25 + 1 + 0.5 + 0.25 = 2.5
□,■の総ポテンシャルはy=0上が0.5、y≧1が2だから全部で2.5しかない
よって不可能である
0653132人目の素数さん
垢版 |
2022/03/18(金) 01:01:15.81ID:+h/OVxVh
>>622

到達目標の座標を(a,b) とした時、φ=(√5+1)/2を用いて、

φ^(-|p-a|-|q-b|)  :p≦a、または、q≦b の場合
φ^(+|p-a|+|q-b|)  :p≧a、かつ、q≧b の場合

のポテンシャルを設定する。

すると、3つ並んだ格子点(x=aあるいは、y=bの直線は跨がない)のポテンシャルの間には、
最も大きいものが、他の2つの和になるという性質を持つ。

Σ[m=0,∞]Σ[n=0,∞]((√5+1)/2)^(-m-n)=((√5+1)/2)^4=(7+3√5)/2=6.8541...
に注意すると、目標座標(a,b)によっては、第ニから第四象限のポテンシャルの総和が 1 を越えないことがある。
そのような場合は、第ニ、第三、第四象限の全ての格子点のポテンシャルの和が、第一象限内のたった一つの格子点(a,b)
のポテンシャルを下回る。
0654132人目の素数さん
垢版 |
2022/03/18(金) 01:05:31.67ID:+h/OVxVh
訂正
前:到達目標の座標を(a,b) とした時、φ=(√5+1)/2を用いて、
後:到達目標の座標を(a,b) とした時、格子点(p,q)には、φ=(√5+1)/2を用いて、
0655&#127822;
垢版 |
2022/03/18(金) 05:11:45.01ID:Ga/3A7/N
SyberMath
sin^5 x+ cos^5x=1
sin^5x+cos^5x=1
ζ(5)sinx/sinx+ζ(5)cosx/cosx=ζ(1)
ζ(5sinx)+ζ(5cosx)=ζ(5)
ζ(sinx+cosx)=ζ(1)
±sinx±cosx=
[±1,0]
x=[±1,0)
x={0,π/2}
0656132人目の素数さん
垢版 |
2022/03/18(金) 08:00:31.38ID:2C5p9SMR
>>653
素晴らしい
正解です
やっぱりφ使いたくなりますね
別にピッタリ=にならなくてもいいんですが
私の用意した解答も
V(a,b)
= φ^((3min{max{0,a},max{0,b}}-|a|-|b|)/4)
でした
元ネタはウインクラーの“Pegs on the Half-plane”
こっちは初期のペグがy≦0全体でその場合はいける限界の理論値=計算機で見つけた解まで見つかってるという話をどっかで聞いた記憶があります(8段目くらいだったかな?)
私のはどこまでいけるんでしょうねぇ?
0657132人目の素数さん
垢版 |
2022/03/18(金) 08:17:25.31ID:ozc8TnUI
あ、訂正

V(a,b)
= φ^((3min{max{0,a},max{0,b}}-max{|a|+|b|})/4)

まぁなんでもいいんですけど
min{max{0,a},max{0,b}}はx=0,y=0を裾野にしたピラミッドみたいなグラフで
-max{|a|+|b|})/4)は原点が頂点のピラミッドのグラフ(いわゆるマンハッタン距離|| OP ||_∞
どちらも隣接点で高々1しか違わない
それを3:1の割合で混ぜたもの
0658132人目の素数さん
垢版 |
2022/03/18(金) 08:41:00.03ID:Ga/3A7/N
SyberMath

x^5-y^5=1993
d^6x^5/d^6x-d^6/d^6y=1993
d/dx-d/dy=1993
d^2y-d^2x=
1993d^2xy
y-x=1993xy
0659132人目の素数さん
垢版 |
2022/03/18(金) 08:44:07.06ID:Ga/3A7/N
訂正
y^5にすべきところをyにしてしまったw
0660132人目の素数さん
垢版 |
2022/03/18(金) 08:54:38.14ID:Ga/3A7/N
訂正
SyberMath

x^5-y^5=1993
d^6x^5/d^6x-d^6y^5/d^6y=1993
d/dx-d/dy=1993
d^2y-d^2x= 1993d^2xy
y-x=1993xy
0661132人目の素数さん
垢版 |
2022/03/18(金) 09:43:55.96ID:dkzYfEig
>>650-652
元の問題をよく読んで。627で問われている最初の問題は

□□□□□□ 
◯◯◯□□□ 
●◯◯□□□ 

であるが、これは減少量で普通に解ける。
そして、次の問題で問われているのが

□□□□□□ 
◯◯□□□□ 
●◯◯□□□

であって、こちらは減少量による解法が未だに提示されてない、ということ。
0663132人目の素数さん
垢版 |
2022/03/18(金) 10:09:24.21ID:t3U2pU+z
>>661
もしかして粘土で半分のやつ?
それおかしくない?
元の粘土は1
y≧2の総ポテンシャルだけで1、そこにy=1,x≧2の0.125とy=0,x≧3の0.25を加えると総ポテンシャルは1.375で元の1を軽く超えてしまう
だからそもそも0≦x≦2, 0≦y≦1の場合ですらポテンシャル論法だけでは解けないと思うんだけど
0664132人目の素数さん
垢版 |
2022/03/18(金) 10:21:17.22ID:dkzYfEig
>>663
言われてみればそうだな
でも石の配置をちょっと考察するだけで言えた気がする
まあそれだと「>>650-652でもよい」ということになるが、どのみち本題は

□□□□□□ 
◯◯□□□□ 
●◯◯□□□

の方であって、こちらは「減少量+石の配置の考察」でもぜんぜん歯が立たない
0666132人目の素数さん
垢版 |
2022/03/18(金) 11:36:04.11ID:dkzYfEig
一応、もともとの

□□□□□□ 
◯◯◯□□□ 
●◯◯□□□ 

については、

・ どこまで操作を施しても、x=0の範囲にはちょうど1個しか石がない
・ どこまで操作を施しても、y=0の範囲にはちょうど1個しか石がない

が成り立つことが簡単に言えるので、これを使って自明なポテンシャル計算をすると、
もし該当の領域から石が追い出せたなら、残りの領域でのポテンシャルが
ギリギリ1を保っていなければならないことが分かる。しかし、そのためには
無限個のマスが石で覆われている必要があり、かろうじて矛盾する。

・・・というやり方でいいはず。
0667132人目の素数さん
垢版 |
2022/03/18(金) 11:52:57.99ID:DNWDAMvf
>>664
できた

----
ある回数行列下で解があると仮定

□□□□□ □□□□□ □□□□□ 
◯◯□□□ ●●■□□ ●●■□□ (←y=0を取り去って下げる)
●◯◯□□ ◯◯◯■□ 

ここで(0,0),(1,0),(2,0)の石をA,B,Cとする
ここで回数制限ルールに加えて次のルールを追加する

--- 優先子孫ルール ---
移動可能な石が複数ある場合Cの子孫、Bの子孫、Aの子孫の優先順位で移動する

このルール下でも移動不能状態に到達すれば石は居残り不能点には残らないことがわかる
そこでC,Bの子孫が全て移動不能になった後の事を考えると

□□□□□
●◯□□□

の状態になっている
この状態からAが居残り不能点に残らず分裂するためにはB,Cの子孫が(1,1),(2,0)に残っていることはできない
よってB,Cの子孫の移動は

□□□□□
□◯□□□ 
◯●●□□ 

の解でなければならない
同じ論法を用いてCの子孫については

□□◯□□
□◯◯◯□ 
◯◯●◯□ 

の解でなければならない
コレはx≧2の部分が元の問題と同じ形であるが、既に何回か分裂して回数行列は小さくなっているので元の問題より真に制限が厳しくなっているから矛盾
0668132人目の素数さん
垢版 |
2022/03/18(金) 12:05:35.98ID:dkzYfEig
>>667
うーむ。これが正しいのかすぐには判断できんが、
仮に正しいとしても、使われている減少量は「回数行列」であって、
マスごとに重さを付与するような、いわゆる通常の減少量ではないですね

で、回数行列の減少性に回帰させるのは有限オートマトン的な方法であって、特に

>コレはx≧2の部分が元の問題と同じ形であるが、

の部分がまさしく有限オートマトン的であって、結局それは、
かつて見かけたGrundy数の方法と本質的に同じなんじゃないかっていう
0670132人目の素数さん
垢版 |
2022/03/18(金) 12:19:29.53ID:dkzYfEig
>>669
実は、本当にGrundy数だったかは覚えてない
それっぽい感じの「〇〇数」を使う解法だった
でもGrundy数で合ってた気はする

過去ログにあるはずだが、ちょっと候補が多すぎて探せない

経緯としては、>>627の問題が書き込まれたのが「面白い問題おしえて〜な 十三問目」で、
Grundy数(?)を使った解法が書き込まれたのは大分あとのはずで、
それがどのタイミングだったか全く覚えてないので、検索のとっかかりがない。
なので、探すなら今までの過去ログを全部探すしかない・・・
というか、ワンチャン全く別のスレだった可能性もなくはない・・・
0671132人目の素数さん
垢版 |
2022/03/18(金) 12:22:08.25ID:DNWDAMvf
>>670

コレかな?

https://algo-logic.info/combinatorial-games/#:~:text=Grundy%E6%95%B0(Nim%E6%95%B0%2FNimber,%E3%81%A7%E3%81%8D%E3%82%8B%E3%82%88%E3%81%86%E3%81%AB%E3%81%AA%E3%82%8A%E3%81%BE%E3%81%99%E3%80%82

この手の問題で必ずできるもの的なこと書いてあるけどこの問題に関してどうやればいいかは一般論からはまるでわからないね
0672132人目の素数さん
垢版 |
2022/03/18(金) 13:58:22.71ID:os5B4Sl+
□□□
□□
↑この形のピースだけで5マス×nマスの長方形を隙間、はみ出し、重なりが無いように
敷き詰めることが可能であるような正の整数nを全て求めよ。
ただし敷き詰めるにあたって、ピースの回転や反転はして良いものとする。
0673132人目の素数さん
垢版 |
2022/03/18(金) 17:23:35.81ID:dkzYfEig
「面白い問題おしえて〜な 25問目」までをざっと手作業で目を通してみたが、
「面白い問題おしえて〜な 十八問目」で次の問題が書かれていた。

891 :132人目の素数さん:2011/10/24(月) 16:41:46.46
座標平面上の原点(0,0)に駒が1個置かれている。
(a,b)に置かれている駒を取り除き、(a,b+1)と(a+1,b)に駒を1個ずつ置くという操作を考える。
ただし、(a,b+1)と(a+1,b)のどちらにも駒が置かれていない場合のみ可能とする。
この操作を繰り返し行ったところ、(0,0),(0,1),(1,0),(1,1)のどの点にも駒が無い状態になった。
この時、(0,2),(1,2),(2,0),(2,1)の4点全てに駒が置かれていることを証明せよ。

ご覧のとおり、>>627の最後に書かれている問題よりもさらに強い問題になっている。
で、これの解答は同じ「面白い問題おしえて〜な 十八問目」の中で提示されていて、
やり方は回数行列と同じもの(スレの中では回数図と呼ばれている)を使っていた。
0674132人目の素数さん
垢版 |
2022/03/18(金) 17:25:43.36ID:dkzYfEig
解答の書かれ方が、自分の記憶の中にうっすらある映像と何となく一致しているので、
おそらくこれが、自分が目撃した解答だと思われる。
つまり、「Grundy数を使った解答」というのは自分の記憶違いだったようだ。すまん。

ただし、自分がGrundy数という言葉を知ったのは、「この問題の解答でGrundy数が使われていたから」
だったはずなので、この問題で実際に使われていたのがGrundy数ではなく回数行列(or回数図)なのであれば、
じゃあ自分は一体どこでGrundy数を知ったのか、という個人的な謎が残ってしまうが、
まあそれはさておき、回数行列(or回数図)を使えばこの問題が解けるというのは確定っぽい。
0675132人目の素数さん
垢版 |
2022/03/18(金) 17:28:55.04ID:dkzYfEig
そして、マスごとに重さを付与するような、いわゆる通常の減少量による解答が
未だに存在しないのも確定っぽくて、実際に「面白い問題おしえて〜な 十八問目」の中でも、
そこでの出題者が

967 :891:2011/10/30(日) 00:39:47.72
>>965
ある本で読んだ問題が元ネタなんですが、その問題は
「3×3の領域から駒を追い出すことができない」ことを示すもので、
解答は>>927の考え方を用いたものでした。
さらに強い結論を求めてみたところ、>>891の問題ができました。
>>927の考え方を知ってる人は、逆に悩まれたかもしれませんね。

と述べている。ここでの「927の考え方」とは、
いわゆる普通の減少量(不変量・ポテンシャル・エネルギーとも呼ばれる)
を使った手法のことを指していて、少なくともここでの出題者は、
そのような手法による解答を持ってないことになる。
なので、現状でのまとめとしては、

・ 回数行列(or回数図)を使えば解ける
・ いわゆる普通の「エネルギー的な手法」による解法は、やはり今のところない

といった感じか。
0676132人目の素数さん
垢版 |
2022/03/18(金) 18:22:58.18ID:ZNhbQwbz
>>675
まぁしかし

□□□□□
◯◯□□□
●◯◯□□

が解なしなのであとは

□□□□□
◯◯◯□□
●◯□□□

が解なしを確認するだけやな

ここから

□□□□□
●●●□□
◯◯□■□

に入ってコレが解なしは既出

なので

❇︎❇︎□□
◯◯❇︎□
◯◯❇︎□

では❇︎は全部石が残る
0677132人目の素数さん
垢版 |
2022/03/18(金) 21:37:36.15ID:mPg+/YjH
訂正
◯◯◯ ●●◯
●◯□ ◯◯■
の(2,0)の石は分裂させたらダメだなので

□□□
●●◯

にしか還元できない
ここから(1,0)の子孫を先に分裂したあとの(0,0)の子孫の問題は

□□□ ■□□ ■■□ ■■■□
●◯◯ ◯●◯ ◯◯● ◯◯◯■ 

なので(1,0)の子孫は

◯◯◯□□
◯●◯◯□

を解かねばならないがコレは解なしであった
0678132人目の素数さん
垢版 |
2022/03/19(土) 00:57:11.23ID:2j4iXk9D
別板で出てた問題
有名どこだけど暇つぶしに

n^2 | 2^n + 1

を満たす自然数nを決定せよ
0679132人目の素数さん
垢版 |
2022/03/19(土) 04:10:42.17ID:5alSoufR
nを非負の整数とする

Σ[k=0,n] C[n,k]^2 (1+x)^k = Σ[k=0,n] C[n,k] C[n+k,k] x^(n-k)

を示せ(ただしC[n,k]は二項係数)
0681132人目の素数さん
垢版 |
2022/03/19(土) 12:58:37.83ID:CZn82nev
>>672
スーパー力技ではできた

■■■
■■□

のように欠けてる1マスを埋め戻した3×2の長方形の重心をそのブロックの概重心と呼ぶとする

領域0≦x≦5、0≦y≦nのタイリングに対してy≧aに概重心があるタイルだけを集めてできる領域の0<x<5、0<y<nの部分にある折線P(a)について考える
P(a)は慨重心がy=a,a-1/2,a-1,a-3/2のいずれかであるブロックの上ヘリのなす折線で全てa-1≦a+1の範囲に収まらなければならない
よってP(a)のx座標が1/2,3/2,5/2,7/2,9/2である点のy座標はa-1,a,a+1のいずれかでなければならない
記述を簡明化するためにそれぞれのy座標が例えばa+1,a+1,a,a-1,a-1となる場合にP(a)=a+[1,1,0,-1,-1]のように書くとしよう
中央の数字が0であるP(a)をよい分割と呼ぶ
P(a)より下の領域の面積が5の倍数でなければならないから[〜]の中の数字の和は5の倍数でなければならない
また1番目と2番目、4番目と5番目の数字の差が2以上であるとタイリングできないのは容易にわかるからよい分割で許されるのは
[1,1,0,-1,-1], [1,0,0,-1,0], [1,0,0,0,-1], [0,1,0,-1,0], [0,0,0,0,0]
とこれらの幾何学的対称で移り合うものに限られる
しかしこの中でa+[1,0,0,-1,0]はy≦aの部分がタイリング不能であり、a+[0,1,0,-1,0]のものはy≧aの部分がタイリング不能とわかるので可能であるよい分割は
[1,1,0,-1,-1], [1,0,0,0,-1], [0,0,0,0,0]
の3種類だけであり、これらは

ああいいい うううええ きききくく
あああいい おううええ ききくくく
      おおかかえ けけけここ
      おおかかか けけこここ

のように実例がある
それぞれI型、II型、III型と呼ぼう
奇数nについて領域0≦x≦5、0≦y≦nのタイリングが可能であるものがあるとしてnが最小である反例をとる
今P(a)がよい分割であるとき実例から分かる通りI型のときはa又はn-aのどちらか奇数である方を1に、II型、III型のときはa又はn-aのどちらか偶数の方を2に取り替えられるので、その操作でサイズが小さくなることは最小反例では許されない
よって最小反例はn≧9ではあり得ない
(例えばn=9ならP(3),P(4),P(5)のいずれかがよい分割でそれが何型でもより小さいサイズに変更できる)
よって最小反例はn=3,5,7に限られる
0682132人目の素数さん
垢版 |
2022/03/19(土) 12:58:43.35ID:CZn82nev
n=7で最小反例となりうるのはP(2),P(5)がよい分割でどちらもII型でさらにP(3),P(4)はよい分割であってはならない
この条件を満たすには下4ブロックは

  あ
 ああいい
うああいい
ううええい
ううえええ

しか許されないがこの後左側が埋まらない
n=5で最小反例となりうるのはP(2)またはP(3)がII型のよい分割であるときかP(2)もP(3)もよい分割でなくP(1),P(4)が共にI型の分割となるときしかない
それぞれup to 対称性で

あああいい ◯◯えええ
◯ああいい ◯◯◯ええ
◯◯◯◯い ◯◯◯◯◯
◯◯◯◯◯ おお◯◯◯
◯◯◯◯◯ おおお◯◯

のどちらかであるがどちらもこの先つまる
n=3が反例たり得ないのは容易
0683132人目の素数さん
垢版 |
2022/03/19(土) 14:25:53.97ID:u8BB+Mfa
>>681
『〜〜に取り替えられるので』の部分がギャップな気がするなあ

最小反例をnとした時、例えばあるaについて
P(a)が[1,1,0,-1,-1]のパターンだったとしても、P(a)付近のタイリングが実際に
ああいいい
あああいい
のようになってるとは限らないと思うんだけどどうだろう

下みたいなパターンの可能性もある気がする
……………
ええううお
ええううい
ああういい
あああいい
0684132人目の素数さん
垢版 |
2022/03/19(土) 15:04:09.83ID:LukHuTT/
>>683
P(a) = a + [1,1,0,-1,-1]という状態はともかくP(a)による分割の境界集合が

[0,1]×{a+1}∪[1,2]×{a+1}∪[2,3]×{a}∪[3,4]×{a-1}∪[4,5]×{a-1}
∪(...最大四つの縦線分...)

という事を主張しているのでこの時点までの主張が正しいなら取り替えられるはず

ええううお
ええううい
ああういい
あああいい

の下ヘリがx軸の場合

P(1) = 1 + [1,1,0,-1,-1] (慨重心がy≦1は“あ”のみ、よい分割)
P(2) = 2 + [ 0,0,-1,0,1] ((慨重心がy≦2は“あ”と“い”、よい分割でない)
P(3) = 3 + [ -1,-1,1,1,0] ((慨重心がy≦3は“あ”,“い”,"う"、よい分割でない)
よい分割でないものはもちろんもっとたくさんあります
それは相手にしてない
0685132人目の素数さん
垢版 |
2022/03/19(土) 16:07:12.12ID:u8BB+Mfa
>>684
あーーやりたいことがわかったぞ、例えばもし
……………(偶数行)
ええううお
ええううい
ああういい
あああいい
……………(奇数行)
のように[1,1,0,-1,-1]型が出現した場合、P(a)より下の領域をそのままにして上の領域を
ああいいい
あああいい
……………(奇数行)
のようにしてより小さい反例が作れるってことか、なるほど

P(a)を左端から出発して途中で←の方向に進まないこととかは明らかだしいいか、正解!(というより何よりお疲れ様…)
0686132人目の素数さん
垢版 |
2022/03/19(土) 16:16:26.35ID:u8BB+Mfa
想定解はこんな感じ

nを奇数とし、5×nの長方形の一部を以下のように塗る
□□□□□
□■□■□
□□□□□
□■□■□
□□□□□
……………
□■□■□
□□□□□
各々のピースは2x2の正方形を含むので、少なくとも一つの■を覆うことになる。
5×nの長方形を覆うためにはn個のピースが必要であるが、
長方形には■がn-1個しかないため、ピースが重ならないように置くためには
高々n-1個しか置けない。ゆえに不可能
0687132人目の素数さん
垢版 |
2022/03/19(土) 17:30:40.92ID:1DHE0GmN
>>679
LHS
=Σ( (n_k/1_k (-1))^k )^x^k
= 2F1(-n,-n,1,1+x)
= 2F1(-n,n+1,1,(x+1)/x)(-x)^n
(∵ Pfaff's transform )
RHS
= Σ ( (-n)_k/1_k (-1)^k) (n+1)_k/1_k x^n/x^k
= 2F1(-n,n+1,1,-1/x)x^n
よって
2F1(-n,n+1,1,t)=2F1(-n,n+1,1,1-t)(-1)^n
を示せば十分
両辺共にODE
t(1-t)u''+(1-2t)u' +n(n+1)u = 0
の解であり
t=1において左辺は明らかに1、右辺は
(1-(n+1))_n/1_n(-1)^n = 1
t=1において左辺'は-n(n+1)/1、右辺'は
n(n+1)(1-(n+1))_(n-1)/2_(n-1)(-1)^n = -n(n+1)
で等しいからODEの解の一意性により主張は成り立つ
0689132人目の素数さん
垢版 |
2022/03/19(土) 18:50:36.49ID:5alSoufR
>>687
正解です。おめでとうございます。

この問題の解答は複数あって、用意していた解答の1つを紹介します。

f(t) = (1/n!) (d/dt)^n (t^n (1+t)^n)
を二通りで評価する

(1)ライプニッツルールで展開
f(t) = (1/n!)Σ[k=0,n] C[n,k] ((d/dt)^k t^n) ((d/dt)^(n-k) (1+t)^n)
= Σ[k=0,n] C[n,k] n(n-1)...(n-k+1) t^(n-k) n(n-1)...(k+1) (1+t)^k / n!
= Σ[k=0,n] C[n,k]^2 t^(n-k) (1+t)^k

(2)普通の二項展開
f(t) = (1/n!) (d/dt)^n Σ[k=0,n] C[n,k] t^(n+k)
= Σ[k=0,n] C[n,k] (n+k)(n+k-1)...(k+1) t^k / n!
= Σ[k=0,n] C[n,k] C[n+k,k] t^k

ゆえに
Σ[k=0,n] C[n,k]^2 t^(n-k) (1+t)^k = Σ[k=0,n] C[n,k] C[n+k,k] t^k
が成り立ち 1/t=xと置くことで目的の結果が得られる
0690132人目の素数さん
垢版 |
2022/03/19(土) 23:02:26.44ID:DoAI5f0Q
20世紀最難問らしい

凸六角形 ABCDEF において,AB//DE,BC//EF,CD//FA とする。また,三角形 FAB,BCD,DEF の外接円の半径を Ra,Rc,Reとおく。
また,六角形の周の長さをP とおく。
このとき

Ra+Rc+Re ≧ P/2

を証明せよ。
0691132人目の素数さん
垢版 |
2022/03/20(日) 11:01:40.41ID:6UYxivWx
>>690
ついでなのでコレも貼っとく

2009 IMO ドイツ大会

a(1),a(2),⋯,a(n) を相異なる正の整数とし、M を n-1個の正の整数からなる集合と
する。また、M は s=a(1)+a(2)+⋯+a(n) を含まない。数直線の 0 の地点にいるバッタが
数直線の正の向きに n 回ジャンプする。 n 回のジャンプの距離は a(1),a(2),⋯,a(n) の並べ替えである。このとき並べ替えをうまく選べば、バッタがM の要素に対応するn-1点に一度も着地しないようにできることを証明せよ。

完答者の少ない3問だって
他2つは

https://manabitimes.jp/math/956>>690>>678
0693132人目の素数さん
垢版 |
2022/03/21(月) 01:42:27.50ID:02awIXLb
自作問題です
「一辺の長さ1の正素数角形の対角線の長さは必ず無理数となることを示せ.」

次の問題は予想なのですが解けていません
もし分かったらお願いします
「辺の長さと対角線の長さが有理数比になりえる正多角形は正六角形のみか?」
0694132人目の素数さん
垢版 |
2022/03/22(火) 13:33:40.64ID:WD7pVtm7
>>693

以下自然数nに対してθn=2π/n、ζn=exp(iθn)、cn=cos(θn)、sn=sin(θn)とする
γ(n)をnと4の最小公倍数とする

補題 nを自然数とし、l = γ(n)とするときQ(ζl) = Q(sn,i)であり
[ Q(ζl) : Q(sn) ] = 4 ( n ≡ 4 ( mod 8 ) , n≠4 )
. = 2 ( otherwise )
である
(∵) (a) n ≡ 0 ( mod 4) のとき
Q(sn)はsn,cnを含むからQ(sn,i)はζlを含み[Q(ζl):Q(sn)]は高々2である
しかし(sn)は虚数であるζlを含めないのでちょうど2である
(b) n ≡ 1,3 ( mod 4) のとき
Q(sn)はc_nを含む(∵sin(θ×[(n±1)/4]2∓1) = c_n)からQ(sn,i)はζnを含むとわかる
ζl はζn^kのうち最もiに近いものを用いてi/ζn^kに一致するかその逆数に一致するかなのでζlもQ(sn,i)に含まれる
よって[Q(ζl):Q(sn)]は高々2である
しかし(sn)は虚数であるζlを含めないのでちょうど2である
(c) n ≡ 2 ( mod 4 ), n≠4のとき
Q(sn,cn,i)はζlを含むから[Q(ζl):Q(sn)]は高々4である
ここでGal(Q(ζl)/Q)の4個の元
ζ→ζ、ζ→1/ζ、ζ→-ζ、ζ→-1/ζ
は全てsn=(ζ-1/ζ)/(2ζ^(n/4))を動かさないから[Q(ζl):Q(sn)]は少なくとも4である□
0695132人目の素数さん
垢版 |
2022/03/22(火) 13:34:44.87ID:5oU5xQVR
補題 nを自然数、mをその約数とするときQ(sm) = Q(sn)となるのは
・(m,n) = (4,12)
・m = n
・n ≡ 2 ( mod 4 )、m = n/2
のときに限る

(∵) k = γ(m),l=γ(n)として次の図式を考える

0 → L → Gal(Q(ζl)/Q) → Gal(Q((sn)/Q) → 0
. ↓ ↓ ↓
0 → K → Gal(Q(ζk)/Q) → Gal(Q((sm)/Q) → 0

でK,Lが位数2,4しかないから可能であるのは
(1) (♯K,♯L) = (2,4)、[Q(ζl):Q(ζk)] = 2
(2) ♯K=♯L 、[Q(ζl):Q(ζk)] = 1
しかない
(1)において[Q(ζl):Q(ζk)] = 2となるのは(k,3)=1, l = 3kである場合かkが偶数でl = 2kである場合しかない( ∵ [Q(ζl):Q(ζk)] = φ(l)/φ(k)でこれが2しか違わないなら奇素数の因子の違いが3だけ違うか同じであるかしかない )
前者の場合(♯K,♯L) = (2,4)となるのはn≡4 (mod 8)である必要があり、さらにm/≡4 (mod 8)またはm=4が必要である
さらにn/mが奇素因子を持たないならm=n/2,n/4に限られる
しかしこの時k=lとなり[Q(ζl):Q(ζk)] = 2に反する
よってn/mは奇素因子pをもつがφ(k)/φ(l)=2によりpのnにおける多重度は高々1でありφ(p)=2でなければならない
∴p=3である
またそれ以外の素因子をn/mが持てはφ(k)/φ(l)=2にはなれないからn/m=3である
さらにn≡4 (mod 8)によりm≡4 (mod 8)だが♯K=2となるにはm=4しかない
よって(m,n)=(12,4)しかないとわかる
(2)後者の場合[Q(ζl):Q(ζk)] = 1からφ(k) = φ(l)でありk,lともに4の倍数だからk=lである
よって4|mの場合にはm=k=l=nである
そうでないときは4|nとするとn≡4 (mod 8)なら♯L=4、♯K≠4となり矛盾、よってnは4の倍数ではなくm=nでなければn≡2 (mod 4)、m=n/2しか有り得ない□
0696132人目の素数さん
垢版 |
2022/03/22(火) 13:35:24.74ID:5oU5xQVR
定理 正多角形で一辺の長さと対角線の長さの比が有理数となるのは正六角形の最長対角線の場合に限る

(∵) ある単位円に内接する正l角形のある対角線と辺の長さの比が有理数であったとすれば対角線の長さはb = 2sin(πk/l) (2≦k≦l/2)とおける
一辺の長さはa = 2sin(π/l)である
n = 2l、θ=2π/n、d = (n,k) とおけばd = pn + qkとなるp,qがとれるがnが偶数ゆえqは奇数であるよってこのとき
sin(dθ)
= sin(pnθ)cos(qkθ) + cos(pnθ)sin(qkθ)
= sin(qkθ)
となる
よって
Q(sin(dθ)) = Q(sin(qkθ)) = Q(sin(kθ))
である
(∵qが奇数だからsin(qkθ)∈Q(sin(kθ)であり、qr≡1 ( mod n )となる奇数r=q^(φ(n)-1)がとれるからsin(kθ)∈Q(sin(qkθ))
よってm=n/dとおけばsin(dθ) = sin(2π/m) =smでありmはnの約数でQ(sm) = Q(sn)となる
よって補題のいずれかの条件が成立する
(m,n) = (4,12)の場合は正六角の場合であるからよい
m=nの場合はd=1であり(sin(kθ))^2と(sin(θ))^2はガロア群の作用で共役元である
よってr = (sin(kθ))^2/(sin(θ))^2が有理数なら
(sin(kθ))^2=r(sin(θ))^2
において両辺のトレースをとればt = tr((sin(kθ))^2) = tr((sin(θ))^2) > 0により
rt = t
であるからr=1でなければならず矛盾
n ≡ 2 (mod 4)、m=n/2の場合はl=n/2-kとすればsin(lθ) = sin(kθ)、(n,l) = 1となってd=1の場合に帰着される□
0697132人目の素数さん
垢版 |
2022/03/22(火) 19:27:36.89ID:5J3VcTm7
>>694-696
おおおおお解答ありがとうございます!!
すごい 超大作をありがとう

Q(sin(2π/n))=Q(sin(2π/m))
の条件の一つとして(m,n)=(4,12)が出てきて、それが正六角に対応するというのはとても面白いです
論文にしてほしいレベルだ
先行研究みたいなのはあるのかな?
0698132人目の素数さん
垢版 |
2022/03/22(火) 19:28:29.10ID:5J3VcTm7
供養のため>>693の用意していた解法を載せます

pを奇素数としてp角形で考える

対称性から2≦k≦p/2を満たす自然数kに対して
L=sin(kπ/p)/sin(π/p)の無理性を示せば十分

(k-1)次第二チェビシェフ多項式Uを用いて
L=U(cos(π/p))と書ける
よってLが有理数ならば、cos(π/p)は有理数係数の(k-1)次多項式の根となるが、
[Q(cos(π/p)):Q]=φ(2p)/2=(p-1)/2>k-1
より矛盾
0699132人目の素数さん
垢版 |
2022/03/23(水) 00:12:19.42ID:6RHgbvl8
楽しめてもらえてよかったんですが今見返すとちょっと議論にジャンプがあるので差し替えます
どの部分かはわかると思いますが図式から位数の議論から始めて(m,n)=(4,12)の場合を導出するところです

----
(1)において[Q(ζl):Q(ζk)] = 2となるのは(k,3)=1, l = 3kである場合かkが偶数でl = 2kである場合しかない( ∵ オイラー関数φの性質からわかる)
しかし(♯K,♯L) = (2,4)となるのはn≡4 (mod 8)である必要があり、よってl ≡ 4 (mod 8)である
さらに4|kでもあるからl=2kの場合は起こり得ない
∴(k,3)=1, l = 3kである
図式において右の↓が誘導されるには左の↓が誘導されなければならない、すなわちLの元が↓によってKに入ることが必要である
Lの元は補題の議論で示された通りζl→1/ζlとζl→ζl^(l/2+1)によって生成される
ζl→1/ζlは複素共役をとるガロア群の元であり↓によってKに移る
ζl→ζl^(l/2+1)は↓によってζk→ζk^(l/2+1)に移されるがこれがKの元であるのは
l/2 + 1 ≡ ±1 ( mod k )
のときである
さらにk = l/3の場合を考えているので
l/2 + 1 ≡ ±1 ( mod l/3 )
3l + 6 ≡ ±6 ( mod 2l )
l + 6 ≡ ± 6 ( mod 2l )
であるが+の方は解なしであり
l + 12 ≡ 0 ( mod 2l )
の解はl = 4, 12であるが 3 | l よりl=12となり(m,n) = (4,12)が必要である
これは条件を満たす
0700132人目の素数さん
垢版 |
2022/03/23(水) 00:13:09.46ID:6RHgbvl8
この手の話どのくらい先行研究があるかよくは知りませんが円分体のガロア群の作用は死ぬほど調べられてると思います
今回のζへの作用のテクニックとかはネットでも論文いっぱい見つかります
こういうふうにガロア群の作用が実際に“見て取れる”のがアーベル拡大がよく研究されてる理由であり、あるいはでない場合どうすればいいかわからず途方に暮れる理由であり、だから数体以外のアーベル拡大の構成問題とかが求められるんでしょうね
これと同じような議論を数体より大きい体でやりたいから
0702132人目の素数さん
垢版 |
2022/03/23(水) 02:29:51.60ID:cobDS5or
主張も綺麗だしワンチャンどっかのハゲタカ雑誌に掲載出来ないかな?
0703132人目の素数さん
垢版 |
2022/03/24(木) 11:30:07.30ID:KE7UIA+G
130km/hで走って20分かかる距離を100km/hで走ると何分かかりますか?
0704132人目の素数さん
垢版 |
2022/03/25(金) 00:03:41.27ID:Ofdt7rKg
半分問題半分質問

微分方程式

ds/√(1-s^2)(1-k^2s^2) = du (u(0)=0)

の解をヤコビの楕円関数s = sn(u)と呼ぶ
要するに∫[0,s]dx/√((1-x^2)(1-k^2x^2)の逆関数
さてこの時次が成立する事を示せ

sn(u+v) = ( sn(u)sn(v')+ sn'(u)sn(v) )/√(1-k^2sn^2(u)sn^2(v))
(加法定理)

答えあるんだけどかなりめんどい
おまけに同じような関数cn、dnがあって同じく加法定理を証明せんといかん
誰かいいの見つけたらあげてください
よくある証明だと丸々1ページ計算せんといかん
0706132人目の素数さん
垢版 |
2022/03/25(金) 23:07:11.29ID:W5jHHXLF
>>705
ありがとうございました。
0707132人目の素数さん
垢版 |
2022/03/26(土) 03:53:58.59ID:V658FGdN
P(x)を複素数係数の多項式とする
Σ[k=1,∞] (-1)^k P(k^4) k^5 / sinh(πk) = 0
を示せ
0708132人目の素数さん
垢版 |
2022/03/26(土) 12:15:36.92ID:KL22jCSR
そういや似たやつでk^5/(e^2πk-1)みたいなのなかった?
どつかのスレで見た記憶が
誰かログ残ってない?
0712132人目の素数さん
垢版 |
2022/03/26(土) 17:18:18.57ID:EGlRSlrq
ん?
イヤ
普通にアーベルプラナで足せばいいのかな?

k≧5に対してf(x) = x^k/sinh(x) はx=0 でf(0)=0で連続に拡張できるから

I=Σ n^k/sinh(πn)
= ∫[0,∞] x^k/sinh(πx)dx + i∫[0,∞]( f(iy) - f(-iy) )/(e^(2πy)-1)dy

だけど第2項は0

J=Σ (2n)^k/sinh(2πn)
=∫[0,∞] (2x)^k/sinh(2πx)dx + i∫[0,∞]( f(i2y) - f(-i2y) )/(e^(2πy)-1)dy

で同じく第2項は0
第一項は2x→xの置換で結局I=2J
で偶数項の寄与がちょうど半分だから交代和は0
0714132人目の素数さん
垢版 |
2022/03/26(土) 17:50:02.63ID:V658FGdN
続きの問題を貼ります

P(x)を定数項が1の有理数係数の多項式とする
Σ[k=1,∞] (-1)^k P(1/k^4) k / sinh(πk)
は0にはならないことを示せ
0716132人目の素数さん
垢版 |
2022/03/26(土) 20:11:13.78ID:V658FGdN
>>715
Γ(1/4)は出てこない
πの超越性のみでいける

>>712
よく考えたらこの解答、不備があると思う
分母のsinh(πx)のπがπ以外の正の数でよいことになるがこれはおかしい

正しくは第二項は0ではない
f(i),f(2i),f(3i),...が極でアーベルプラナを導出するときの虚軸上の積分が発散積分になるので
極を中心とする小半円だけ路をよけて留数の半分の値を評価する必要がある
0717132人目の素数さん
垢版 |
2022/03/26(土) 20:32:01.77ID:8YCFjQD5
>>715
でも定数項の積分には出てくるよね?
定数項以外は全部同じ値になるんじゃないの?
0718132人目の素数さん
垢版 |
2022/03/26(土) 20:38:01.56ID:8YCFjQD5
極のところちょっとよければいいんじゃないの
避ける幅小さく取れば極に近づいた分で大きくなるのと路が小さくなる効果で結局回避する半円部分の積分値はres×πiになるけど上下で±0になるでしょ?
途中のむすんでるところももちろん±ゼロだし
0720132人目の素数さん
垢版 |
2022/03/26(土) 21:34:32.56ID:J4In1tXb
>>719
なぜ?
f(iy)-f(-iy)を半円上積分だよね?
半円上でこれ自体そもそも消えてない?
なんか何違いしてるかな
0721132人目の素数さん
垢版 |
2022/03/26(土) 21:39:46.60ID:J4In1tXb
あ、ホントだ
Σk^a/sinh(k)は必ずΓ(1/4)絡みになると思ってた
交代和でΓ(1/4)消されるのか
0723132人目の素数さん
垢版 |
2022/03/26(土) 22:22:55.01ID:V658FGdN
>>720
アーベルプラナのwikipediaの証明(f(z)は偶関数で虚軸上に極があると仮定)
https://en.wikipedia.org/wiki/Abel%E2%80%93Plana_formula
でa=i+ε→i+0で極限を取るときの被積分を見ると
a f(at)→i f((i+0)t)は虚軸の右側境界を積分し
a^(-1)f(a^(-1)t)→-i f((-i+0)t)=-i f((i-0)t)は虚軸の左側境界を逆方向に積分する
したがってf(z)の留数が出る
0724132人目の素数さん
垢版 |
2022/03/26(土) 22:25:08.21ID:J4In1tXb
せやね
それで結局虚軸上の極はf(iy)とf(-iy)は反対に交わさないといけなくなるから一周することになるんやね
0726132人目の素数さん
垢版 |
2022/03/28(月) 00:31:12.15ID:Nb46Se1t
>>712 の解答の修正版を以下に残しておきます。

k≧5, k≡1(mod 4)に対して
虚軸上の留数補正したアーベルプラナの公式より
I = Σ n^k/sinh(πn)
= ∫[0,∞] x^k/sinh(πx)dx - 2Σ(-1)^n n^k/(e^(2πn)-1)

J = Σ(2n)^k/sinh(2πn)
= ∫[0,∞](2x)^k/sinh(2πx)dx - Σ(-1)^n n^k/(e^(πn)-1)

2J-I = Σ(-1)^n n^k/sinh(πn)
= 2Σ(-1)^n n^k(1/(e^(2πn)-1)-1/(e^(πn)-1))
= -Σ(-1)^n n^k/sinh(πn)
= -(2J-I)

したがって
2J-I = 0


>>707 の用意していた解答は以下の通りです。

f(z) = P(z^4) z^5 π/(sinh(πz)sin(πz))と置く
f(z)の極はz=n, inにあり(nは0でない整数)留数は
Res[z=n]f(z) = Res[z=in]f(z) = (-1)^n P(n^4) n^5 / sinh(πn)

したがって留数定理より
4Σ[n=1,N] (-1)^n P(n^4) n^5 / sinh(πn) = 1/(2πi)∫[C]f(z)dz
ここでCは点(1+i)(N+1/2),(1-i)(N+1/2),(-1-i)(N+1/2),(1-i)(N+1/2)を周回する路

n→∞とすると|∫[C]f(z)dz| = O(N^(4+deg(P))/e^(πN)) → 0
より目的の結果が得られる
0727132人目の素数さん
垢版 |
2022/03/28(月) 00:35:38.45ID:JCoUMP1h
>>726
あれ?
そこ2J-I = -(2J-I)になつた?
オレ計算した時2J-I = 2J-Iで振り出しに戻ってダメだった気がするんだけど
0728132人目の素数さん
垢版 |
2022/03/28(月) 00:51:23.85ID:JCoUMP1h
>>726
その計算ほんとにk≡1(mod4)使ってる?
多分素直に留数をまとめ直すと2J-I = 2J-Iに戻ってしまう
k≡1(mod4)ならまとめ方弄って2J-I = -(2J-I)にたどり着けるのかもしれないけど
少なくともこの条件なしでは反対符号に行き着くはずないと思う、その条件ない場合にはそもそも数値実験的に0にならないのは確定的だから
0729132人目の素数さん
垢版 |
2022/03/28(月) 01:00:11.31ID:Nb46Se1t
>>272
k=5,k=9に関して数値的に確認したところ
>>726 のIとJの積分と和の式は合ってる

和を有限にして2J-Iの式を数値的に確認したけど問題なさそう
0730132人目の素数さん
垢版 |
2022/03/28(月) 01:11:50.22ID:JCoUMP1h
>>729
ホント?
それどこでk≡1(mod4)使ったん?
もちろん留数まとめ直す作業はなんならkが偶数でも行う事ができて和はΣ[n]n^k(q^(2n)-1)の形になって偶数項だけ拾い上げた方Σ[n]n^k(q^(n)-1)の形にはなる(コッチは留数が0でない点が半整数になって/(q^2n-1)の和が/(q^n-1)の和になる
偶数項の方2倍する効果が留数が半分になってて打ち消しあって引き算したとききれいにq^n(q^n-1)になって偶数項からやってきたq^n-1の方が綺麗に消えてくれてq^n/(q^(2n)-1)になってもとのsinh(x)に戻るとこまで確認して「お、できたか?」と思ったんだけどよくよく考えたらそんなはずない、そもそもどこにもk≡1(mod4)使ってない、使わずにできるはずないと思って係数見たら振り出しに戻ってただけと気づいたまでが昨日の寝る前だったんだけど
同じ事やってない?
0731132人目の素数さん
垢版 |
2022/03/28(月) 01:29:11.76ID:Nb46Se1t
>>730
f(x) = x^k/sinh(πx)
と置くと
Res[x=ni]f(x)/(e^(-2πix)-1)
= ((ni)^k/((-1)^n))/(e^(2πn)-1)
ここでk≡1(mod4)の時
= i (-1)^n n^k/(e^(2πn)-1)
0732132人目の素数さん
垢版 |
2022/03/28(月) 01:38:14.38ID:JCoUMP1h
>>731
イヤ別にk≡1(mod4)であろうがなかろうが留数を取る点に違いは出ないでしょ
分母が0になるのはsinh(πx)でこれが0になるのはx=miとなるときである事にkの値は関係ない
で、その時のxの値であるmiをx^k/(e^(2πx-1)に代入する事もkの値がなんなら偶数でも支障なく代入出来る
これだけでは元に戻らないけど、後で2×偶数項のみにも同じ事ができるのもk≡1(mod4)出なくても出来るでしょ?
0733132人目の素数さん
垢版 |
2022/03/28(月) 01:39:48.35ID:Nb46Se1t
>>731
1/πつけ忘れたので訂正
Res[x=ni]f(x)/(e^(-2πix)-1)
= ((ni)^k/(π(-1)^n))/(e^(2πn)-1)
ここでk≡1(mod4)の時
= (i/π) (-1)^n n^k/(e^(2πn)-1)
0734132人目の素数さん
垢版 |
2022/03/28(月) 01:41:36.74ID:JCoUMP1h
なんか信じられないなぁ
やっぱり元に戻ってるだけなの勘違いしてるんだと思うよ
0735132人目の素数さん
垢版 |
2022/03/28(月) 01:44:42.81ID:Nb46Se1t
>>733
だからx^kにx=miを代入すると
m^k i^k になるから i=i^kが成り立つ条件はk≡1(mod4)
0736132人目の素数さん
垢版 |
2022/03/28(月) 02:02:31.51ID:JCoUMP1h
やっと昨日のノート見つけた
やっぱり元に戻ってるだけじゃない?
0737132人目の素数さん
垢版 |
2022/03/28(月) 02:05:25.49ID:JCoUMP1h
>>735
イヤ、どっちかというとk≡1(mod4)でない場合は元に戻って意味のない式になり、なぜかk≡1(mod4)のときだけ元に戻らず和が0になる証明になるという現象が起こった事になる
長いこと勉強してるけどそんなの見たことないよ
0738132人目の素数さん
垢版 |
2022/03/28(月) 02:07:34.83ID:JCoUMP1h
偶数と奇数で片側だけ0にならなくて〜はあるけどな
k≡1(mod4)のときだけ元に戻らず、k≡1(mod4)でないときは元に戻るとか流石にないやろ
0739132人目の素数さん
垢版 |
2022/03/28(月) 02:09:12.32ID:Nb46Se1t
> イヤ、どっちかというとk≡1(mod4)でない場合は元に戻って意味のない式になり、なぜかk≡1(mod4)のときだけ元に戻らず和が0になる証明になるという現象が起こった事になる
そのとおり!

例えばラマヌジャンの有名な和
Σn^k/(e^(2πi)-1)
はk=5,9,13,…の時に有理数になる
0740132人目の素数さん
垢版 |
2022/03/28(月) 02:10:56.71ID:JCoUMP1h
さようですか
それはそれは大発見をなされましたな
素晴らしい
おやすみなさい
0741132人目の素数さん
垢版 |
2022/03/28(月) 02:31:39.32ID:Nb46Se1t
ところどころミスがあるから適当に修正してください

>>739 の訂正
Σn^k/(e^(2πn)-1) = (1/2) B[k+1] / (k+1)
ただしk=5,9,13,…でB[n]はベルヌーイ数
この式はよく知られていてWolframのデータベースにある

>>726 の想定解
× n→∞とすると|∫[C]f(z)dz| = O(N^(4+deg(P))/e^(πN)) → 0
〇 N→∞とすると|∫[C]f(z)dz| = O(N^(6+deg(P))/e^(πN)) → 0

ではおやすみなさい
0742132人目の素数さん
垢版 |
2022/03/28(月) 20:05:36.89ID:Nb46Se1t
>>726 の補完解答、一晩寝て冷静に考えたけど間違いが見つからない。
Wolfram先生も正しいらしいと言っているのですが、なんか勘違いしていたら指摘お願いします。

以下Wolframでの検証:k=5に固定
Iの式
https://www.wolframalpha.com/input?i=%CE%A3%5Bn%3D1%2C%E2%88%9E%5D+n%5E5%2Fsinh%28%CF%80n%29&;lang=ja
Σ[n=1,∞] n^5/sinh(πn)
≒ 0.253522
https://www.wolframalpha.com/input?i=%E2%88%AB%5B0%2C%E2%88%9E%5D+x%5E5%2Fsinh%28%CF%80x%29dx&;lang=ja
https://www.wolframalpha.com/input?i=%CE%A3%5Bn%3D1%2C%E2%88%9E%5D+%28-1%29%5En+n%5E5%2F%28e%5E%282%CF%80n%29-1%29&;lang=ja
∫[0,∞] x^5/sinh(πx)dx - 2Σ[n=1,∞] (-1)^n n^5/(e^(2πn)-1)
≒ 0.25 - 2*(-0.00176091)
= 0.25352182
で数値的には正しいらしい

Jの式
Σ[n=1,∞] (2n)^5/sinh(2πn)
≒ 0.126761
∫[0,∞](2x)^5/sinh(2πx)dx - Σ[n=1,∞] (-1)^n n^5/(e^(πn)-1)
≒ 0.125 - (-0.00176091)
= 0.12676091
でこちらも正しいらしい

最後に2J-Iの右辺と左辺の和の項の符号のチェック
和の全項を調べるのは面倒なので、初項と第二項のみを調べる
2J-I左辺(定義式)の2項=2(Jの初項)-(Iの初項と第二項)
= 2*((2*1)^5/sinh(2π*1))-(1^5/sinh(π*1)+2^5/sinh(π*2))
≒ 0.03292721409509456116
2J-I右辺(アーベルプラナ)の2項=2J-Iの積分を省いた和の初項と第二項
-2((-1)^1 1^5/(e^(π*1)-1)+(-1)^2 2^5/(e^(π*2)-1))
+ 2((-1)^1 1^5/(e^(2π*1)-1) + (-1)^2 2^5/(e^(2π*2)-1))
≒ -0.03292721409509456116
で右辺と左辺の項の符号がちょうど逆になっていて問題なさそう
0743132人目の素数さん
垢版 |
2022/03/28(月) 21:39:32.66ID:LlhnH+Nu
すまん、オレが昨日間違ってた
問題はその議論がn^a/sinh(πn)のaの類によらず成立してるとまずいからおかしいかという話だけど、よく考えたらそもそもaが偶数ならx^a/sinh(x)が偶関数にならないからその時点で通用しないし符号はi^(a+1)の形でaは寄与してるからa≡1(mod 4)で反符号、a≡3(mod 4)で同符号でもおかしくはない
よくよく考えてみたらこういう現象が起こっててもおかしくないと思う
しかし間違いは確実に間違ってると断言できることが多いけどあってる方は神に祈るしかない
0744132人目の素数さん
垢版 |
2022/03/28(月) 22:51:06.57ID:Nb46Se1t
確認ありがとう

ところで>>741 の等式
> Σn^k/(e^(2πn)-1) = (1/2) B[k+1] / (k+1)
> ただしk=5,9,13,…でB[n]はベルヌーイ数
はWolframのサイトなどを検索しても出てこないようで
よく知られているというのは勘違いだったようです、すまない
自分なりの証明はできているけどきちんとした文献等が検索しても出てこないので
誰か出典を知っていたら教えてほしい
あるいはこの等式を証明してくれても結構です
0745132人目の素数さん
垢版 |
2022/03/29(火) 23:53:37.06ID:H7yi5y5a
>>691
a(1)<...<a(n)としてよい
M={m(1),...m(n-1)},
m(1)<...<m(n-1)としてよい

nについての帰納法
n≦2のとき容易だからn≧3とする

(I) m(n-1)≦s-a(m)のとき
必要ならa(1)〜a(n-1)を並び変えて帰納法の仮定からa(1),a(2),...,a(n-1)と飛んでm(n-1)以外の地点は全てかわしてs-a(n)地点に至れるとしてよい
Σ[k≦i]a(i)≧m(n-1)となる最小のkがとれる(∵仮定から少なくともi=n-1で成立するので必ず存在する)
Σ[k≦i]a(i)=p、Σ[k≦i]a(i)=qとしてp<m(n-1)≦q、q-p=a(k)<a(n)であるからp<m(n)<p+a(n)である
よってa(1),a(2),...,a(k-1),a(n),a(k),..,a(n-1)と飛べば全ての禁止地点をかわせる

(ii) s-m(n) < m(n-1) < s、かつs-a(n)が禁止地点でないとき
(ii)と同じく必要ならa(1)〜a(n-1)を並び変えて帰納法の仮定からa(1),a(2),...,a(n-1)と飛んでm(n-1)以外の地点は全てかわしてs-a(n)地点に至れるとしてよい
この場合はs-a(n)の地点からsへ飛べはよい

(iii) s-m(n) < m(n-1) < s、かつs-a(n)が禁止地点のとき
s-a(m) = m(k)とする
この時あるlをs-a(l)とs-a(l)-a(m)のいずれも禁止地点でないl∈[1,n-1]がとれる、なぜならそうでないとすると全てのl∈[1,n-1]でs-a(l)かs-a(l)-a(m)のいずれかひとつが必ず禁止地点となり、それらの地点は全て相異なるからこれでn-1個の禁止点全てが尽くされなければならないが、この中にs-a(n)は含まれないので仮定に反してしまう
帰納法の仮定からa(l)とa(n)以外のn-2回のジャンプを並べ替えてm(n-1)とm(k)以外の全ての禁止地点をかわしてs-a(l)-a(n)の地点まで到達出来る
そこからa(n)、a(l)と飛べば追加される着地地点はs-a(l)だけだから条件を満たす
0746132人目の素数さん
垢版 |
2022/03/30(水) 17:55:27.85ID:nol0jTHH
>>707
これ当然楕円関数論の解答あるやろと思って図書室で調べてきたらあった
Jacobiの
Fundamenta Nova Theoriae Functionum Ellipticarum
にいっぱい公式載ってる
読んでよくわかった
ラマヌジャンのnotebookにも似た公式いっぱい載ってるけど大本はこのヤコビの理論なんやな
この本に「詳細は省くけどこの調子で任意の自然数に対して計算するアルゴリズムがとれる」というのを実際にやってみてるのがラマヌジャンのnotebookの計算なんだと改めて知った
もうヤコビの時代から基本アルゴリズムは確立してたんやな

------

ノームをq=exp(−πK′/K)とし、引数をv=πu/(2K) と変換する。このとき

sn^2(u)
= 4K^2/π^2-4KE^I/π^2
- 8 Σlq^l/(1-q^(2l)) cos(2lv)

であるから

sn(K-u)^2
= 4K^2/π^2-4KE^I/π^2
- 8 Σlq^l/(1-q^(2l))(-1)^l cos(2lv)

でありj≧1に対してvの関数としての2j次のmaclaurin expansionの係数は

-8(-1)^j2^(2j)/(2j)! Σ l^(2j+1)q^(l)/(1-q^(2l))

である
一方で

sn^2(K-u)
= cd^2(u)
= (cn/dn)^2(u)
= (dn^2 + (-1+k^2) sn^2)/dn^2
= 1 - k†^2sd^2(u)

により-1+sn^2(K-u)は偶関数でありさらに

-1+sn^2(K-iu)
= - k†^2sd^2(iu)
= - k†^2(sn(iu)/cn(iu))^2
= - k†^2(i sn†(u)/cn†(u) cn†(u)/dn†(u))^2
= + k†^2(sd†(u))^2
= 1 - sn†^2(K†-iu)

である
よって特にk=1/2のときは2j ≡ 0 (mod 4)の項はvanishする□
0748132人目の素数さん
垢版 |
2022/03/30(水) 22:47:45.49ID:7F3OQ1oJ
参考までに上で利用した証明に利用した公式のうち骨がおれる公式はp109 sec.41 の(1)式

(2kK/π)^2 sn^2(2K/π x)
= const. -8Σl q^l(1-q^(2l)) cos(2lx)

と言っても骨がおれるのはconst.の決定でそれ以外の項はその直前の

2kK/π sn(2Kx/π) = Σ[l:odd]4√q^l/(1-q^l)‥@

が絶対収束してる事を利用して足し算の順番を組み替えるだけなのでそこまで難しくはない
@は有理形関数の部分分数展開公式とsn(u)の極の位置がu =2mK + (2n-1)K'iである事からも出せる(本では無限乗積展開を分解すればいいとだけ書いて詳しい解説はなし)
ちなみにRamanujan' notebookのch. 17 entry 15 にΣk^a/sinh(ky)の形の公式が山のように載ってるけど結局entry 16(p. 135)にあるようにsn(u)のlambert expansionして両辺のmaclaurin expansionを比較するという方法で本質はsn(u)のlambert expansion、この計算の方針がヤコビの教科書のsec 43以降に書いてあるようだ
でもあくまで方針で実行されてるのはsn^(-6)〜sn^6くらいまで
あくまでsn^kのlambert expansionがほしいと思ったときどうやればいいか指針があるだけで結果を手短にまとめるのは難しい模様
アルゴリズム整理してパッと一瞬で出せるプログラムとかできそうだけどな
0749132人目の素数さん
垢版 |
2022/03/31(木) 17:18:30.95ID:ScwMDuej
>>744 の出典がわかった

Eisenstein級数
G_{2k}(τ) = Σ[(m,n)≠(0,0)] 1/(m+nτ)^(2k)
のFourier展開
G_{2k}(τ) = 2ζ(2k) + 2(2πi)^(2k)/(2k-1)! Σ σ_{2k-1}(n) e^(2πinτ)
をLambert級数に直してζ(2k)の特殊値と2k≡2(mod 4)のときの値
G_{2k}(i) = 0
∵G_{2k}(1/i) = i^(2k) G_{2k}(i)
を代入することで得られるようです
0750132人目の素数さん
垢版 |
2022/03/31(木) 21:48:20.84ID:ScwMDuej
>>744 のk<0の公式の記述と出典が
リーマンのゼータ関数、松本耕二、朝倉書店 pp21-22にあった。
それによるとRamanujan's Notebooks Part II の Ch.14 にもあるようで
よく知られているようです。

ついでにk≧5の手持ちの解答を貼っておく。

kを5以上の整数でk≡1(mod 4)とする
このとき
Σ[n=1,∞] n^k/(e^(2πn)-1) = ∫[0,∞] x^k/(e^(2πx)-1) dx ---- (1)
が成り立つ
これを仮定すれば
(1)の右辺
= Σ[n=1,∞]∫[0,∞] x^k e^(-2πnx) dx
= Γ(k+1)Σ[n=1,∞] 1/(2πn)^(k-1)
= k!ζ(k+1)/(2π)^(k+1)
= B_{k+1}/(2(2k+1))

以下(1)を示す
f(z) = z^k πcot(πz)coth(πz),
g(z) = z^k πcoth(πz)
Nを正の整数としてCを点(-1-i)(N+1/2)から点(1-i)(N+1/2)の向きの直線とする

補題
1/(2πi)∫[C] f(z)dz = Σ[n=1,N] n^k coth(πn),
1/(2π)∫[C] g(z)dz = Σ[n=1,N] n^k + 2∫[0,N+1/2] x^k/(e^(2πx)-1) dx - R,
R = 2im(∫[0,N+1/2](N+1/2+iy)^k/(e^(2π(N+1/2+iy))-1) dy)
が成り立つ

証明の概要は
f(z)に関してはCを原点中心にπ/2づつ回転させた正方形の閉曲線で積分し
g(z)に関してはCと実軸を含む長方形の閉曲線で積分し対称性を用いて簡略化する

この補題でN→∞とすると
∫[C]|f(z)-ig(z)||dz| = O((N+1/2)^(k+1) e^(-2π(N+1/2))) → 0,
R = O((N+1/2)^(k+1) e^(-2π(N+1/2))) → 0
であることから(1)が得られる
0751132人目の素数さん
垢版 |
2022/04/01(金) 00:53:39.07ID:DpJuElcS
というわけで結局楕円関数のランベルト級数分解が自由にできるようにならないと何も始まらない

-- 問題 --

sn(u), cn(u), dn(u)のnormに関するランベルト級数表示を導出せよ

ヤコビの教科書だと解説が一瞬すぎて笑ってしまうやつです
これくらいできて当たり前というノリなんですかねぇ?
0752132人目の素数さん
垢版 |
2022/04/02(土) 21:24:21.52ID:AN4rv3Vl
>>751
sn(u)についてFourier級数で計算してみた

まずsn(u)はu=iK',2K+iK'に極を持つ周期4Kと2iK'の二重周期関数であることに注意する
(これは加法定理とJacobi's imaginary transformationから確認できる)

sn(u) = Σ[n∈Z] c[n] e^(iπnu/(2K))
この係数は
c[n] = 1/(4K)∫[0,4K] sn(u)e^(-iπnu/(2K)) du
である
そして被積分関数をf(u)と置き0から4Kまでの積分を
・C1:0から2iK'の線分(ただしiK'の極は左側の微小半円でよける)
・C2:2iK'から4K+2iK'の線分
・C3:4K+2iK'から4Kの線分(ただし4K+iK'の極は左側の微小半円でよける)
の積分に置き換える
C1とC3の積分は周期性から打ち消しあい
c[n] = 1/(4K)∫[C2]f(u)du + 2πi/(4K)(Res[u=iK']f(u)+Res[u=2K+iK']f(u))
= c[n] e^(πnK'/K) + πi/(2kK) (1-(-1)^n)e^(πnK'/(2K))
したがって
n:odd のとき c[n] = πi/(kK) e^(πnK'/(2K))/(1-e^(πnK'/K))
n:even のとき c[n] = 0 (n=0の時はsn(u)が奇関数であることから成り立つ)

以上q=e^(-πK'/K), v=πu/(2K)と置いてまとめると
sn(u) = 2π/(kK)Σ[n=0,∞] q^(n+1/2)/(1-q^(2n+1)) sin((2n+1)v)


同様にしてsn(u)^(2m+1),m=1,2,..を展開してみた
sn(u)^(2m+1) = 2π/(k^(2m+1)K)Σ[n=0,∞] a[m,n] q^(n+1/2)/(1-q^(2n+1)) sin((2n+1)v),
a[m,n] = k^(2m+1) Res[u=iK'] sn(u)^(2m+1) e^(-iπ(2n+1)(u-iK')/(2K))
= Res[u=0] e^(-iπ(2n+1)u/(2K)) / sn(u)^(2m+1)

この留数をsn(u)のマクローリン展開から具体的に計算すると
a[1,n] = (1/2)(1+k^2 - π^2 (2n+1)^2/(2K)^2),
a[2,n] = (1/8)(3+2k^2+3k^4) - (5/12)(1+k^2)π^2 (2n+1)^2/(2K)^2 + (1/24)π^4 (2n+1)^4/(2K)^4,
...
でヤコビの教科書のSec.44 Iと一致する
0753132人目の素数さん
垢版 |
2022/04/02(土) 22:11:30.51ID:lvAcfRlC
>>752
多分正解されてそうですね
細かい計算はチェックしょうがない
まさにフーリエ変換を計算せよです
私が用意してた解答はとりあえずまずsn(u)、cn(u)、dn(u)の部分分数分解表示を求めてしまうとこから始めます
とりあえず
s(x) = 2kK/π sn( 2Kx/π )
とまず規格化します
α=π/2、β=πK'/(2K)iとおいてs(x)は周期4α、4βの周期関数
極はx = aα+bβで一位の極であり留数は(-1)^(b+1)/2
よってcsc(x)の部分分数分解表示により
s(u) + C = Σ[b:odd]( csc(x - bβ) + csc( x + bβ ) )
= Σ[b:odd](sin(x+bβ) + sin(x-bβ))
/ (sin(x+bβ)sin(x-bβ))
= Σ[b:odd](sin(x+bβ) + sin(x-bβ))
/ (sin(x+bβ)sin(x-bβ))
= Σ[b:odd]4sin(x)sin(bβ))
/ (cos(2bβ) - cos(2x))
= Σ[b:odd]4sin(x)√q^b(1+q^b)
/ ( q^(2b) - 2q^bcos(2x) + 1)
でx=0を代入してC=0を出します
同様の作業を
c(x) = 2kK/π cn( 2Kx/π )
d(x) = 2K/π( 2Kx/π )
に対して行うと
c(x) = Σ[b:odd](-1)^((b+1)/2)4sin(x)√q^b(q^b-1)
/ ( q^(2b) - 2q^bcos(2x) + 1)
d(x) = 1 + 4Σ[b:odd]σ(b)(q^2b-1cos(2x))
 / ( q^(2b) - 2q^bcos(2x) + 1)
を得ます(少なくともsn(x),cn(x)はヤコビの教科書に結果が載っててそれと同じなのであってると思います)
最後のd(x)の定数項は
∫[0,K]dn(u)du
= ∫[0,K](asin(sn(x))' dx
= [ asin(sn(x)) ]^K_0
= π/2
で求められます
この部分分数分解表示を利用すれば実方向へのフーリエ展開も学部生の演習レベルの積分です
しかしこのランベルト級数めちゃくちゃ面白い
例えばs(π/2)=c(0)ですがこれから
√q/(1-q) - √q^3/(1-q^3) + √q^5/(1-q^5) - ...
=
√q/(1+q) + √q^3/(1+q^3) + √q^5/(1+q^5) - ...
なんでのが出てきますが、マジか?と思って確認してみると確かに正しい、分母展開すればわかります
ちなみにこれはヤコビの教科書の§40くらいで最も基本的な等式としている(5)の等式です
ヤコビは(4)と(5)の同値性をモジュラー変換で出してますが、実はdn(x)の展開の定数項の計算からも導くことができるようです
とはいえモジュラー変換はこの教科書の目玉中の目玉でそれを避けたら読む意味ないですけど
0754132人目の素数さん
垢版 |
2022/04/02(土) 22:35:36.68ID:cBoHVSYd
次を満たす実数 c∈(1,2], K>0 は存在するか:
どんな正の整数もある 0 以上の整数 m,n を用いて [Kn^c + Km^c] と表せる
0756132人目の素数さん
垢版 |
2022/04/03(日) 20:01:24.49ID:mYXJRv+I
>>755
ヒント 答えは『存在する』
S(x,t) := (x+t)^c + (x-t)^c とおくと、
0≦t≦x^(cのみに依存するある定数) の時 |∂S/∂t|≦(cのみに依存するある定数) が言えるのでごにょごにょ
0759132人目の素数さん
垢版 |
2022/04/03(日) 20:58:06.76ID:5iImpOWr
c = 1+1/mとする、m≧3とする

S(t) = (x+t)^(1+1/m) + (x-t)^(1+1/m)において
S'(t) = (1+1/m)( (x+t)^(1/m) - (x-t)^(1/m) )
. = (1+1/m) 2t/( (x+t)^((m-1)/m) + ... + (x-t)^((m-1)/m) )
. < 3t / ( m (x - t)^((m-1)/m)

∴ S'(t) < 1
← 3t < ( m (x - t)^((m-1)/m)
← (3t)^m < m^m (x-t)^(m-1)
← t < x^((m-1)/m) & 3≦m
よって0≦t≦[x^((m-1)/m)] で変化するとき[S(t)]は[2x^c]〜[S([x^((m-1)/m)])の間の全ての整数値をとる
ここで
[S(x^((m-1)/m)])]≧[2(x+1)^(1+1/m)]
←S(x^((m-1)/m)])≧2(x+1)^(1+1/m)
←(x+x^((m-1)/m)-1)^(1+1/m) ≧ 2(x+1)^(1+1/m)
←x + x^((m-1)/m) -1 > 2^(1/m)(x+1)
←x≧3, m≧100000000
そこでm≧100000000を
・2×3^(1+1/m)<7
・[ a^(1+1/m) ] = a ( if a =1,2,3,4,5,6 )
となるように選べば良い
0760132人目の素数さん
垢版 |
2022/04/03(日) 23:04:59.09ID:mYXJRv+I
>>759
方針は良い感じだけどいくつかミスがあるっぽい

> ← (3t)^m < m^m (x-t)^(m-1)
> ← t < x^((m-1)/m) & 3≦m
これ十分条件になってるかねえ
最後の条件からは (3t)^m < m^m x^(m-1) までしか言えない気がする

> ←(x+x^((m-1)/m)-1)^(1+1/m) ≧ 2(x+1)^(1+1/m)
> ←x + x^((m-1)/m) -1 > 2^(1/m)(x+1)
ここも最後の右辺は 2^(m/(m+1)) (x+1) じゃないかな だから
> ←x≧3, m≧100000000
が十分条件にならなくてまずい

とは言えパラメータうまくいじればうまくいけそうなのでちょっと助け船↓

c = 3/2 で固定する。x≧9, 0≦t≦1+2x^(1/2) の時
∂S/∂t = (3/2)((x+t)^(1/2) - (x-t)(1/2))
= (3/2)∫_[x-t,x+t] (1/2)X^(-1/2)dX
< (3/2)∫_[x-t,x+t] (1/2)(x-t)^(-1/2)dX
= 3t/(2(x-t)^(1/2))
≦ 3(1+2x^(1/2))/(2(x-1-2x^(1/2))^(1/2))
= 3(1+x^(-1/2))/(2(1-2x^(-1/2)-x^(-1))^(1/2))
≦ 21/(2√2).
ゆえに、0<K≦(2√2)/21 かつ x≧9 の時、t が 0 から [1+2x^(1/2)] まで動くことで
[KS(x,t)] は [2Kx^(1/2)] から [KS(x,2x^(1/2))] までの全ての整数をとる。
(ここまででだいたい半分)
0761132人目の素数さん
垢版 |
2022/04/04(月) 00:05:04.79ID:OA48AZ9m
>>760


> ← (3t)^m < m^m (x-t)^(m-1)
> ← t < x^((m-1)/m) & 3≦m
これ十分条件になってるかねえ

ここあかんね

3^m < m^m、t^m < (x-t)^(m-1)
が十分条件としてとれる
後者は
t^((m/(m-1))<x-t
⇔t+t^((m/(m-1)) < x
でこれは2t^((m/(m-1)) < xが十分でt<(x/2)^((m-1)/m)が十分
2(x+1)^(1+1/m) < ( x+ (x/2)^((m-1)/m) )^(1+1/m)
⇔2^(m/(m+1)) (x+1) < x + (x/2)^((m-1)/m)
のためには2^(m/(m+1))<2、2<(x/2)^((m-1)/m) が十分
m=10000000、x≧4で十分

大丈夫だと思うけど今出先のマクドで書くものが何もないからスマホの画面でしかできない
もうこれ以上こん詰めるのもしんどいのでこれでダメなら不正解でいいです
0764132人目の素数さん
垢版 |
2022/04/04(月) 05:57:03.45ID:Lk6rwb85
c = 5/4とする
s'(t) = 5/4 2t/((x+t)^(3/4) + .. + (x-t)^(3/4))
< 5/2 t / (4 (x-t)^(3/4)) < 1
if 5/2 t < 4(x-t)^(3/4)
iff (5/8)^(4/3) t^(4/3) + t < x
if t < x^(3/4) and x ≧ 22
∵ LHS mono. increase and
(5/8)^(4/3) x + x^(3/4) < x
if x^(3/4) < (1-0.535)x
iff x>(1-0.535)^(-4)
iff x > 21.388887117622...

2(x+1)^(5/4) < (x + x^3/4)^(5/4) + (x-x^3/4)^(5/4)
holds if x ≧ 63

(1+1/x^(1/4))^(5/4) + (1-1/x^(1/4))^(5/4) - 2*(1+1/x)^(5/4)


計算機解禁
c = 5/4とする
s'(t) = 5/4 2t/((x+t)^(3/4) + .. + (x-t)^(3/4))
< 5/2 t / (4 (x-t)^(3/4))
s'(t)<1には5/2 t < 4(x-t)^(3/4)が十分で、よって
(5/8)^(4/3) t^(4/3) + t < x‥@
で十分、さらにt < x^(3/4) and x ≧ 22で十分
( ∵ @左辺はtの関数として単調増大
よってt=x^(3/4)で成立すれば良い
つまり
(5/8)^(4/3) x + x^(3/4) < x
であればよく、(5/8)^(4/3)=0.5343..によりx^(3/4) < (1-0.535)xで十分でこれはx>21.388887117622の時成立 )
よって
2(x+1)^(5/4)
<(x+x^3/4)^(5/4)+ (x-x^3/4)^(5/4)
が成立すればよく、これは数値実験よりx≧63で成立
(note)
大先生の計算によりn≧63で整立するようである
ただしグラフは反転してその後0に収束するが0を下回ることはないようである
また[63^1.25 + 63^1.25] = 354であり、1000以下の自然数が表示可能であるのは計算機で直接確認出来る

https://ideone.com/UAuqsb

plot ((x + x^(3/4))/(x+1))^(5/4) +((x-x^(3/4))/(x+1))^(5/4), from 62 to 63

https://www.wolframalpha.com/input?i=plot+%28%28x+%2B+x%5E%283%2F4%29%29%2F%28x%2B1%29%29%5E%285%2F4%29+%2B%28%28x-x%5E%283%2F4%29%29%2F%28x%2B1%29%29%5E%285%2F4%29%2C+from+62+to+63&;lang=ja
0765132人目の素数さん
垢版 |
2022/04/04(月) 06:26:56.33ID:R6UikYkS
ありゃ、しまった
旧バンも貼ってしまった
後半だけ読んでください
0766132人目の素数さん
垢版 |
2022/04/04(月) 09:26:57.51ID:4CyWYgBl
>>764
計算機を信じればロジックは正しそうだしもう正解でいいかな
お疲れ様でした

ちなみに >>760 から続ければ計算機の力を借りずともギリ可能なレベル↓

x≧9 の時
S(x,2x^(1/2)) > 2S(x+1,0)
iff (x+2√x)^3 + (x-2√x)^3 + 2(x^2-4x)^(3/2) > 4(x+1)^3
iff (x^2-4x)^(3/2) > x^3-6x^2+6x+2
iff (x^2-4x)^3 > (x^3-6x^2+6x+2)^2 (∵x≧9の範囲では両辺とも正)
iff x^3-3x^2-6x-1 > 0
であり、最後の不等式は x≧9 で常に成り立つため、
x≧9 ならば [KS(x,2x^(1/2))] ≧ [2K(x+1)^(3/2)] が成り立つ。

以上より、0<K≦(2√2)/21 ならば [KS(9,6)]=[(15√15+3√3)K] 以上の整数は全て表せる。
よって、K=1/100 とおけば [(15√15+3√3)/100] = 0 以上の整数を
全て表せることが導けるので、示された。
0767132人目の素数さん
垢版 |
2022/04/04(月) 09:51:01.17ID:4CyWYgBl
ほんとはウェアリングの問題みたいに
[Kn^c]+[Km^c] で全ての自然数を表せるみたいなことをしたかったんだけど、
S(x,t):=[(x+t)^c]+[(x-t)^c] で t を動かした時の差がカクカクしてしまって
どうにも評価が難しかったんで断念し、妥協の結果生まれた問題でした
0768132人目の素数さん
垢版 |
2022/04/05(火) 02:25:04.01ID:XCG/mj0d
両方切り捨てでいけるか検証した
いけるようです

両方切り捨てだと最悪ケースでx+x^3/4,x-x^3/4の両方で1近く切り捨てられる可能性があるけどそれでも(x,x) 〜(x+x^(3/4),x-x^3/4)までのfの像が次の(x+1,x+1)〜のブロックにつながる十分条件として
2(x+1)^(5/4)
≦(x+x^3/4-1)^(5/4)+ (x-x^3/4-1)^(5/4)
がとれる
文字をx=1/t^4と置換して
2(1/t^4+1)^(5/4)
≦(1/t^4+1/t^3-1)^(5/4)+ (1/t^4-1/t^3-1)^(5/4)

2(t^4+1)^(5/4)
≦(1+t-t^4)^(5/4)+ (1-t-t^4)^(5/4)

2≦((1+t-t^4)/(1+t^4))^(5/4)+ ((1-t-t^4)/(1+t^4))^(5/4)

大先生によるとこれはt≦0.25では成立する
https://www.wolframalpha.com/input?i=plot+%28%281%2Bt-t%5E4%29%2F%281%2Bt%5E4%29%29%5E%285%2F4%29%2B+%28%281-t-t%5E4%29%2F%281%2Bt%5E4%29%29%5E%285%2F4%29++from+0.24+to+0.25&;lang=ja
すなわちx≧256では成立するので256までは計算機で確認すればいい
すなわち2×[256^5/4] = 2048まで表示できるか確認すれば良い
それは簡単

https://ideone.com/L4Oe7U
0769132人目の素数さん
垢版 |
2022/04/05(火) 10:14:16.82ID:z5XPBqrI
>>768
うん、ブロックが繋がらない心配はほとんどしてない。

問題なのは、そもそもブロック内の整数全体を本当にとるのかということ。
T(x,t) := [K(x+t)^(5/4)] + [K(x-t)^(5/4)] とすると、
>>764 の定義による S(x,t) との関係は KS-2<T≦KS となるが、
実際に KS(t)-T(t) が t ごとに [0,2) のどこに落ちるかを制御するのはほぼ不可能。
だから t を 0 からある値まで増やしていった時の T(x,t) の値が、例えば
100, 101, 102, 104, 105, …
のようにある値を抜かす可能性を排除できない。
これは K を十分小さく定めて解決できる問題でもないし、
他に良い方法が見つからない限りは、おそらく簡単には無理かなと

とは言えそれでも、定数 K>0, c>1 を適切にとれば
連続する二つの正の整数のうち少なくとも一方は [Kn^c]+[Km^c] と表せる、という
まあまあな結果は得られるけどもね
0770132人目の素数さん
垢版 |
2022/04/05(火) 11:45:21.56ID:mSuog164
僕的に面白いと思った今年のとある中学受験の問題だけどいいかな?

(2)2022にある整数をかけると、6けたの数□□□674となる。

※□が全て同じ数とは限りません!
0771132人目の素数さん
垢版 |
2022/04/05(火) 12:49:39.01ID:TPM7MueK
167?
一の位から地道に考えたけど、面白い解き方があるってことなのかな?
0773132人目の素数さん
垢版 |
2022/04/05(火) 15:51:06.74ID:8eouXTJJ
イヤイヤ、待て待て
隣接する2点で[ x^c ]の値の差は1,2に限られる
↓、→と進んで[x^c] + [ y^c ]の差が2以上にはならんのでは?
0774132人目の素数さん
垢版 |
2022/04/05(火) 20:10:34.72ID:mSuog164
>>771 「167」じゃありません 面白い解き方はありますよ

@まず2022=6×337から、337×a=□□□674(aは6の倍数)と考えます
0775132人目の素数さん
垢版 |
2022/04/05(火) 20:43:01.33ID:+wb3ZYke
>>770
ひたすら数えると

> calc(2022,6,674)
[1] 167

応用問題
1234にある整数をかけると、7けたの数□□□□888となる
ある数を列挙せよ。

> calc(1234,7,888)
[1] 132 632 1132 1632 2132 2632 3132 3632 4132 4632 5132 5632 6132 6632 7132 7632
0779132人目の素数さん
垢版 |
2022/04/05(火) 22:56:41.75ID:z5XPBqrI
>>773

tが1ずつ動く時の [(x+t)^c] + [(x-t)^c] の差は2か1(か0)だけど
[(x+t)^c] そのものはもっと大きく動くはず
t が(xに依存して)ある程度大きければだけど
0780132人目の素数さん
垢版 |
2022/04/06(水) 00:47:40.61ID:k7ogVh0h
>>779
せやね
書いた後そんなことないわとわかったけど忙しくて訂正できなかった
まぁ[ ]がバラけるとちょっとパッと処理する手が思いつかんね
0781132人目の素数さん
垢版 |
2022/04/06(水) 01:04:14.98ID:k7ogVh0h
>>690
辺BCに垂直な直線をlとして
BF ≧ BFのlへの射影の長さ
. = AF sinF + AB sinB
. = CD sinC + DE sin E
により
2BF ≧ AF sinF + AB sinB + CD sinC + DE sin E
. = AF sinC + AB sinE + CD sinC + DE sin E
同様にして
2DB ≧ CBsinE + CD sinA + EF sinE + FA sin C
2FD ≧ EDsinA + EF sinC + AB sinA + BC sin C
よって正弦定理より
4(Ra+Rc+Re)
= 2(BF/sinA + DB/sinC + FD/sinE)
≧ (AF sinC + AB sinE + CD sinC + DE sin E)/sinA
+(CB sinE + CD sinA + EF sinE + FA sin C)/sinC
+(ED sinA + EF sinC + AB sinA + BC sin C)/sinE
= AB(sinE/sinA+sinA/sinE)
+ BC(sinE/sinC+sinC/sinE)
+ CD(sinC/sinA + sinA/sinC)
+ DE(sinE/sinA + sinA/sinE)
+ EF(sinE/sinC + sinC/sinE)
+ FA(sinC/sinA + sinA/sinC)
≧ 2(AB+BC+CD+DE+EF+FA)
0784132人目の素数さん
垢版 |
2022/04/06(水) 19:44:40.39ID:0FbcWWVn
>>776 >>783 正解です
167はかけた数でしたね。>>774でのフォローが足りなくてすみません

▼面白い解き方とやら
@まず2022=6×337から、337×a=□□□674(aは6の倍数)と考えます
A下3桁が674(=337×2)なのでaの下3桁は「002」になります
Baの千の位の数を決めます。337をかけて3桁になる数は1と2のみで、
 1002は6の倍数ですが、2002は6の倍数でないのでa=1002
答えは1×337=337です。

つきあってくださりありがとうございました! ちなみに愛知県東海中学の問題でした
0785132人目の素数さん
垢版 |
2022/04/07(木) 01:47:02.08ID:QzEybmwD
  次を満たす 素数qが 存在することを示せ。

  pを素数とし、任意の整数nについて  n^p -p はqで割り切れない。


   背理法で示そうと思い、  n^p-p=kq とする。    −p=kq  のとき、任意の素数pが qのk倍になることはないから背理である

  これから先どうしていいか分からない
0786132人目の素数さん
垢版 |
2022/04/07(木) 04:55:20.30ID:Srfc4IAK
>>785

そもそも数学の命題になってない

> 次を満たす 素数qが 存在することを示せ。

つまり命題に含まれる束縛されてない自由数はqしか許されないのに

>   pを素数とし、任意の整数nについて  n^p -p はqで割り切れない。

この命題に現れるpは束縛されていない
任意のpについてなのかあるpについてなのか束縛しないと命題にならない
0787132人目の素数さん
垢版 |
2022/04/07(木) 14:58:27.13ID:gaf952Pi
   解けなかったらこうやって言い訳をする
0789132人目の素数さん
垢版 |
2022/04/07(木) 16:03:19.96ID:gaf952Pi
  qで 割り切れないという表現があったら数学では普通、割り切れると仮定して背理法を使う。  他の方法を取る場合はただのキチガイ

    意味が分からないと言っているお前がおかしい
0790132人目の素数さん
垢版 |
2022/04/07(木) 16:06:45.04ID:nVSJNays
>>789
もうお前何指摘されてるかすらわかってないやろ?
上の方とかみてたらお前の書き込めるレベルにはないのわからん?
0791132人目の素数さん
垢版 |
2022/04/07(木) 16:13:27.39ID:gaf952Pi
>>790


  上の方にあるのはごちゃごちゃしてる上に意味のないくそつまらない問題で全然面白くないし  俺が出した奴の方が100倍面白い
0793132人目の素数さん
垢版 |
2022/04/07(木) 16:44:56.36ID:gaf952Pi
>>792


  無理なのはお前だろ  
0794132人目の素数さん
垢版 |
2022/04/07(木) 17:25:07.22ID:000iI+fi
>>785
二文目のpを『任意の素数』か『ある素数』にしないと
次のどっちの意味にもとれるよってこと
(A) どんな素数pと整数nをとっても、n^p-p は q の倍数にならない
(B) 適切に素数pをとれば、どんな整数nをとっても n^p-p は q の倍数にならない

背理法の立て方から(A)かなと推測はできるけど、
解釈の幅が生じるような書き方は避けるようにしてね

んで肝心の問題の方だけど、これもしかして
具体的に何かの素数で成り立つことを確認してから出題してる訳ではない?
(p,n) = (q,0) とおけば n^p-p = -q だから q で割りきれるから主張は成り立たないんだけど

もし(A)じゃなくて(B)の解釈なら、
(p,q)=(2,5) とすれば確かに n^2-2 は5では割れないから、問題としては成り立つが
0795132人目の素数さん
垢版 |
2022/04/07(木) 17:30:18.56ID:gaf952Pi
>>794


    問題を解こうという姿勢が伺われない  この問題は数学界でもレベルの高いところにあり

  背理法で背理がいえれば安心する  問題は上のように仮定しておいて背理だというための発見ができないだけ
0796132人目の素数さん
垢版 |
2022/04/07(木) 17:37:48.76ID:gaf952Pi
  お前には数学的な知性が感じられない  数値を代入してちまちました小手先のことばかり


    数学的な驚異的知性とは、  たとえば幾何の問題を解いているときに、  どうやっても証明できないが、パスカルの定理が見えて解けたといった超人間的なこと

  それから基本的にこういう超難問は、1年考えているが解けない人ばかりなので、そう簡単に解ける方がおかしい
0797132人目の素数さん
垢版 |
2022/04/07(木) 17:48:06.62ID:35ZZok+H
お前極々基本的な数学的用語の使い方すらできてない
そもそも数学の教科書読んだことすらないんやろ
勉強などしなくても数学くらいできるという謎の自己過大評価
結果何にもできてない能無しやん
0798132人目の素数さん
垢版 |
2022/04/07(木) 17:53:05.88ID:gaf952Pi
     数学の超難問は普遍的な真理が美しい形でつながっている高次元の存在なのでクズには分からない

  お前の議論は白チャートレベル
0799132人目の素数さん
垢版 |
2022/04/07(木) 17:55:15.16ID:rsUdUvf9
自分が面白いと思ってもそれが一般的に成り立つとは限らないが、今回は成り立たないどころか文章がルール違反してて理解すらしてもらえない例だな。
ルールを守って楽しく数学!
0800132人目の素数さん
垢版 |
2022/04/07(木) 18:04:44.65ID:gaf952Pi
  n^p-p=kq   つまり  n^p = kq+p  が背理であることを証明するための考えられる手段として次のものがある

    フェルマーの小定理

    何か難しい Lemma を一つ考える

    今、我々は、これが背理であることを一生懸命示そうとしてるので、その最中にごちゃごちゃわけのわからんことを言う奴はアホ

 
0801132人目の素数さん
垢版 |
2022/04/07(木) 18:19:33.85ID:gaf952Pi
   フェルマーの小定理を使いたいところだがこの定理は、  n,pが互いに素という条件がある、使えるのか使えないのかよく分からない定理であり

  すぐ使えるのか分からない   フェルマーの小定理をつかえている問題も多いが、それは、n,pが互いに素と分かっている問題だけで、

    本問では互いに素ではないので、分からない
0802132人目の素数さん
垢版 |
2022/04/07(木) 18:23:29.10ID:gaf952Pi
  例えば、  2^(p-1) ≡ 1 ( mod p)  のような場合は、フェルマー小定理にひっついている互いに素の条件を無視できるため、応用できるが、

    n^p では、この定理では手におえない   
0803132人目の素数さん
垢版 |
2022/04/07(木) 18:26:26.99ID:gaf952Pi
  式は、  n^p ≡ p (mod q)  ということを言っているのは明らかである。

    しかし、これが背理であることをどうやって示すかというと非常に難しい
0804132人目の素数さん
垢版 |
2022/04/07(木) 18:53:04.56ID:gaf952Pi
  2^p ≡ p (mod q) の場合で、  フェルマーの小定理から  2^(p-1) ≡ 1 (mod p)

2^p = kq+p の一方で、  2^p = 2lp+2より、

   kq+p = lp+2 kq + (1-2l)p = 2 とやっても中々背理であることが示せないため、これは非常な難問である
0805132人目の素数さん
垢版 |
2022/04/07(木) 19:02:54.15ID:gaf952Pi
   ここで問題になっているのは、  n^p = kq+p が背理であることを示す定理を宇宙人がもっている

    しかし人類はその定理ないし補題を発見できない

   そのため、n^p = kq+pが背理であることを示せない   またおそらくその定理や補題は恐ろしい内容のものである

     そうしてみると、この問題の最終的な解決は、  俺の能力では無理である
0806132人目の素数さん
垢版 |
2022/04/07(木) 19:12:57.95ID:/Oz/8ydl
ガダガタ言ってスレ荒らすな能無し
まず最低限人に自分の考えてる事説明できるようにならんと話にならん
しかしお前はそれができるようになるようになる努力すらせんやろ
人間説明がそもそも学問に向いてないんだよ
消えろ
0807132人目の素数さん
垢版 |
2022/04/07(木) 19:20:33.71ID:gaf952Pi
  バカはお前だ  じゃあ突然、飛行機を作れと言ってお前は飛行機が作れるのか?

    解けないつってんだよ
0808132人目の素数さん
垢版 |
2022/04/07(木) 19:20:50.04ID:OF+4WgWR
そもそもここに投稿するべきかちゃんと読め

面白い数学の問題を紹介して解き合うスレです
質問スレではありません
『出題者が答えを知らない問題はお控えください』
統計学などはスレ違い、数学以外の話題は論外です
『荒らし、煽りはスルー推奨』
てことで誰か面白い問題頼む
0809132人目の素数さん
垢版 |
2022/04/07(木) 19:58:01.68ID:gaf952Pi
  だから  n^p = kq+p  が背理であることを言えばいいんだよ

   背理法というのは部分的に背理とかではないってのが分かってねえんじゃねえの
0810132人目の素数さん
垢版 |
2022/04/07(木) 20:33:38.39ID:gaf952Pi
  これが背理であることを言う方法はあるが非常に難しい、おわり
0811132人目の素数さん
垢版 |
2022/04/07(木) 20:50:02.89ID:OF+4WgWR
GOODBYE
0812132人目の素数さん
垢版 |
2022/04/08(金) 08:56:55.80ID:qZxB5P8f
Since (pp !1)=(p!1) = 1+p+p2 +¢ ¢ ¢+pp!1 ´ p+1 (mod p2), we can get at
least one prime divisor of (pp ! 1)=(p ! 1) which is not congruent to 1 modulo p2. Denote
such a prime divisor by q. This q is what we wanted. The proof is as follows. Assume that
there exists an integer n such that np ´ p (mod q). Then we have np2 ´ pp ´ 1 (mod q)
by the definition of q. On the other hand, from Fermat’s little theorem, nq!1 ´ 1 (mod q),
because q is a prime. Since p2 - q !1, we have (p2; q !1) j p, which leads to np ´ 1 (mod q).
Hence we have p ´ 1 (mod q). However, this implies 1 + p + ¢ ¢ ¢ + pp!1 ´ p (mod q). From
the definition of q, this leads to p ´ 0 (mod q), a contradiction.
0813132人目の素数さん
垢版 |
2022/04/08(金) 09:34:25.51ID:qZxB5P8f
元々問題の欠陥からして素数qは無限に存在することから、Q体上のkのガロア閉包にチェボタレフの密度定理を適用してこのようなqの集合は、1/pの密度を持つことがわかり、無限に多くのqを満たしているといえるが、素数qが無限に存在していることに着目しても、このように難しい定理によるしかない上、この解法は数学界の中でも非常に高性能な解法でありなかなか思いつかないことを複数組み合わせておりレベルが高い。高校程度の数学の問題の中でも最高峰に位置すると言ってよい。
0815132人目の素数さん
垢版 |
2022/04/08(金) 09:42:36.70ID:qZxB5P8f
 n^p = kq+pが背理であることを言おうとしてごちゃごちゃ矛盾をつつこうとしたら

>>812

のような  超ハイレベルな解法が存在してるのがばれて発狂してんのかカス
0816132人目の素数さん
垢版 |
2022/04/08(金) 09:50:47.41ID:wUCOOvCy
ハイレベルwwwww
英語もめちゃくちゃやのにwwwwwwwwww
バーカwwwwwwwwwwwwwe
0817132人目の素数さん
垢版 |
2022/04/08(金) 11:23:50.55ID:iekO0KWa
鉄板の塗装を請負いました何でも屋です。
単位はメートルで測って来ましたが、計算すると109平米になりました。そんなにあるんですか?平米単価2000円程で3万ぐらいの工事かなと先方に伝えましたが、この計算だと218,000円の工事になってしまいます。怒られそうです。
よろしくお願いします。全ての縦を足した数字に横の数字を足したのをかけました。
https://i.imgur.com/8RkWeje.jpg
0819132人目の素数さん
垢版 |
2022/04/08(金) 11:44:26.59ID:iekO0KWa
>>818
ありがとうございます!そちらに書き込みしてみます。すみませんでした。
0820132人目の素数さん
垢版 |
2022/04/08(金) 16:38:21.94ID:R86CoeQ4
Hnをn番目の調和数、ζ(x)をゼータ関数とする
ΣHn/n^2=2ζ(3) (和は、n=1からn=∞まで)
を示せ

自分のやり方が変にテクニカルになったので、どんな解き方があるか知りたい
0823132人目の素数さん
垢版 |
2022/04/08(金) 16:58:25.74ID:zEflv34Z
wikiの方は無限個あるかも未解決らしいから逆数和の方?

1 + (1+1/2)/4 + (1+1/2+1/3)/9+...

で合ってる?
収束遅いから計算機でもハッキリしない
0824132人目の素数さん
垢版 |
2022/04/08(金) 17:00:44.60ID:R86CoeQ4
そうです、逆数和の方です

Wolfram先生にも聞いたから結果だけはあってると思う
0826132人目の素数さん
垢版 |
2022/04/08(金) 17:31:22.44ID:zEflv34Z
H(n)の母関数がg(x) = -log(1-x)/(1-x)は冪展開すれば得られる
作用素IをI(f) = ∫[0,x]f(x)/xdxとして
I(I(g))にx=0,1ほり込んで引いたら答え
I(g) = Li_2(x) + 1/2log^2(x)
I(I(g)) = -Li_3(1 - x) + Li_3(x) + Li_2(1 - x) log(1 - x) + 1/2 log(x) log^2(1 - x) - ζ(3)
は部分分数分解と項別積分して
I(Li_s) = Li_(s+1)
から得られる
0827132人目の素数さん
垢版 |
2022/04/08(金) 17:35:55.57ID:ZwFeUwvN
Hn = Σ[k=1,n]1/k = Σ[k=1,∞]n/(k(n+k))
を代入
Σ[n=1,∞]Hn/n^2 = Σ[n=1,∞]Σ[k=1,∞]1/(kn(n+k))
ここで
f(x) = Σ[n=1,∞]Σ[k=1,∞]x^(n+k)/(kn(n+k))
と置くと
f'(x) = log^2(1-x)/x
より
Σ[n=1,∞]Hn/n^2
= ∫[0,1]log^2(1-x)/x dx (1-x=e^(-t)と置く)
= ∫[0,∞]t^2 e^(-t)/(1-e^(-t)) dt
= ∫[0,∞]t^2 Σ[n=1,∞]e^(-nt) dt
= Γ(3)Σ[n=1,∞]1/n^3
= 2ζ(3)
0828132人目の素数さん
垢版 |
2022/04/08(金) 17:56:21.80ID:qZxB5P8f
  (1)  こんな誰も読んでいない2ちゃんねるの板に

  (2)  何度も議論し尽くされた誰も興味がない数式を

  (3)  右から左に書いているだけ

    結論

      なんの効果もなし

   数学を使って社会を変えたかったらもっと面白くてレベルの高い問題を考えてバクサイにでも投下しろカス
0829132人目の素数さん
垢版 |
2022/04/08(金) 18:35:05.84ID:RueqiSUW
2ちゃんにいる迷惑な方々って、数学をやる以上社会を変えなくてはならないって思想を持ってることが多いですね
0830132人目の素数さん
垢版 |
2022/04/08(金) 19:22:51.65ID:XSnXqCUq
このスレで問題出してくれる人はみんな自分が勉強の道すがら見つけたちょっとした発見やらなんやらを工夫して問題形式にして解答も用意してみんなで楽しくやっている
そういう他人の努力に1ミリも尊敬の念を持てないゴミクズ
0831132人目の素数さん
垢版 |
2022/04/09(土) 00:32:06.31ID:mKqMw7EC
Σ[n=1,∞]Hn/n^2
=Σ[n=1,∞]Σ[k=1,n]1/(kn^2)
=Σ[k=1,∞]Σ[n=k,∞]1/(kn^2)
=ζ(3)+Σ[k=1,∞]Σ[n=k+1,∞]1/(kn^2)
=ζ(3)+Σ[k=1,∞]Σ[n=1,∞]1/(k(n+k)^2)
=ζ(3)+(1/2)Σ[k=1,∞]Σ[n=1,∞](1/(k(n+k)^2) + 1/(n(n+k)^2)
=ζ(3)+(1/2)Σ[k=1,∞]Σ[n=1,∞]1/(kn(n+k))
=ζ(3)+(1/2)Σ[k=1,∞]1/k^2Σ[n=1,∞]k/(n(n+k))
=ζ(3)+(1/2)Σ[k=1,∞]1/k^2Σ[n=1,∞](1/n - 1/(n+k))
=ζ(3)+(1/2)Σ[k=1,∞]1/k^2 Hk

(1/2)Σ[n=1,∞]Hn/n^2 = ζ(3)
0832132人目の素数さん
垢版 |
2022/04/09(土) 02:45:15.08ID:ORLs89zo
>>831
スンゲ
0835132人目の素数さん
垢版 |
2022/04/09(土) 11:44:27.97ID:Wt4GR3pO
(1) Σ[n=1〜∞] (−1)^n H_n / n は絶対収束はしないが収束はすることを示せ。

(2) Σ[n=1〜∞] (−1)^n H_n / n = (1/2)(log 2)^2 − (π^2) / 12 が成り立つことを示せ。
(2)は計算ミスしてる可能性もあるが、log 2 と π に関する何らかの和にはなるはず。
0837132人目の素数さん
垢版 |
2022/04/09(土) 14:18:08.67ID:mKqMw7EC
>>835
(1)
Hn/n = Σ[k=1,∞]1/(k(n+k))
はnに関して単調減少で
1/n≦Hn/n≦(1+log(n))/n→0 (n→∞)
の評価からHn/nの和は収束しないが交代級数は収束する

(2)
Σ[n=1,∞](-1)^n Hn/n
= Σ[n=1,∞]Σ[k=1,n](-1)^n/(kn)
= Σ[k=1,∞]Σ[n=k,∞](-1)^n/(kn)
= Σ[k=1,∞](-1)^k/k^2 + Σ[k=1,∞]Σ[n=k+1,∞](-1)^n/(kn)
= -(1-2/2^2)ζ(2) + Σ[k=1,∞]Σ[n=1,∞](-1)^(n+k)/(k(n+k))

この二項目の和は
d/dx Σ[k=1,∞]Σ[n=1,∞]x^(n+k)/(k(n+k))
= -log(1-x)/(1-x)
= d/dx (1/2)log^2(1-x)

以上まとめると
Σ[n=1,∞](-1)^n Hn/n = -(1/12)π^2 + (1/2)log^2(2)
0838132人目の素数さん
垢版 |
2022/04/09(土) 14:49:27.31ID:mKqMw7EC
>>837
(2)は以下の解答に差し替え(絶対収束しないので和の交換は注意が必要ですね)

Σ[n=1,∞]x^n Hn/n
= Σ[n=1,∞]Σ[k=1,n]x^n/(kn)
= Σ[k=1,∞]Σ[n=k,∞]x^n/(kn)
= Σ[k=1,∞]x^k/k^2 + Σ[k=1,∞]Σ[n=k+1,∞]x^n/(kn)
= Σ[k=1,∞]x^k/k^2 + Σ[k=1,∞]Σ[n=1,∞]x^(n+k)/(k(n+k))
= Σ[k=1,∞]x^k/k^2 + (1/2)Σ[k=1,∞]Σ[n=1,∞](x^(n+k)/(k(n+k)) + x^(n+k)/(n(n+k)))
= Σ[k=1,∞]x^k/k^2 + (1/2)Σ[k=1,∞]Σ[n=1,∞]x^(n+k)/(nk)
= Σ[k=1,∞]x^k/k^2 + (1/2)log^2(1-x)

これにx=-1を代入しアーベルの定理より以下が成り立つ
Σ[n=1,∞](-1)^n Hn/n = -(1/12)π^2 + (1/2)log^2(2)
0839132人目の素数さん
垢版 |
2022/04/09(土) 19:23:34.13ID:ddhyPW5c
テスト
0840132人目の素数さん
垢版 |
2022/04/10(日) 00:18:39.85ID:QMsUjQoO
まぁそもそもHnの母関数が-log(1-x)/(1-x)と割とクセのないいい関数になってるからそこまで難しくない
順に追って行けばa[n]=1/nの母関数が-log(1-n)でそれの累積をとる数列がHnだから母関数は1/(1-x)かけて-log(1-x)/(1-x)
そこから各項を1/n倍する作用素は∫〜dx/x
交代和は∫[0,-1]〜dx
定石で解けてしまう奴は定石で攻めた方がいい気はする
今回の難しさは不定積分がlog[n,x]みたいな奴が出てきて避けたい気持ちにはなってしまうけど、しかしコレは1/n^sの母関数で数列の話してると必然的に出てくる奴だから結局長い目で見れば避けて通るのは損にしかならない
0841132人目の素数さん
垢版 |
2022/04/10(日) 16:36:21.66ID:Bg/+fFe+
Geogebraで遊んでいてたまたま見つかったのですが
代数計算じゃない証明が知りたい。。
ーーーーーーーー
x軸上の6点 0<β<a<H(a,b)<b<αは
H(a,b)はa,bの調和平均、α,βはH(a,α)=b,H(b,β)=aを満たすものとする.
0,H(a,b)を直径の両端とする円をC_0
a,α  を直径の両端とする円をC_a
b,β  を直径の両端とする円をC_b
とするとき、C_0,C_a,C_bの共通の交点PとP'が存在することを示せ.
またPにおけるC_0,C_a,C_bの接線をT_0,T_a,T_bとすると
これらのなす角は各々60度であることを示せ.
0843132人目の素数さん
垢版 |
2022/04/10(日) 19:01:24.05ID:hOTzC2A9
複素数で考え一次変換 z -> 1/z を行うと、
円または直線は円または直線にうつり、角度は保存され、調和平均は相加平均になるから、
4点β, a, b, αは等間隔になって、……
0844132人目の素数さん
垢版 |
2022/04/10(日) 19:11:53.35ID:vFM4oqYB
そう等間隔になって
2b,b,0,a,2a
になったとするb<0<a
円は
(x-a)(x-2b)+y^2=0, (x-2a)(x-b)+y^2=0
になる、もう一つの円はx=0に移っている
確かに(0,√(-2ab))は3円の反転の像に入る
しかしなす角は60°になるとは限らない
0845132人目の素数さん
垢版 |
2022/04/11(月) 02:27:13.68ID:kvYPdUMj
>>678

1990年IMO中国大会の第3問

補題 (cf. Sierpinski gasket ) pを素数、vをp進付値とするとき
v(C(n,k))≧v(n)-v(k)
(ただしv(0)=∞とする)

(∵) C[n,k]に関するinduction
C[n,k]=1ならk=0,nだからRHS≦0で成立
C[n,k]<MでよいとしてC[n,k]=Mとする
p | nでなければRHS≦0より明らか
p | nとする
必要ならkをn-kに取り替えてk≦n/2としてよい(∵ RHS>0としてよいからv(n)>v(k)でありv(k) = v(n-k) )
lをk以下の最大のpの倍数とすれば容易に
v(C[n,k]) ≧ v(C[n,l]) 、C[n,k]≧C[n,l]、v(k)≦v(l)
である
このとき
v(C[n,k])
≧ v(C[n,l])
= v(C[n/p, l/p]) (cf. Sierpinski gasket )
≧ v(n) - v(l) ( ∵ Assump. of the ind. )
≧ v(n) - v(k). □
0846132人目の素数さん
垢版 |
2022/04/11(月) 02:27:21.47ID:kvYPdUMj
補題 

補題 pを素数、vをp進付値とする
整数a,bが共にpと互いに素である整数で、さらにp≧3, p | a-b又はp=2,4| a-bを満たすとき

v(a^n - b^n) = v(a-b) + v(n)

が成立する

(∵) p進整数環で考えればよいのでb=1としてよい
q = a -1とおく
a^n-1
= (1+q)^n -1
= Σ C[n,k]q^k
ここでk≧2のとき補題より
v( C[n,k]q^k )
≧v(n) - v(k) + v(q)k
≧v(n) - k/p + v(q)k
≧v(n) -2/p + 2v(q)
ここでp≧3、またはp=2かつv(q)≧2のいずれかを仮定しているから
v(n) -2/p + 2v(q)
> v(n) + v(q)
= v( C[n,1]q^1 )
である
よって
v( a^n - 1 ) = v(n) + v(q) = v(n) + v( a - b )
を得る□

定理の証明
(∵) nは奇数である
まず3|2^n - (-1)であることからvを3進付値とすれば

v(2^n - (-1)^n) = v(2-(-1)) + v(n) = 1 + v(n)

である
これがv(n^2) = 2v(n)以上だからv(n)=0,1である
故にnは3と互いに素であるかv(n)=1である

(i) nが3と互いに素であるとき
n≠1としてnの最小素因子pをとる
p | 2^n - (-1)^n, 2^(p-1) - (-1)^(p-1)
であるからp | 2^(n,p) - (-1)^(n,p)であるが(n,p)=1によりp|3となって矛盾する
故にこの場合の解はn=1しかない

(ii) nが3の倍数のとき
既に述べたことよりn=3m ((m,3)=1)とおける
m≠1としてmの最小素因子pをとる
p | 8^m - (-1)^m, 8^(p-1) - (-1)^(p-1)
であるからp | 8^(m,p) - (-1)^(m,p)であるが(m,p)=1によりp|9となって矛盾する
故にこの場合の解はn=3しかない
0847132人目の素数さん
垢版 |
2022/04/11(月) 02:28:29.71ID:kvYPdUMj
>>845

コピペミス

補題 (cf. Sierpinski gasket ) pを素数、vをp進付値とするとき
v(C(n,k))≧v(n)-v(k)
(ただしv(0)=∞とする)

(∵) C[n,k]に関するinduction
C[n,k]=1ならk=0,nだからRHS≦0で成立
C[n,k]<MでよいとしてC[n,k]=Mとする
p | nでなければRHS≦0より明らか
p | nとする
必要ならkをn-kに取り替えてk≦n/2としてよい(∵ RHS>0としてよいからv(n)>v(k)でありv(k) = v(n-k) )
lをk以下の最大のpの倍数とすれば容易に
v(C[n,k]) ≧ v(C[n,l]) 、C[n,k]≧C[n,l]、v(k)≦v(l)
である
このとき
v(C[n,k])
≧ v(C[n,l])
= v(C[n/p, l/p]) (cf. Sierpinski gasket )
≧ v(n) - v(l) ( ∵ Assump. of the ind. )
≧ v(n) - v(k). □
0848132人目の素数さん
垢版 |
2022/04/11(月) 14:05:47.03ID:CAsAh0+A
べき乗和婆^pでpが3以上の奇数の時婆^p=qを満たす素数qは存在するか
0849132人目の素数さん
垢版 |
2022/04/11(月) 14:28:34.28ID:jceT/sh2
  より簡単で初等的な問題に対する高度な解答を与えられない時点で  ウソでクソ認定
0852132人目の素数さん
垢版 |
2022/04/11(月) 18:40:25.35ID:wvgKjfCy
やっぱり存在しませんな
S(n) = Σ[1〜n]k^pとする
pが合成数でp=rs,r≧s>1のときS(r)の素因子dをとるときS(p)=S(r) (mod d)、S(p)>S(r)≧dよりS(p)は素数ではない
pが素数のときp|S(p)、S(p)>pよりS(p)は素数ではない
0855132人目の素数さん
垢版 |
2022/04/12(火) 03:44:49.52ID:1JuhDZ1v
>>848
Σk^p = 1^p + 2^p +...+ n^p とする

n=2のとき
Σk^p ≡ 1-1 ≡ 0 (mod 3), Σk^p>3
nが4の倍数のとき
Σk^p ≡ 1+0-1+0+...+1+0-1+0 ≡ 0 (mod 4), Σk^p>4

nが3の倍数のとき
Σk^p ≡ 1-1+0+...+1-1+0 ≡ 0 (mod 3), Σk^p>3
nが5の倍数のとき
Σk^p ≡ 1+2^p-2^p-1+0+...+1+2^p-2^p-1+0 ≡ 0 (mod 5), Σk^p>5
以下同様に
nがrの倍数(rは奇数)のとき
Σk^p ≡ 0 (mod r), Σk^p>r

したがってΣk^pは素数にはならない
0856132人目の素数さん
垢版 |
2022/04/12(火) 10:51:43.03ID:tVRcvwyE
https://i.imgur.com/VrkfquO.jpg

図のように欠円の半分に半円が内接している。欠円の半分を全体と捉えた場合、全体のうち半円の占める比率をZとする。Zの最小値はいくつか。
0857132人目の素数さん
垢版 |
2022/04/12(火) 16:45:06.54ID:ti8Koy61
>>842 >>844
Pの座標計算して円の中心からのベクトル二つの内積を計算したらcosの値が-1/2になったからあってるはず
汚い計算なんで数式処理ソフト使った方がよさげ。elegantな証明が欲しい
0858132人目の素数さん
垢版 |
2022/04/12(火) 18:29:53.06ID:ti8Koy61
>>857
H=2ab/(a+b)
a'=ab/(2a-b) ((2a-b)>0とする)

Solve({x^2+y^2-(a+a')*x+a*a =0,x^2+y^2-H*x=0}, {x, y})

Pの座標 {x = ((-a*a')/(H - a - a')), y = √((-H^2*a*a' + H*a^2*a' + H*a*a'^2 - a^2*a'^2)/(H - a - a'))}

((-a*a')/(H-a-a')-(H/2))*((-a*a'/(H-a-a'))-((a+a')/2))+(((-sqrt(-H^(2) a a'+H a^(2) a'+H a a'^(2)-a^(2) a'^(2)))/(H-a-a')))^2)
= (1/4) (H a + H a' - 2a a') = 内積 = (H/2)*((a'-a)/2)*cosθ

よって cosθ={(2*a*b/(a+b))( a + (a*b/(2a-b))) - 2a^2*b/(2*a-b)}/{( (2*a*b/(a+b)) (-a + (a*b/(2a-b))))}=-1/2
0859132人目の素数さん
垢版 |
2022/04/12(火) 19:47:13.70ID:ti8Koy61
>>858
> Solve({x^2+y^2-(a+a')*x+a*a =0,x^2+y^2-H*x=0}, {x, y})
a*a' の'が消えてた

最後の変数置換計算は便利なコマンドがあった
Expand(Substitute((H*(a+a')-2*a*a')/(H*((a'-a))),{H=2a*b/(a+b),a'=a*b/(2a-b)}))
=-1/2
0860イナ ◆/7jUdUKiSM
垢版 |
2022/04/12(火) 23:52:36.41ID:HuyjeHir
>>725
>>856
直感で解くと、
比=3π/(10π-6)
=0.37082173445……
4割弱かなぁって感じ。
cos75°=(√6-√2)/4とかを駆使した。
0861132人目の素数さん
垢版 |
2022/04/13(水) 18:44:05.67ID:uHr0HH6j
>>857
>>843 は読んだ?
a'=1/a, b'=1/bとおくと、
C_0は点(a'+b')/2を通る実軸に垂直な直線、
C_1は点a'を通る中心b'の円、
C_2は点b'を通る中心a'の円
にうつる。あとは図を描けば、角度が保存されることから明らか
0862132人目の素数さん
垢版 |
2022/04/13(水) 19:16:23.57ID:vF6aYcCM
反転したら互いの中心を通る半径の等しい二つの円とその2交点を結ぶ直線になるから
図を描けば60°になるのはすぐわかる
0863132人目の素数さん
垢版 |
2022/04/14(木) 07:39:57.16ID:psn5K0Pb
>>856
欠円の半分をx=√(1-y^2), cos(t)≦y≦1とすると
π/6≦t<π のとき半円の半径はr = (2 - √3 cos(t))/4
0<t<π/6 のとき解なし

Z = (π/8)(2 - √3 cos(t))^2/(2t - sin(2t)), π/6≦t<π
この最小値Zminは
2 sin(t) - √3 t = 0 の正の解をaとするときZmin = π(2 - √3 cos(a))/(8a)
a = 0.915582309673212459699344087310133383...
Zmin = 0.405149796624800679921346843672297349...
0864イナ ◆/7jUdUKiSM
垢版 |
2022/04/15(金) 10:47:01.64ID:7W5RFpGo
>>860
>>856
比Z=(半円πr^2/2)/[欠円πθ/2π-{h√(1-h^2)}/2]
=πr^2/{θ-h√(1-h^2)}
=πr^2/(θ-cosθsinθ)
Z'の分子=2πr(θ-cosθsinθ)-πr^2(1+sin^2θ-cos^2θ)=0
(θ-cosθsinθ)-rsin^2θ=0
0865イナ ◆/7jUdUKiSM
垢版 |
2022/04/15(金) 11:43:57.47ID:7W5RFpGo
>>864
>>856
rとθの関係式がわかれば文字一つ、
微分=0の式一つで解けるはず。
0867イナ ◆/7jUdUKiSM
垢版 |
2022/04/16(土) 23:40:33.01ID:8+uDJBlK
>>865
>>856
r=(19-cosθ√105)/16
r^2=(361-38cosθ√105+105cos^2θ)/256
Z=πr^2/(θ-sinθcosθ)
=π(361-38cosθ√105+105cos^2θ0)/256(θ-sinθcosθ)
Z'の分子=2πr(θ-sinθcosθ)-πr^2(1-cos^2θ+sin^2θ)
=2πr(θ-sinθcosθ)-2πr^2sin^2θ=0
θ-sinθcosθ-rsin^2θ=0
θ-sinθcosθ-19sin^2θ/16+(sin^2θcosθ√105)/16=0
16θ-16sinθcosθ-19sin^2θ+sin^2θcosθ√105=0
(今日はここまで)
0868 【吉】
垢版 |
2022/04/18(月) 00:53:06.25ID:DJDHjpWG
>>867
>>856
半円と欠円の半径の比を1:4とすると、
Z=3π/(16π-12√3)=0.31969128035……
1/4より少し長そう。
0869イナ ◆/7jUdUKiSM
垢版 |
2022/04/18(月) 12:02:53.62ID:VLxftDNP
(θ-cosθsinθ)-rsin^2θ=0
r=(θ-cosθsinθ)/sin^2θ
Z=πr^2/(θ-sinθcosθ)
=π(θ-cosθsinθ)/sin^4θ
Z'の分子=π(1-cos^2θ+sin^2θ)sin^4θ-π(θ-cosθsinθ)4sin^3θcosθ=0
2sin^2θsinθ-4(θ-cosθsinθ)cosθ=0
sin^3θ-2(θ-cosθsinθ)cosθ=0
sin^3θ-2θcosθ+2(1-sin^2θ)sinθ=0
2sinθ-2θcosθ=sin^3θ
(つづく)
0870イナ ◆/7jUdUKiSM
垢版 |
2022/04/18(月) 15:09:10.85ID:C5y02W6t
2sinθ-2θcosθ=sin^3θ
2θcosθ=2sinθ-sin^3θ
8θcosθ=8sinθ-4sin^3θ
sin3θ=3sinθ-4sin^3θと辺々引いて、
8θcosθ-sin3θ=5sinθ
π/3<θ<2π/5で、
8θcosθ-sin3θ-30)/15-5sinθ=0となるθを探す。
0872イナ ◆/7jUdUKiSM
垢版 |
2022/04/18(月) 18:47:28.06ID:1itbzodt
>>871
>>856
比Zの式を立て、
rをθで表しrを消去、
微分Z'の分子=0より、
8θcosθ-sin3θ-5sinθ=0
θ=57.0648791992273°のとき、
θ=0.99597002927
半円の半径rの式と比Zの式のメモを探し中。
あとは代入するだけ。
メモ探し。
0873イナ ◆/7jUdUKiSM
垢版 |
2022/04/18(月) 21:51:46.39ID:Rb257F1w
>>871
>>856
比Zの式を立て、
rをθで表しrを消去、
微分Z'の分子=0より、
8θcosθ-sin3θ-5sinθ=0
θ=57.0648791992273°のとき、
θ=0.99597002927
半円の半径rの式と比Zの式のメモを探し中。
あとは代入するだけ。
メモ探し。
0874イナ ◆/7jUdUKiSM
垢版 |
2022/04/19(火) 16:35:18.51ID:OPZDe8Uy
>>873
>>856
欠円の半径を1、欠円の半分の弧に対する中心角をθとすると、
欠円の半径から矢高を引いた長さはcosθ
弦長の半分はsinθ/2
半円の半径をrとすると、
Z=(πr^2/2)/(θ/2-sinθcosθ/2)
=πr^2/(θ-sinθcosθ)
rをθの式で表す。
作図して、y軸に沿って原点に向け楔形の直角三角形においてピタゴラスの定理より、
(1-r)^2=[cosθ+√{r^2-(sinθ/4)^2}]^2+(sinθ/4)^2
1-2r=cos^2θ+2cosθ√{r^2-(sinθ/4)^2}-(sinθ/4)^2+2(sinθ/4)^2
sin^2θ-2r=2cosθ√{r^2-(sinθ/4)^2}
sin^4θ-4rsin^2θ+4r^2=4cos^2θ{r^2-(sinθ/4)^2}
sin^4θ-4rsin^2θ+4r^2-4cos^2θr^2+sin^2θcos^2θ/4=0
sin^2θ-4r+4r^2+cos^2θ/4=0
16r^2-16r+3sin^2θ+1=0
r=[8-√{64-16(3sin^2θ+1)}]/16
=[8-4√{4-(3sin^2θ+1)}]/16
=(8-4cosθ√3)/16
=(2-cosθ√3)/4
rの値を代入し、
Z=πr^2/(θ-sinθcosθ)
=π(4-4cosθ√3+3cos^2θ)/16(θ-sinθcosθ)
Z'の分子=(4πsinθ√3-6πsinθcosθ)・16(θ-sinθcosθ)-π(4-4cosθ√3+3cos^2θ)・16(1+sin^2θ-cos^2θ)=0
32πsinθで辺々割ると、
(2√3-3cosθ)(θ-cosθsinθ)-(4-4cosθ√3+3cos^2θ)sinθ=0
2θ√3-2sinθcosθ√3-3θcosθ-4sinθ+4sinθcosθ√3=0
2θ√3+2sinθcosθ√3-3θcosθ-4sinθ=0
2(θ+sinθcosθ)√3=3θcosθ+4sinθ
12θ^2+24θsinθcosθ+12sin^2θcos^2θ=9θ^2cos^2θ+24θ sinθcosθ+16sin^2θ
12θ^2+12sin^2θ(1-sin^2θ)=9θ^2(1-sin^2θ)+16sin^2θ
3θ^2+9θ^2sin^2θ=4sin^2θ+12sin^4θ
3θ^2(1+3sin^2θ)=4sin^2θ(1+3sin^2θ)
1+3sin^2θ≠0
3θ^2=4sin^2θ
θ√3=2sinθ
2sinθ-θ√3=0
θ=0.91558230967322……
(θ=52.4590021411°)
θの値を代入し、
r={2-cos(0.91558230967322)√3}/4
=0.23615282276……
θの値を代入し、
∴Z=π(4-4cosθ√3+3cos^2θ)/16(θ-sinθcosθ)
=π[4-4cos(0.91558230967322)√3+3{cos(0.91558230967322)}^2]/[16{(0.91558230967322)-sin(0.91558230967322)cos(0.91558230967322)}]
=0.40514979662……
0875132人目の素数さん
垢版 |
2022/04/20(水) 15:49:46.68ID:8Gk4KCp+
△ABCの内部に点Pがあり
∠BAP=A1 ∠CAP=A2 ,A1+A2=A
∠CBP=B1 ∠ABP=B2 ,B1+B2=B
∠ACP=C1 ∠BCP=C2 ,C1+C2=C
とし、以下sin(A)≡s(A) などと略記する

問1
s(A1)s(B1)s(C1)=s(A2)s(B2)s(C2) を示せ.

問2
s(A)s(A1)s(A2)+s(B)s(B1)s(B2)+s(C)s(C1)s(C2)=s(A)s(B)s(C)
を満たすPの集合を求めよ
0877132人目の素数さん
垢版 |
2022/04/22(金) 09:16:24.27ID:R3cM+K0P
(1)は簡単だよね
両辺ともにAB×BC×CA/(sin∠APBsin∠BPCsin∠CPA)かければAP×BP×CPになるから
(2)の答えは円かなんかになる?
0878132人目の素数さん
垢版 |
2022/04/22(金) 10:59:59.74ID:IE/o+gH1
鈍角三角形のときは垂心が三角形内部にあって条件を満たすことは確認できる。
他は知らん
0879132人目の素数さん
垢版 |
2022/04/22(金) 14:41:44.26ID:LedtzuTW
垂心のとき成立する?
A(5,0),、
Bはy=2x+5とy=x/2の交点(10/3,-5/3)、
Cはy=-2x+5とy=-x/2の交点(-10/3,-5/3)、
とすれば
s(A1)=s(A2)=1/√5、s(A)=4/5
s(B1)=1/√5、s(B)=2/√5よりs(B2)=s(B-B1)=3/√5,
同様にして
s(C2)=1/√5, s(C1)=3/√5,s(C)=2/√5
よって
LHS = (1/√5)^2×4/5 + 2/√5 ×1/√5×3/√5 + 2/√5 ×1/√5×3/√5
RHS = 4/5 × 2/√5 × 2/√5
合わんけど
0880132人目の素数さん
垢版 |
2022/04/22(金) 16:14:32.75ID:IE/o+gH1
>s(B2)=s(B-B1)=3/√5,

3/5になるみたい。一般の場合やればいいやん。加法定理からすぐでる
0881132人目の素数さん
垢版 |
2022/04/22(金) 16:23:27.87ID:hXfOMgih
あ、失礼、
イヤまぁ一般に垂心で成り立つ事自体わかっても答えには繋がらんからいい
0882132人目の素数さん
垢版 |
2022/04/22(金) 20:49:33.11ID:IE/o+gH1
計算間違ってなければ直角二等辺三角形のときにきったない6次曲線になったから
一般の場合はもっと汚くなりそう。これ解く価値ないやろ
0883132人目の素数さん
垢版 |
2022/04/22(金) 21:57:42.09ID:7mtORxJ/
出題者らしき人のレスも全然ないから出題ミスだし逃げパターンかな
0884132人目の素数さん
垢版 |
2022/04/23(土) 01:33:14.35ID:x5HAzY7D
qを、|q|<1を満たす複素数とする
k,l∈{0,1}に対して指標付きθ関数(Auxiliary functions)θkl(z,t)を
θkl(z,t)
= Σ[n-k/2∈Z] q^n exp(πinl+2πinz)
で定める(ただしq=exp(πit))
また直交行列Aを
A =1/2[[1,1,1,1],[1,1,-1,-1],[1,-1,1,-1],[1,-1,-1,1]]
で定める
4次元複素列ベクトルx,yをy=Axととる時次の等式が成立する事を示せ

Σθ00(x[j]) - Σθ01(x[j]) - Σθ10(x[j]) + Σθ11(x[j])
=2Σθ00(y[j])

ただし和は全て1〜4を動くとし、θklの第二引数は話に関係ないので省略しているとする
0885132人目の素数さん
垢版 |
2022/04/23(土) 02:56:14.48ID:KftmWp8u
ノーヒントは難しすぎるのでヒント出しときます
まずほとんどヒネリはいりません
数ある指標付きθ関数の定義の中でこの問題を解くのに1番扱いやすい定義を問題文中では採用してます
定義に従って左辺を整理していきます
キーは行列Aが引き起こす変換R^4→R^4のZ^4の像がどうなるかです
結論は半整数の四つ組(Z/2)^4の部分集合
{ (mi)∈(Z/2)^4 | Σmi∈2Z }
になります
これをどう生かすかがミソ
0886132人目の素数さん
垢版 |
2022/04/23(土) 09:55:52.36ID:/oUdiAAh
>>884
訂正

> k,l∈{0,1}に対して指標付きθ関数(Auxiliary functions)θkl(z,t)を
> θkl(z,t)
> = Σ[n-k/2∈Z] q^(n^2) exp(πinl+2πinz)
> で定める
でした
qの肩のところが訂正されてます
0887132人目の素数さん
垢版 |
2022/04/24(日) 01:52:19.48ID:R4c/Y0ml
a,b,c>0 a+b+c≦3/2
f= √ (a^2+(1/b^2))+ √ (b^2+(1/c^2))+ √ (c^2+(1/a^2))とする時、fの最小値を求めよ
0888132人目の素数さん
垢版 |
2022/04/24(日) 05:18:10.17ID:3EjJSrYV
>>887
√((px+q)^2 + 1/x^2) は凸関数である
実際適当にスケール変換してp=1としてよくその場合2階微分は
((3 q^2 x^2 + 8 q x^3 + 6 x^4 + 2) abs(x))/(x^4 (x^2 (q + x)^2 + 1)^(3/2))
となり正値をとる
よってf(a,b,c)は凸関数であり端点に向かう時→∞だから領域内で唯一の最小値をとる点を持つ
対称性からa=b=cとして良い
この時
f(a,b,c) = 3 √ (a^2+(1/a^2))
であるからこの最小値が求める最小値てわあり3√2である
0889132人目の素数さん
垢版 |
2022/04/24(日) 20:58:47.69ID:rL/qYR9J
>>888
問題文には制約条件a+b+c≦3/2があるので最小値は(3/2)√17だと思う

>√((px+q)^2 + 1/x^2) は凸関数である
>...
>よってf(a,b,c)は凸関数であり
の論理がよくわからないのでできれば説明お願いします

まあ相加相乗平均2回でほぼ解けてしまう問題だけど...
0891132人目の素数さん
垢版 |
2022/04/24(日) 21:07:22.01ID:IPSA9SFS
>>889
f(x,y,z)が凸関数⇔∀pqrsuv f(px+q,rx+s,ux+v)が凸関数
つまり任意の直線上で凸関数になっていれば良い
本問の関数は√××3つの和なので各々が凸関数なら凸関数
√××の××部分の変数に一次式入れると全部√((一次式)^2+1/(一次式)^2)のタイプになるのでコレがいつでも凸関数が言えれば良い
必要なら平行移動して1/(一次式)^2は1/t^2と思っていいし定数括り出して√((t+a)^2+1/t^2)のタイプに限定して良い
→大先生→2回微分>0→凸
0892132人目の素数さん
垢版 |
2022/04/24(日) 21:21:32.72ID:rL/qYR9J
>>891
ありがとう、理解できた。
非対称の解が出る問題だともっと面白いと思う。
ついでに相加相乗平均を使う解答を貼っておきます。

f/3 ≧ ((a^2+1/b^2)(b^2+1/c^2)(c^2+1/a^2))^(1/6)
= (a^2+b^2+c^2 + 1/a^2+1/b^2+1/c^2 + (abc)^2 + 1/(abc)^2)^(1/6)
≧ (3(abc)^(2/3) + 3/(abc)^(2/3) + (abc)^2 + 1/(abc)^2)^(1/6)
最初の不等号で等号成立 ⇔ a^2+1/b^2 = b^2+1/c^2 = c^2+1/a^2 ⇔ a=b=c
2つ目の不等号の等号成立条件もこれと同じ

g(x) = 3x^2 + 3/x^2 + x^6 + 1/x^6 と置く
g'(x) = -6(1-x^4)(1+x^4)^2/x^7 < 0 (0<x≦1/2)
ゆえに
f ≧ 3(g((abc)^(1/3)))^(1/6)
≧ 3(g((a+b+c)/3))^(1/6)
≧ 3(g(1/2))^(1/6)
= (3/2)√17
最後の不等号はa+b+c=3/2のときに等号が成り立つ
0893132人目の素数さん
垢版 |
2022/04/25(月) 00:37:45.79ID:7ntPebR7
>>884 の問題
>Σθ00(x[j]) - Σθ01(x[j]) - Σθ10(x[j]) + Σθ11(x[j])
>=2Σθ00(y[j])
だけどwolfram先生と数値が合わない

もしかして和でなくて積
Πθ00(x[j]) + Πθ01(x[j]) - Πθ10(x[j]) + Πθ11(x[j])
=2Πθ00(y[j])
の誤り?
0894132人目の素数さん
垢版 |
2022/04/25(月) 02:24:08.92ID:ahoe1rg9
>>893
失礼しました
積です
Πθ00(x[j]) + Πθ01(x[j]) - Πθ10(x[j]) + Πθ11(x[j])
=2Πθ00(y[j])
が正解
私の読んだ教科書ではθ間形式という名前が付いてました
0895132人目の素数さん
垢版 |
2022/04/25(月) 02:35:23.90ID:ahoe1rg9
あ、まだ間違ってる

Πθ00(x[j]) - Πθ01(x[j]) - Πθ10(x[j]) + Πθ11(x[j])
=2Πθ00(y[j])

です
私の読んだ教科書ではこの公式と熱方程式と呼ばれる方程式を足がかりに数あるθ関数の公式を次々と導出していくというスタイルをとってて中々気分がいい
どうもMumfordのスタイルのようです
ただこんなの思いつかねーのが欠点
0896132人目の素数さん
垢版 |
2022/04/25(月) 04:08:04.42ID:Klt3+Atn
>>895

また訂正orz

Πθ00(x[j]) + Πθ01(x[j]) + Πθ10(x[j]) + Πθ11(x[j]) = 2Πθ00(y[j])

全部プラスです
スマヌ
0897132人目の素数さん
垢版 |
2022/04/26(火) 00:33:57.51ID:NvtQufOW
代数関数の値域の上限、下限は係数体の代数的拡大の元
決定するためのアルゴリズムも見つかってる
0899132人目の素数さん
垢版 |
2022/04/27(水) 16:44:42.02ID:/TaoCG1j
平面上に円とその中心が与えられている時、定規のみを用いて
円の面積を(1)3(2)4(3)5等分する方法を示せ。
(予想解答での分割形は扇型ですが、それ以外の解法もあるかもしれないので、
分割形は限定しません)
どこまで等分可能な数を広げられるか挑戦している最中です。
0900132人目の素数さん
垢版 |
2022/04/27(水) 16:45:55.71ID:/TaoCG1j
(1)〜(3)は小問番号です
わかりにくかったらすいません
0901132人目の素数さん
垢版 |
2022/04/28(木) 01:12:28.79ID:jJRm8+pR
問題の設定で平面に複素座標が与えられているとし作図されている円は|z|=1であり0,1は作図されている定点とする
作図可能な点の全体をSとする

補題 -1,±i∈S
(∵) 実軸は作図可能だからその単位円との2交点1,-1は作図可能
実軸のx>1の部分に任意にaをとる
aを通る2接線が作図可能なのは既出なので2接点をu,vとする
u,0を通る直線の交点を作図して-uを作図できる
-uにおける接線とvにおける接線の作図方は既出
この交点は虚軸上にあるので結局虚軸も策す可能
よって±iも作図可能□

補題 im(z) = ±1, ±1/2, re(z)=±1, ±1/2は作図可能
(∵)±1,±iが作図可能だからこれらの点における接線であるim(z) = ±1,re(z)=±1, re(z)=±1は作図可能
よって±1±iも作図可能であり±1/2±1/2iも作図可能□

補題 a∈S∩R → ai,±a±i, ±ai ±1∈S
(∵) aiとa+iについてだけ示せば十分である
aiはaとiを結ぶ直線とim(z)=1/2の交点α=(a+i)/2をまず作図しαとaを結ぶ直前と虚軸の交点がaiであることからわかる
次にa+iは今作図したαと0を結ぶ直線とim(z)=1との交点であるから作図可能である□
0902132人目の素数さん
垢版 |
2022/04/28(木) 01:12:48.85ID:jJRm8+pR
補題 a,b∈S∩R→a+b∈S
(∵) 補題より-b+i,(a+i)/2の3点が作図可能であり、-b+iと(a+i)/2を結ぶ直線と実軸の交点がa+bである□

補題 a,b∈S∩R→ab∈S
(∵) 補題よりa+i, 1+bi,-1+biが作図可能であり
原点とa+iを結ぶ直線と1+bi,-1+biを結ぶ直線の交点ab+biが作図可能である
同様にab-biみ作図可能だからこれら2点を結ぶ直線と実軸の交点であるabも作図可能である□

補題 a∈S∩R,a≠0→1/a∈S
(∵) 1+aiと0を結ぶ直線とim(z)=iの交点1/a+iは作図可能である
同様に1/a-iも作図可能だから主張を得る□

補題 a∈S∩R→√a∈S
(∵) 補題により0<a<1の場合に示せば良い
c=(1-a)/(1+a)は作図可能である
re(z)=cと単位円の交点のひとつをc+si(s>0)とする
c+siでの接線とre(z)=1の交点をαとすればα=1+√aiである□

定理 定規とコンパスによる作図で作図可能な点は全て作図可能
0903132人目の素数さん
垢版 |
2022/04/28(木) 05:50:26.52ID:UXvWRkiH
トランプをゆがませた時にできるU字型の曲線の一般式を求めよ
0904132人目の素数さん
垢版 |
2022/04/28(木) 06:20:19.30ID:7u46cckq
>>667
Wikiの文献1。
宇宙際タイヒミュラー理論(うちゅうさいタイヒミュラーりろん、英語: Inter-Universal Teichmüller Theory、略称: IUT)は、数学者望月新一によって開発された、数論におけるさまざまな予想、特にABC予想を解く要件[1]の考察により、遠アーベル幾何などを拡大した圏の宇宙際 (IU) 幾何を構想した数学理論である[2]。

>考察で得た「ABC予想を解く要件」と書かれている。
2008年の文献1だから新論文の後付けはでなくれ、着想のフロー図は、
スタートの「ABC予想を解きたい」から始まるが、
下波線で「重要なポイント」となっており、「通常の集合論を拡大する必要がある」として、ABCを解く解決策で導入した部分で肝であることが分かる。

Nスぺが「これまでにない数学への進化」が含まれたところで論議が起きており、この新しい数学の解説が求められるに対し、ANDとORの論文が回答であるとした。

だから分かり易くなるようにアブストラクトを付けたのではないのかな。
どこが乗法の情報で、どこが加法の情報かを書き加えて、
新たな論理展開はANDで担保しているとの説明(ORを使うのは誤り)との論旨になった。
0905sage
垢版 |
2022/04/28(木) 06:23:38.48ID:7u46cckq
あ、スレ違いの記入をしました。誠に失礼しました。
0906132人目の素数さん
垢版 |
2022/04/28(木) 09:11:28.27ID:wGgxQj5y
>aを通る2接線が作図可能なのは既出なので2接点をu,vとする

コレは覚え間違いで出てないみたいだから以下に修正

実軸と公差る任意の接線をとりその交点をαとする
この時はαを通る残りのもうひとつは作図できる
何故ならはαを通る別の円と2交点β、γを持つ直線をとりβ、γでの接線の交点をδとすればαとδを通る直線がaの極線となる
0907132人目の素数さん
垢版 |
2022/04/28(木) 09:43:58.43ID:DuvHBHDv
イヤ、記憶違い出なかった
円に内接する四角形ABCDにおいて直線ABと直線CDの交点をX、直線BCと直線ADの交点をYとすればYはXの極線上が既出
よってXの極線を作図するには上の条件満たすセットA'B'C'D'X'Y'でX=X'、Y≠Y'となるものを用意すれば直線YY'が極線
0908132人目の素数さん
垢版 |
2022/04/28(木) 20:01:08.53ID:B/I+FSJy
>>901
>aiはaとiを結ぶ直線とim(z)=1/2の交点α=(a+i)/2をまず作図しαとaを結ぶ直前と虚軸の交点がaiである
ここaとiを結ぶ直線上の点とaを結んでない?
0909132人目の素数さん
垢版 |
2022/04/28(木) 20:29:21.45ID:J2tXLzft
>>908
失礼、訂正

補題 a∈S∩R → ai,±a±i, ±ai ±1∈S
(∵) aiとa+iについてだけ示せば十分である
aiはaとiを結ぶ直線とim(z)=re(z)の交点αをまず作図しαと1を結ぶ直前と虚軸の交点がaiであることからわかる
次にa+iはaとiを結ぶ直線とim(z)=1/2の交点βを作図しβと0を結ぶ直線とim(z)=1との交点がa+iであるから作図可能である□
0911132人目の素数さん
垢版 |
2022/04/29(金) 17:55:09.10ID:zMHGWc8T
つまらないかもしれん

Q.直線ℓ上に半径が5の大円Aと、半径が2の中円Bがあり、1点で接している。
また小円Cは大円Aとも中円Bとも接していて、直線ℓ上にある。
このとき、小円Cの半径を求めなさい。(算額の改題)
0912132人目の素数さん
垢版 |
2022/04/29(金) 17:59:17.25ID:zMHGWc8T

0913132人目の素数さん
垢版 |
2022/04/29(金) 19:23:52.67ID:5d2yIH3X
デカルトの円定理より
(1/5+1/2+1/x)^2 = 2*(1/25+1/4+1/x^2)の解
x = 70/9 ± (20 sqrt(10))/9
コレの小さい方
0914イナ ◆/7jUdUKiSM
垢版 |
2022/04/29(金) 21:18:35.14ID:Vuxs6cd9
>>910
>>911
小円Cの半径をrとすると、
ピタゴラスの定理より、
√{(5+r)^2+(5-r)^2}+√{(2+r)^2+(2-r)^2}=√{(5+2)^2+(5-2)^2}
√(20r)+√(8r)=√40
√(5r)+√(2r)=√10
5r+2r√10+2r=10
(7+2√10)r=10
9r=10(7-2√10)
∴r=(70-20√10)/9
=0.75049408851……
0915132人目の素数さん
垢版 |
2022/04/30(土) 02:06:36.82ID:u3jDJHUm
円Γ上に相異なる4点ABCDがこの順に並んでいる
直線ABと直線CDが交点Pわもち、直線ACと直線BDの交点をQとする
Γ上の相異なる2点S,Tの接線がPを通るとする
この時直線STはQを通る事を示せ
0917132人目の素数さん
垢版 |
2022/04/30(土) 11:54:51.46ID:tHEpg0dM
>>912 半径を平方数にするの忘れてた...スマン
0918132人目の素数さん
垢版 |
2022/05/01(日) 00:38:22.29ID:K4NwIJy0
https://imgur.com/a/r9gzuXC
㋐〜㋤に当てはまる0〜9の数字または-を入れよ。
0919132人目の素数さん
垢版 |
2022/05/01(日) 01:01:34.56ID:avyYxJTz
>>912
Aの半径をa、Cの半径をcとすると、中心間の距離がa+c、中心の縦の差がa-cだから
中心の横の差の平方は (a+c)^2-(a-c)^2=2a*2c
同様にBとCの中心の横の差の平方は2b*2c、AとBの中心の横の差の平方が2a*2b
先の二つの横の差の和2√c(√a+√b)が後の横の差2√abと一致し c=ab/(√a+√b)^2
0920132人目の素数さん
垢版 |
2022/05/02(月) 11:03:50.93ID:voEke4JC
中心O半径rの円Cに対して平面上の点A≠Oに対して方程式l:{P|OA→・OP→=r^2}によって定められる直線lを対応させる
この対応でOと異なる点の全体とOを通らない直線の全体の間の一対一に対応が得られる
点Aと直線lが対応付けられているときlをAの極線(poler)と呼びAをlの極(pole)と呼ぶ

半径rの円と2点P,Qについて以下は同値である事を示せ
(1) PはQの極線上
(3) OP²+OQ²=PQ²+2r²
0921132人目の素数さん
垢版 |
2022/05/02(月) 18:35:11.79ID:/hdzuoX2
r^2=OP(→)・OQ(→) (ベクトルOPとOQの内積)の方がシンプルやん.

Q(a,b) , P(x,y)とすると極線がax+by=r^2ってなる
0923132人目の素数さん
垢版 |
2022/05/03(火) 10:05:48.91ID:bbVNCNxa
>>921
書き忘れましたけどそれ正解です
そして接点S,TはP(u,v)の極線の方程式ux+vy=r²を満たすことから(∵接線の方程式の公式)直線STはPの極線だと解ります
つまり915は915の設定の元で
「QはPの極線上にある事を示せ」
と同じとわかります
0924132人目の素数さん
垢版 |
2022/05/04(水) 19:57:40.38ID:Ad86gHvU
C[7,k]をk=0から7まで変化させた時の値を順に書くと、{1,7,21,35,35,21,7,1}
これを3で割った時の余りは {1,1,0,2,2,0,1,1} で、0の数と2の数は共に2個で一致します。

このように、kが0からnまで変化する時、
C[n,k]≡0 (mod 3)となる k の数と、
C[n,k]≡2 (mod 3)となる k の数が一致するのは、どのような場合か予想し、それを証明せよ。
ヒント:
このような性質を持つ n は、(恐らく意外に少なく) 100 以下では、例に挙げた 7 を除くと、後二つしかなく、
10000まで拡大してもさらに4つを加えるだけ。この性質を持つ n を列挙すれば、予想は簡単。
0925132人目の素数さん
垢版 |
2022/05/05(木) 18:21:33.55ID:PlaGatr+
>>924
aₙ, bₙ,cₙを0≦n<3ᵉに対してC[n,k]の中の≡0,≡1,≡2 (mod 3)となるkの数を与える列とする
書き出せば
a: 0 0 0 2 1 0 4 2 0‥
b: 1 2 2 2 4 4 2 4 5..
c: 0 0 1 0 0 2 1 2 4..
である、ただし項は第0項から数えるとする
aₙ+bₙ+cₙ=n+1であるから条件はbₙ+2cₙ=n+1と同値である
各列の初項から3ᵉ項を取り出して得られる部分列をaᵉ, bᵉ, cᵉとする
パスカルの三角形から漸化式
bᵉₙ₊₁=bᵉₙ⊕2bᵉₙ⊕2bᵉₙ+cᵉₙ
cᵉₙ₊₁=cᵉₙ⊕2cᵉₙ⊕2cᵉₙ+bᵉₙ
が得られる、ただし⊕は2つの有限列を繋げて得られる有限列を与える二項演算子とする
漸化式を用いて以下の主張が帰納的に示される

主張 0≦n<3ᵉₙ、n = k×3ᵉ + l, (k = 1,2, 0≦l<3ᵉ⁻¹) とする以下が成立する
bₙ+cₙ = 3ᵉ (n = 3ᵉ - 1)
. ≦ 2×3ᵉ⁻¹ (n ≠ 3ᵉ - 1)
bₙ+2cₙ = (3n+1)/2 ( n = 2×3ᵉ⁻¹- 1)
. = (3n+2)/2 ( n = 3ᵉ - 1)
. = n+1 ( n = 3ᵉ - 2)
. < n+1 ( n = 2×3ᵉ⁻¹- 1,3ᵉ - 1,3ᵉ - 2)

主張により求めるnの条件はn+2が3のベキとなる時である
0926132人目の素数さん
垢版 |
2022/05/05(木) 19:24:08.79ID:GQOABAfW
>>925
例外の選び方が甘かった
bₙ+cₙの方は合ってる
bₙ+2cₙ>n+1となる例外はn = 2×3ᵉ⁻¹- 1, 3ᵉ- 4, 3ᵉ- 1の3つあるわ
0927132人目の素数さん
垢版 |
2022/05/05(木) 22:07:08.22ID:oKb3+ULH
解答ありがとうございます。

「n+2が3の冪」が求める条件であることは正解です。

>> パスカルの三角形から漸化式
>> bᵉₙ₊₁=bᵉₙ⊕2bᵉₙ⊕2bᵉₙ+cᵉₙ
>> cᵉₙ₊₁=cᵉₙ⊕2cᵉₙ⊕2cᵉₙ+bᵉₙ

については、式の意図が分かりませんでした。
もう少し詳しく説明していただけないでしょうか?
0928132人目の素数さん
垢版 |
2022/05/05(木) 23:21:15.34ID:DgN/EzuL
>>927
いわゆるシェルピンスキーのガスケットです
パスカル三角形を3で割ったあまりは

11
121
1001
11011
121121
1002001
11022011
121212121
のようにある種のフラクタル図形の構造が入ります
上の三角形の中に小さい三角形が9個

△△
△△△
の形で出現して△の外部には0しか来ません(次の段から△の中にも0が出ますが△の外側に1、2が漏れ出ることはありません)
よって1の数bₙと2の数cₙはこの△の中だけ見ればよいとわかります
大きい三角(次の項)の1段目は小さい三角そのもの、すなわちb₁₊₁の出だし1/3はbₙそのものが来ます
次の段には△2つ分、すなわち2bₙがきます
最後の段の両端にくる三角形は△そのものですが真ん中の△は頂点が2になるので△の1と2を入れ替えたものになりそこに現れる1の数はcₙとなり3段目は2bₙ+cₙとなります
そこから
bₙ₊₁=bₙ⊕2bₙ⊕2bₙ+cₙ
がbₙの満たすべき漸化式とわかります
cₙも同様です
0929132人目の素数さん
垢版 |
2022/05/06(金) 00:01:46.44ID:+17u7Xrj
>> bₙ₊₁=bₙ⊕2bₙ⊕2bₙ+cₙ
の左辺は、パスカルの三角形 n+1 段目に書かれている 1 の「数」
右辺の最後の項 c_n は n段目に書かれている 2 の「数」 はよく分かります。
それ以外の部分、
>> bₙ⊕2bₙ⊕2bₙ
は、「有限列を繋いでできた有限列」と説明があるのですが、
「数」=「有限列」+「数」
となっている様に見え、意味がよく分からないのです。再度ご説明を。
0930132人目の素数さん
垢版 |
2022/05/06(金) 00:36:35.24ID:+17u7Xrj
訂正
×:の左辺は、パスカルの三角形 n+1 段目に書かれている 1 の「数」
○:の左辺は、パスカルの三角形の3による剰余版の n+1 段目に書かれている 1 の「数」
0931132人目の素数さん
垢版 |
2022/05/06(金) 00:59:18.60ID:4ohmBcow
>>929
上つきと下付き間違えました

bᵉ⁺¹=bᵉ⊕2bᵉ⊕2bᵉ+cᵉ
cᵉ⁺¹=cᵉ⊕2cᵉ⊕2cᵉ+bᵉ

です
例えばe=1の場合からe=2の場合を計算するのは
b¹=(122)、c¹=(001)
なのでb²を計算するにはまず長さ3の列
b¹=(122)
2b¹=(244)
2b¹+c¹=(245)
なので
b²=(122244245)
とわかります
同様にして
b²=(001002124)
となります
実際9段目までパスカル三角形書いてみると

11
121
1001
11011
121121
1002001
11022011
121212121
となってb,cが正しく1の数、2の数を表している事がわかります
0933132人目の素数さん
垢版 |
2022/05/06(金) 07:05:15.50ID:+17u7Xrj
丁寧な説明、ありがとうございます。

なるほど、式の意味が分かりました。

シェルピンスキーのガスケット の構造の再帰性を
bᵉ⁺¹=bᵉ⊕2bᵉ⊕2bᵉ+cᵉ
の様な形で取り入れたんですね。驚きました。

ほとんど正解だと思います。

>>926でいくつか「例外」として書かれているものがありますが、他にもあると思います。
それら「例外」の共通点を考えると、別のアプローチが見えてくるかもしれません。
0934132人目の素数さん
垢版 |
2022/05/06(金) 09:10:27.71ID:dgVmC0G+
>>933
ありますね
計算機で探してみました
最初から使えばよかった
一部抜粋(3進表示4桁のやつと5桁のやつ)

(44,"1122",36,"1100",52,"1221")
(50,"1212",36,"1100",52,"1221")
(52,"1221",36,"1100",52,"1221")
(53,"1222",54,"2000",80,"2222")
(71,"2122",54,"2000",80,"2222")
(77,"2212",54,"2000",80,"2222")
(79,"2221",54,"2000",80,"2222")
(80,"2222",81,"10000",121,"11111")

(134,"11222",108,"11000",160,"12221")
(152,"12122",108,"11000",160,"12221")
(158,"12212",108,"11000",160,"12221")
(159,"12221",108,"11000",160,"12221")
(161,"12222",162,"20000",242,"22222")
(215,"21222",162,"20000",242,"22222")
(233,"22122",162,"20000",242,"22222")
(239,"22212",162,"20000",242,"22222")
(241,"22221",162,"20000",242,"22222")
(242,"22222",243,"100000",364,"111111")
0935132人目の素数さん
垢版 |
2022/05/06(金) 09:10:35.15ID:dgVmC0G+
左がn、真ん中がbₙ+cₙ、右が件のbₙ+2cₙで“nの3進数表示に0はなく最高位以外の1の数が高々1個”の場合
これ以外ではbₙ+2cₙ<n+1となる事は漸化式から出ます
すなわちdₙ=bₙ+cₙ、eₙ=bₙ+2cₙとでもおいて漸化式としてn が3進数でe桁、最高位がk、残りの桁がlでn = k×3ᵉ⁻¹ + l (k = 1,2, 0 ≦ l≦ 3ᵉ⁻¹ -1 ) とするとき

k=1のとき
dₙ = 2dₗ、eₙ=2eₗ
k=2のとき
dₙ = 3dₗ、eₙ=2eₗ + dₗ

となりこれから

上記例外のn(3進数表示に0はなく最高位以外の1の数が高々1個”の場合)を除きdₙ≦2×3ᵉ⁻¹ 、eₙ≦n

となるのでeₙ=n+1(⇔aₙ=cₙ)となり得るのは上記例外値内にしかないとわかります
そして例外値においてdₙ、eₙが各桁で3通りの値しか取り得ないことは計算結果から見てとれるし、証明も簡単です(nが例外値なら最高位を落としたlも例外値であるから、例外値は“最高位を落とす”という操作で閉じている)
よって例外値ないで実際eₙ=n+1となるnは各桁1個しかでないとわかります
0936132人目の素数さん
垢版 |
2022/05/06(金) 21:23:11.73ID:+17u7Xrj
お疲れ様でした。例外としていたものもきちんと、検証下に納められているので、完全解答だと思われます。

お気づきのように、はnを三進法表記した時に、特徴が見えてきます。私が用意していた回答方針は、

・a_n + b_n + c_n = n + 1
・b_n + c_n = 2^p * 3^q
・c_n = 2^p * [3^q/2]
・ ただし、n を三進法で表現した時、使われる 1 の数を p、2 の数を qとする

を用いるものです。真ん中の式について少し説明を加えると、

a_n は C[n,k],k=0,1,2,...,n の中の3の倍数の数なので、b_n + c_n は、3で割り切れなかったものの数。
そのようなC[n,k]を素因数分解すると、3の因子の数は0ですが、
それを求める式は Σ{[n/3^i]-[(n-k)/3^i]-[k/3^i]} なので、
全ての i に対し、[n/3^i]=[(n-k)/3^i]+[k/3^i] が成立している事が、C[n,k]が3で割り切れない条件になります。

これは、(n-k) と kの和を求める計算を、三進法で行った時、「繰り上がり」が無い場合と言い換えることができます。
そのような和のパターンは、nを三進法で表現した時、使われた 1 の数をp、2の数をqとして 2^p * 3^q 通りあるというものです。

面白い見識をいただきました。ありがとうございました。
0942132人目の素数さん
垢版 |
2022/05/07(土) 19:57:50.73ID:xZyba7WB
(1) 選択肢4つの内、3つは不正解で1つが正解
(2) ランダムに選択肢を選ぶとはどの選択肢が選ばれる確率も同様に確からしいことを言う

(1)と(2)が満たされている限り正解する確率は25%

(2)を認めず推論により候補を絞った選択肢からランダムに選ぶ立場をとった場合
(1)より正解の選択肢は1つのみなのでそれが100%でなければ矛盾する

従って(1)は成り立たず、当然、4択問題として破綻する
0947イナ ◆/7jUdUKiSM
垢版 |
2022/05/09(月) 02:53:10.55ID:DnbVySQs
>>914
>>937
(c)50%
∵25%という同じ答えが選択肢に存在し、これらは正解ではない。
もし仮にどちらか一方が正解なら他方は不正解となり、同じ答えが正解かつ不正解となり矛盾するからである。
よって正解は0%と50%の2択となり、
確率は1/2すなわち50%である。
∴積極的に(c)を選んでいい。
0948132人目の素数さん
垢版 |
2022/05/09(月) 05:12:49.13ID:XWF4qCV7
正四面体にa,b,c,dを書き込んで降ったときにcが出る確率は50%であると
0949132人目の素数さん
垢版 |
2022/05/09(月) 05:14:26.91ID:XWF4qCV7
無作為に投げてcが底面になったときにcが出たと呼ぶ
0950132人目の素数さん
垢版 |
2022/05/09(月) 11:05:16.14ID:sY2KIhfu
イヤ、その問題の作者は「50%が正解」と設定したのだから実際に4面サイコロを振って"A,Dとでる確率”が“無作為に解答を選択して正解となる確率”になる、だから“A,Dとでる確率”=“50%”が実験結果になるハズでその値は問題作問者の設定した答えと矛盾してないでしよ?というのが元ネタのロジックなんじゃないの?
元ネタの文章は出てきてないから分からんけど
0951132人目の素数さん
垢版 |
2022/05/09(月) 11:21:10.00ID:OgBVtQy2
アレ、変なこと書いた
問題作問者は「Cを正解」と設定、従って“正解する事象”=“25%を選択する事象”と考えられる
ここで実際サイコロ振ってやってみると
Aの割合ー25%→25%と解答
Bの割合ー25%→0%と解答
Cの割合ー25%→50%と解答
Dの割合ー25%→25%と解答
で50%が確かに正解してるでしょというロジックなんでしょ?
つまり「選択肢」と「選ばれる解答」の間にズレができててその事を利用して“正解の選択肢を選ぶ確率”と“正解の確率を解答する確率”をずらしてるんでしょ
ちなみにオレはそのロジックを面白いとは思うけど文句つけようがないと思ってるわけじゃないよ
この手の話しムキになると無限論争にしかならんからな
0953132人目の素数さん
垢版 |
2022/05/09(月) 12:57:04.67ID:sOMWlny1
違うな
やっぱりおかしいな
スマリャンの本のロジック上がってないから分からんわ
0955132人目の素数さん
垢版 |
2022/05/09(月) 17:12:54.61ID:czLHEFz0
昔、次のような問題を見たことがある。たぶん、類題。

『 このボードには、
  1が()個、 2が()個 3が()個、 4が()個、 5が()個、
  6が()個、 7が()個 8が()個、 9が()個、 0が()個
  書かれている。
  矛盾しないように、()に数字を入れよ。

            2022年 05月 某日 加○ 一二三 出題 』

出題者名を付けたのは冗談ですが、日付次第で、複数の解がある場合も、解が無い場合もあり得る。

ロジック云々というより、くだらない問題に分類すべきと思う。
0956132人目の素数さん
垢版 |
2022/05/09(月) 17:53:26.25ID:5K97qtoQ
やっぱり問題として無理がある気がするな
どっかの図書館に原著ない?
気になる
0957132人目の素数さん
垢版 |
2022/05/09(月) 22:57:58.10ID:ByouY7Cl
>>954
有名ってのはVIPやなんJにたまにスレが立てられていろんなまとめサイトでまとめられてるって話ね
明確な答えが出ないから馬鹿が釣れてスレが伸びる
0958132人目の素数さん
垢版 |
2022/05/09(月) 23:04:33.13ID:vuiuH+n4
>>957
なんだ、答え知ってるわけじゃないのか
スマリヤンの本ではどうなってるんだろ?
0961132人目の素数さん
垢版 |
2022/05/10(火) 11:14:01.85ID:Cdz+Sk3J
そこからだったか…

ない答えを探して永遠に彷徨い続けるのもよかろう
0962132人目の素数さん
垢版 |
2022/05/10(火) 11:20:19.40ID:HrTvP1Nh
>>961
マジでコラなん?
まぁ答えはないとは思うけどミリオネアの放送の答えとスマリャンの答えはどうなってたか知りたいだけなんだけど、そもそもこの放送自体なかったって事?
どこソース?
0963132人目の素数さん
垢版 |
2022/05/10(火) 12:39:50.78ID:Cdz+Sk3J
なかった証拠を出せと言われてもな

元スレでも指摘されてることだが、文字部分の画質が違うのは画像を拡大するとわかるだろう
選択肢左のA: B: C: D: 部分に比べて選択肢の数字が鮮明すぎる
0964132人目の素数さん
垢版 |
2022/05/10(火) 12:43:14.53ID:mJD6V0X7
でもメチャクチャレスついてるやん?
「こんな放送なかったやろ」ってレス全然見当たらないけど?
ともかく幻の放送だったつてのは画像から見た自分の判断なのね
流石に放送はあったんやろ
放送自体なくてそのことが話題になってないとかありえん
0966132人目の素数さん
垢版 |
2022/05/10(火) 16:32:07.76ID:vYJIIUHj
なるほど
つまりはreditのあのスレは壮大な釣り堀って意見なわけね
ちょっとないと思うけどな
当然一人や二人は「そもそもそんな放送あったか?」ってツッコミそうじゃないか?
0967132人目の素数さん
垢版 |
2022/05/10(火) 17:13:23.11ID:zLPLLkTg
とりま情報提供っすわ
あったかどうか気になるならミリオネアのファンサイトなりを数時間程度漁ってみては
0968132人目の素数さん
垢版 |
2022/05/10(火) 18:12:48.71ID:5xo2AWhM
イヤ、流石にそこまでしてホントに放送があったかなかったか調べる気にはならんw
0970132人目の素数さん
垢版 |
2022/05/11(水) 10:34:51.40ID:g+dgVRn6
別スレにあったやつ

放物線上の異なる3点の接線l,m,nにおいてm,nの交点、n,lの交点、m,nの交点、と放物線の焦点の4点は同一円周上にある事を示せ
0971132人目の素数さん
垢版 |
2022/05/11(水) 11:27:48.39ID:A7/HQJbV
正の整数の長さを辺にもつ三角形を考える。この時、辺の長さの合計が100になるようなものはいくつあるか。
0972132人目の素数さん
垢版 |
2022/05/11(水) 12:37:39.61ID:30o3lAhW
u = b+c-a, v = c+a-b, w = a+b-cとおけばu+v+w = a+b+c = 100, u ≡ v ≡ w ≡ 0 ( mod 2 ), u,v,w>0を動く
∴ C[49,2] = 1176個
0975132人目の素数さん
垢版 |
2022/05/11(水) 13:42:38.72ID:vRWWY6CF
u = b+c-a, v = c+a-b, w = a+b-cとおけばu+v+w = a+b+c = 100, u ≡ v ≡ w ≡ 0 ( mod 2 ), u,v,w>0を動く
∴ C[49,2] = 1176個
このうちa=b,b=c,c=aの数が24個ずつ
∴1176/6 + 24/6 + 24/6 + 24/6 = 208個
0976132人目の素数さん
垢版 |
2022/05/11(水) 19:14:23.40ID:PSpgXNY3
>>975
ひたすら列挙して数えてみたw

> head(z)
[,1] [,2] [,3]
[1,] 2 49 49
[2,] 3 48 49
[3,] 4 47 49
[4,] 4 48 48
[5,] 5 46 49
[6,] 5 47 48
> tail(z)
[,1] [,2] [,3]
[203,] 31 33 36
[204,] 31 34 35
[205,] 32 32 36
[206,] 32 33 35
[207,] 32 34 34
[208,] 33 33 34

208種類と合致。
0977132人目の素数さん
垢版 |
2022/05/11(水) 19:22:49.90ID:PSpgXNY3
応用問題

正の整数の長さを辺にもつ三角形を考える。この時、辺の長さの合計が1000になるようなものはいくつあるか。
0978イナ ◆/7jUdUKiSM
垢版 |
2022/05/11(水) 22:21:33.49ID:rjn8ZqSt
>>947
>>997
{(500+667)/2}×(667-500)=1167×84=98028
二等辺三角形251〜333
83×3=249
98028-249=97779
∴97779個
0980132人目の素数さん
垢版 |
2022/05/12(木) 08:01:42.39ID:I1e7nnfK
>>977
対称含む合同なものは1つ
a≦b≦c≦a+b=1000-c≦2c
334≦c≦500
a≦b≦c≦a+b≦2b
c/2≦b≦c
c=2n-1
n≦b≦2n-1
2n-1-n+1=n
c=2n
n≦b≦2n
2n-n+1=n+1
c=334
n=167
c=335,336
n=168
c=499,500
n=250
167+1+[n=167,250](n+n+1)=168+(250-167+1)(334+1+500+1)/2=168+84×836/2=168+84×418=168+35112=35280
0981132人目の素数さん
垢版 |
2022/05/12(木) 09:41:21.14ID:Gg1x46K5
プログラムに数えさせたらこうなった。

> re=NULL
> n=1000
> for(i in 1:n){
+ for(j in i:n){
+ for(k in j:n){
+ if(i+j>k & i+j+k==n) re=rbind(re,c(i,j,k))
+ }
+ }
+ }
> tail(re)
[,1] [,2] [,3]
[20828,] 331 333 336
[20829,] 331 334 335
[20830,] 332 332 336
[20831,] 332 333 335
[20832,] 332 334 334
[20833,] 333 333 334
0982132人目の素数さん
垢版 |
2022/05/12(木) 10:33:48.03ID:YyOuPo1m
>>980
>c≦a+b=1000-c≦2c
c≦a+b-1=1000-c-1≦2c
333≦c≦499
>b≦c≦a+b≦2b
b≦c≦a+b-1≦2b-1
(c+1)/2≦b≦c
c=2n-1
n≦b≦2n-1
2n-1-n+1=n
c=2n
n+1≦b≦2n
2n-(n+1)+1=n
c=333,334
n=167
c=497,498
n=249
c=499
n=250
[n=167,249](n+n)+250=(249-167+1)(334+498)/2+250=83×832/2+250=83×416+250=34528+250=34778
0983132人目の素数さん
垢版 |
2022/05/12(木) 10:33:49.36ID:GYkOsAz8
なんでn=100の場合に数式で解いてるレスついてる問題をn=1000で解き直す必要があるの?
それで答え合わないならまだしも
( C[499,2] + 498/2 × 3 )/6 = 20833
になってるやん?
オレ問題作るときかなり時間かけてチェックとかもしてるんだよ
なんで数字100から1000にしただけの、しかも答え出てる問題でレス流してくれるん?
ちょっとは良心傷まんの?
0984132人目の素数さん
垢版 |
2022/05/12(木) 10:38:29.18ID:YyOuPo1m
>>982
>c≦a+b-1=1000-c-1≦2c
c≦a+b-1=1000-c-1≦2c-1
>333≦c≦499
334≦c≦449
167+[n=168,249](n+n)+250=167+(249-168+1)(336+498)/2+250=167+82×834/2+250=167+82×417+250=167+34194+250=34611
0985132人目の素数さん
垢版 |
2022/05/12(木) 12:03:52.19ID:/xPIl9mS
応用発展問題

正の整数の長さを辺にもつ三角形を考える。この時、辺の長さの合計が1000になるようなものは20833個ある。
[,1] [,2] [,3]
[1,] 2 499 499
[2,] 3 498 499
[3,] 4 497 499
[4,] 4 498 498
[5,] 5 496 499
[6,] 5 497 498
...
[20828,] 331 333 336
[20829,] 331 334 335
[20830,] 332 332 336
[20831,] 332 333 335
[20832,] 332 334 334
[20833,] 333 333 334
のように並べていくとき
12345個目となる三辺の数値を求めよ。
0986132人目の素数さん
垢版 |
2022/05/12(木) 12:54:03.95ID:No5Flp6y
もうやめてくれや
数学的になんの意味もないそんな並び順求めるクズ問題で頑張って用意した問題流れてまうやん?
ココはかなりみんな真面目に一生懸命問題用意してるのわからんの?
0987132人目の素数さん
垢版 |
2022/05/12(木) 12:56:44.53ID:VVdfMwmd
rを正の数とする
a[n] = 1^r - 2^r + 3^r - 4^r +...+ (-1)^(n-1) n^r
とおくとき
lim[n→∞](1/2^n)Σ[k=1,n] C[n,k] a[k] = (1-2^(1+r))ζ(-r)
を示せ
0988132人目の素数さん
垢版 |
2022/05/12(木) 13:15:52.12ID:/xPIl9mS
>>983
誤答が投稿されているよね。
決着をつけるのは列挙してのカウントだよ。
普通は検算ありがとうございます、と反応するのが「良心」のある人間だと俺は思う。
異論は認める。
俺に噛みつくより、誤答を投稿している人にアドバイスできるのがネ申なんだがね。
数学板って助言より罵倒を喜ぶクズ人間が多いというのが俺の印象。
異論は認める。
0989132人目の素数さん
垢版 |
2022/05/12(木) 13:20:02.77ID:UUXjxJYZ
>>988
いらない
消えてください
あなたにこのフレスレに書き込む水準の数学力ありません
0990132人目の素数さん
垢版 |
2022/05/12(木) 14:27:27.51ID:tqwTyXBE
>>987
aₙ(r) = 1^r - 2^r + 3^r - 4^r +...+ (-1)^(n-1) n^rをrの関数と見る
1/2^n)/ (1-2^(1+r))Σ[k=1,n] C[n,k] a[k] = ζ(-r)
を示せばよい
re(r) << 0 では明らかなので左辺がrについての正則関数になる事を示せはよい
よって左辺が局所一様コーシーである事を示せはよい
差分は
1/2^(n+1)a_(n+1)(r)
+(1/2^(n+1))Σ[k=1,n](C[n,k-1]-C[n,k])a_k(r)
で第一項は明らかに局所一様に絶対収束する
第二項はO(1/2^(n+1)(C[n,k-1]+C[n,k])n^(r+1))なのでn≧kで足し合わせるとき局所一様に絶対収束する
0991132人目の素数さん
垢版 |
2022/05/12(木) 15:05:58.58ID:l11GDzk8
>>989
誤答した人に助言もできないようなクズ人間にはなりたくないなぁ。列挙やモンテカルロでの検証は検算に役立つ。
ありがとう、と言えるのが良心をもった人間だと思う。
0993132人目の素数さん
垢版 |
2022/05/12(木) 15:17:49.59ID:0PNUXyz1
>>991
助言も何もあなた975の解答の意味すらわかってないやん?
わかってたら数値チョロチョロ変えるだけで解けてしまう>>977なんて出さないでしょ?
975レベルの解答が読めない人間はこのスレでは何にもできません
0997132人目の素数さん
垢版 |
2022/05/12(木) 15:24:50.41ID:0upEEGTC
>>994
あなた自分で正しいって確認してるやん?
そして>>977の問題出す時点でもこの解法で瞬殺できてしまうかどうか確認できるやん?
頭悪いなぁ
10011001
垢版 |
Over 1000Thread
このスレッドは1000を超えました。
新しいスレッドを立ててください。
life time: 137日 16時間 38分 52秒
10021002
垢版 |
Over 1000Thread
5ちゃんねるの運営はプレミアム会員の皆さまに支えられています。
運営にご協力お願いいたします。


───────────────────
《プレミアム会員の主な特典》
★ 5ちゃんねる専用ブラウザからの広告除去
★ 5ちゃんねるの過去ログを取得
★ 書き込み規制の緩和
───────────────────

会員登録には個人情報は一切必要ありません。
月300円から匿名でご購入いただけます。

▼ プレミアム会員登録はこちら ▼
https://premium.5ch.net/

▼ 浪人ログインはこちら ▼
https://login.5ch.net/login.php
レス数が1000を超えています。これ以上書き込みはできません。

ニューススポーツなんでも実況